You are on page 1of 226

TEST 1

BẢNG TỪ, CỤM TỪ NÂNG CAO


take the bull by the horns = to do something difficult in a brave and determined way= đối phó
với tình huống khó khăn một cách dũng cảm và mạnh mẽ.
be on the horns of a dilemma= trong tình thế tiến thoái lưỡng nan
'blow your own trumpet', = khoe khoang về thành tích và khả năng của mình (thổi kèn khen
lấy).
Summon (v) + a meeting = tổ chức một cuộc họp chính thức
Feather in one's cap = cái lông vũ cài trên mũ -> niềm tự hào của ai về những thành tựu/thành
tích mà họ đạt được.
Curtail one’s spending = cắt giảm chi tiêu
be wet behind the ears =to be young and without experience
ring in your ears = if something such as someone’s words ring in your ears, you feel that you
can still hear them after that person has stopped speaking
Feel one's ears burning = cảm thấy tai nóng bừng -> hẳn là có ai đang nói xấu mình.
Keep one’s ears open = to pay attention to what is happening, especially in order to find out
something
pipe dream = an idea or plan that is impossible or very unlikely to happen
a riot of colour ~ extremely colourful and bright
Indubitably= 1 cách chắc chắn, không thể nghi ngờ gì được
Haphazardly =in a way that does not have an obvious order or plan
Begrudgingly =unwillingly, often because you have no choice
Jovially =in a very friendly and pleasant way
"Cut and thrust" = (cuộc đánh giáp lá cà) chém và đâm (lưỡi lê) -> cụm từ này nghĩa là cuộc
cạnh tranh/tranh luận hết sức sôi nổi và ác liệt, nhất là trong giới kinh doanh (thương trường
như chiến trường).
"The ebb and flow (of something)" nghĩa là sự tăng giảm đều đặn về cường độ, thường xuyên
dao động, trầm bổng của điều gì.
the nuts and bolts=the practical facts about a particular thing, rather than theories or ideas
about it:
be part and parcel of something =to be a feature of something, especially a feature that cannot
be avoided
take the bull by the horns = to do something difficult in a brave and determined way= đối phó
với tình huống khó khăn một cách dũng cảm và mạnh mẽ.
be on the horns of a dilemma= trong tình thế tiến thoái lưỡng nan
'blow your own trumpet', = khoe khoang về thành tích và khả năng của mình (thổi kèn khen
lấy).
Summon (v) + a meeting = tổ chức một cuộc họp chính thức
Feather in one's cap = cái lông vũ cài trên mũ -> niềm tự hào của ai về những thành tựu/thành
tích mà họ đạt được.
Curtail one’s spending = cắt giảm chi tiêu
be wet behind the ears =to be young and without experience
ring in your ears = if something such as someone’s words ring in your ears, you feel that you
can still hear them after that person has stopped speaking
Feel one's ears burning = cảm thấy tai nóng bừng -> hẳn là có ai đang nói xấu mình.
Keep one’s ears open = to pay attention to what is happening, especially in order to find out
something
pipe dream = an idea or plan that is impossible or very unlikely to happen
a riot of colour ~ extremely colourful and bright
Indubitably= 1 cách chắc chắn, không thể nghi ngờ gì được
Haphazardly =in a way that does not have an obvious order or plan
Begrudgingly =unwillingly, often because you have no choice
Jovially =in a very friendly and pleasant way
"Cut and thrust" = (cuộc đánh giáp lá cà) chém và đâm (lưỡi lê) -> cụm từ này nghĩa là cuộc
cạnh tranh/tranh luận hết sức sôi nổi và ác liệt, nhất là trong giới kinh doanh (thương trường
như chiến trường).
"The ebb and flow (of something)" nghĩa là sự tăng giảm đều đặn về cường độ, thường xuyên
dao động, trầm bổng của điều gì.
the nuts and bolts=the practical facts about a particular thing, rather than theories or ideas
about it:
be part and parcel of something =to be a feature of something, especially a feature that cannot
be avoided
weigh on (one's) mind =To cause someone a lot of worry, concern, or anxiety, especially for a
long period of time.

close to sth = almost a particular amount, number, level, etc


focuse on = tập trung vào ai/ điều gì
no link to sb/ sth = không có liên kết, kết nối với
- get sth across: giải thích rõ ràng
- get the wrong end of the stick: hiểu lầm
- not make heads or tails: không hiểu
- at the back of one’s mind: cảm thấy, trực giác
- go awry = go wrong
exult in/over sth: vui vẻ

A. LISTENING (50 points):


Section 1: Complete the table below. Write NO MORE THAN THREE WORDS for each
answer. Write your answers in the corresponding numbered boxes. (10 points)

Date Event Importance for art

farmers from China built temples ornamented with (1) _______ and stone
3000 BC
carvings and statues in Bali
th artists employed by the ruling families and focused on
14 introduction of (2) _______
century epic narratives

(3) ______ establishment of Dutch East


art became expression of opposition to (4) ________
Indies Company
1920s beginning of tourism encouraged use of new materials, techniques and subjects

started to describe the (5) ______ of the Balinese people


1945 beginning of independence
– as well as the myths and legends of their history

Your answers:
1. 2. 3. 4. 5.

Section 2. You will hear a radio discussion about writing a novel. For questions 1-5, choose
the answer (A, B, C or D) which fits best according to what you hear. (10 points)
1. What does Louise say about Ernest Hemingway's advice to writers?
A. It is useful to a certain extent.
B. It applies only to inexperienced novelists.
C. It wasn't intended to be taken seriously.
D. The advice can mislead aspiring writers.
2. Louise believes that getting feedback is important when you ______
A. are experiencing a writer’s block.
B. are struggling with structuring your writing properly.
C. are unsure of the quality of your own writing.
D. finish your book and need an opinion on it.
3. Louise states that getting feedback from a colleague is important because _____
A. non-writers’ opinion can be hard to take seriously.
B. a writer can be less harsh with their criticism.
C. others can be reluctant to help with such thing.
D. a writer can understand the real purpose of your work.
4. For Louise, what does good feedback mean?
A. both general and detailed observations on the content
B. identifying problematic aspects as well as suggesting ways to overcome them
C. feedback that mostly focuses on the stylistic part of the writing
D. an extensive list of good and bad points
5. What does Louise say about the people she gets feedback from?
A. They shouldn’t be more skilled than you.
B. She prefers not to talk about her texts.
C. They exchange feedback.
D. Collaborating with them can be difficult.
(Adapted from CAE practice tests)
Your answers:
1. 2. 3. 4. 5.

Section 3. You will hear a tour guide talking to a group of tourists in New York about a visit
they will make to the Museum of Immigration on Ellis Island. Decide the following statements
are true (T) or false (F). Write your answers in the corresponding numbered boxes. (10
points)
1. The first part of the museum you go through used to be the Baggage Room.
2. In the Registry Room, immigrants had only medical check-ups.
3. They haven't made reservations for the play "Ellis Island Stories" in the Theatre 2.
4. At the Ellis Island Oral History Library you can listen to 20 recordings of people's first-hand
experiences at Ellis Island.
5. "The Peopling of America" exhibition is located in a place which used to be a ticket office.

Your answers:
1. 2. 3. 4. 5.

Section 4. For questions 1-10, listen to an authentic recording about chess and fill in the
missing information using words taken from the recording. Write NO MORE THAN THREE
WORDS. Write your answers in the corresponding numbered boxes. (20 points)
- Having existed for more than 1500 years, chess has been regarded as a tool of military strategy,
a yardstick for genius, and a metaphor for (1) _______.
- The Arab world was introduced to chess as a result of the Islamic (2) _______ in the seventh
century.
- Over time, chess was not only a tactical simulation - it was also a prolific source of (3)
_______.
- The terminology of chess was used by (4) _______ to talk about their political authority.
- The spread of chess to Asia gave rise to a great number of (5) _______.
- By 1000AD, chess served as (6) _______ for different social ranks carrying out their
corresponding duties.
- Despite being frowned upon by the Church and moralists, chess still developed rapidly and the
15th century witnessed it (7) _______ into its modern version.
- The birth of chess theory was marked by the creation of (8) _______ in which common chess
openings and endgames were analyzed.
- As formal competitive chess emerged in the 19th century, the (9) _______ of the past was
eventually eclipsed by strategic calculations.
- In a time period when chess carried a new (10) _______, it was the Soviet Union that
dominated international competition.

Your answers:
1. 2. 3. 4. 5.

6. 7. 8. 9. 10.

B. LEXICO- GRAMMAR (50 points)


Section 1. Choose the best option A, B, C, or D to complete the following sentences and write
your answers in the corresponding numbered boxes. (20 points)
1. My teacher advises me to juice _____ my presentation with more colorful illustrations.
A. down B. up C. over D. off
2. Every Christmas of my childhood was the same. My father _____ late for lunch, weighed
down with presents for the family.
A. would arrive B. had arrived
C. was arriving D. was used to arriving
3. Jack: Did you know Jim's car broke down on the highway late at night?
Jane: Unfortunately, that's a situation anyone _____.
A. might have to confront with B. might be confronted with
C. might be confronted D. might have been confronted
4. I don't like the way that Jack is always trying to _____ trouble between us.
A. dish out B. rub up C. stir up D. spark out
5. It’s time ____ so extravagant and started being a bit thriftier.
A. you will stop being B. you had stopped being
C. you stopped being D. you don’t be
6. ____ you’ve filled in the forms correctly, you shouldn’t have a problem opening a new
account.
A. Unless B. Provided that C. But for D. Supposing
7. ____ she had no experience, they left her to her own devices.
A. Despite the fact that B. However C. While D. Nevertheless
8.The different colors of _____ the different temperatures of the stars’ surfaces.
A. the stars indicate B. indicating stars
C. the indication that the stars D. stars indicating that
9. Only a few sounds produced by insects are heard by humans most of the sounds are
pitched either too low or too high.
A. in spite of B. because C. as a result of D. instead of
10. Mathematics helps meteorologists to predict the weather more accurately, to calculate the
speed of storms, and _____.
A. for the wind to blow determines B. causes the wind blowing to determine
C. to determine what causes the wind to blow D. determine the wind’s blowing
11. Would passengers please _____ from using any electronic equipment until the plane is
airborne?
A. refer B. resist C. restrain D. refrain
12. If you want to solve the problem forever, you must face a difficulty or danger boldly, that is,
you must _____.
A. pull the bull's horns B. take the bull by horns
C. blow your own horn D. be on the horns of a dilemma
13. The international conference of the Cardiological Association has been _____ in Cairo to
discuss the revolutionary discovery of doctor Gonzales from Mexico.
A. deployed B. collected C. mobilized D. summoned
14. If she wins the prize again this year, it'll be a real _____ in her cap.
A. nutshell B. gemstone C. feather D. landmark
15. When times are good, people can spend freely, but during bad times we have to _____ our
spending.
A. multiple B. disperse C. curtail D. obstinate
16. The manager hesitated to assign the job to the newcomer as he was _____.
A. wet behind the ears B. feeling your ears burning
C. ringing in your ears D. keeping your ears open
17. It is important to set goals to strive for in life; however, make sure they are attainable,
otherwise they are nothing more than just _____ dreams.
A. pipe B. cloud C. drain D. tube
18. With just a little preparation and regular feeding and watering, you can guarantee that you
have a ________ of colour throughout the summer.
A. disturbance B. riot C. demonstration D. rally
19. With a wide range of qualifications and a wealth of experience, Rosa is____ the most suitable
candidate.
A. haphazardly B. begrudgingly C. indubitably D. jovially
20. Working night shifts at the weekend is _______ of any job in hospitality.
A. cut and thrust B. ebb and flow C. nuts and bolts D. part and parcel

Your answers:
1. 2. 3. 4. 5.

6. 7. 8. 9. 10.
11. 12. 13. 14. 15.

16. 17. 18. 19. 20.

Section 2. Write the correct form of the words given in the brackets. Write your answers in the
corresponding numbered boxes. (10 points)
1. The main character in her first novel is very unconventional and is full of _______, brutal but
generous and loyal. (CONTRADICT)
2. My mom thought she’d _______ her account but suddenly she received nine notifications.
(ACTIVE)
3. There is a _________ belief that flamenco is typical throughout Spain, but that is not true.
(HOLD)
4. He left us with the ______ impression that we had been speaking to a future leader.
(ERADICATE)
5. With free-market globalization, investment funds can move _____ from the rich countries to
the developing countries. (IMPEDIMENT)
6. We've tried to anticipate the most likely problems, but it's impossible to be prepared for every
______. (EVENTUATE)
7. The organization insists that it is ______ and does not identify with any one particular party.
(POLITICS)
8. Bill Gate is a totally millionaire ______ who started his own business with no financial help at
all. (SELF)
9. Globalisation has become the ______ of governments and policy-makers across the world,
being alternately blamed and celebrated, depending on what particular action (or outcome) is
being justified. (WORD)
10. A sound dome could thus be regarded as a gigantic well-tempered spatial instrument with
_____ loudspeakers on its periphery. (DISTANT)

Your answers:
1. 2. 3. 4. 5.

6. 7. 8. 9. 10.

C. READING (50 points)


Part 1. Read the following passage and circle the best answer to each of the following
questions. Write your answers in corresponding numbered boxes. (15 points)
For those people who go out in search of adventure, a long-distance flight in a hot-air
balloon is a particularly exciting (1)_______. Indeed, a round-the-world balloon trip is widely
regarded as the ultimate challenge. One well-known adventurer, David Hemplemann-Adams
would not agree, however. Recently, he became the first man to (2)_______ the North Pole in a
hot-air balloon, a more significant (3)_______ in his eyes. Given that the distance and altitudes
(4)_______ are comparatively modest, you might wonder why the trip from Canada to the Pole,
should present such a challenge.
Part of the (5)_______ was that such a flight had not even been attempted for over a
century. In those days, such expeditions were huge events, with a nation's pride (6)_______ on
their success, and so resources were (7)_______ to them. Although he eventually managed to
secure a substantial sponsorship deal from an insurance company, Hemplemann-Adams had the
added challenge of having to raise sufficient funds for his trip.
Then, of course, he had to face major survival concerns, such as predicting the weather
(8)_______ and coping with the dangerously low temperatures. But most challenging of all was
the incredibly complex problem of navigation. As the earth's magnetic field gets stronger, only
the most sophisticated of satellite-linked navigation systems can (9)_______ that one has got to
the Pole. Without them, the chances of getting anywhere near it are extremely slim. Not to
mention an even greater problem that (10)_______ on Hemplemann-Adams' mind: getting back!

1. A campaign B prospect C motion D engagement


2. A meet B reach C attain D fulfil
3. A recognition B acquisition C achievement D realisation
4. A engaged B regarded C involved D connected
5. A appeal B beauty C charm D allure
6. A leaning B resting C waiting D standing
7. A commended B confided C confirmed D committed
8. A tendencies B conditions C circumstances D elements
9. A approve B confirm C reinforce D support
10. A pushed B stressed C pressed D weighed

Your answers:
1. 2. 3. 4. 5.

6. 7. 8. 9. 10.
Part 2. Read the following text and fill in the blank with ONE suitable word. Write your
answers in corresponding numbered boxes. (15 points)
The huge stone figures of Easter Island have beguiled explorers, researchers and the
wider world (1) ______ centuries, but now experts say they have cracked one of the biggest
mysteries: why the statues are where they are.
Researchers say they have analysed the locations of the megalithic platforms (2) ______
which many of the statues known as moai sit, as well as scrutinising sites of the island’s
resources, and have discovered the structures are typically found close (3) ______ sources of
fresh water. This led to the belief that such constructions could be tied to the abundance and
quality of the supplies.
Professor Carl Lipo from Binghamton University in New York commented that (4)
______ is important about the findings is that the statue locations seem to carry a symbolic
meaning to them while integrating into the lives of the community, rather than being a weird
ritual place. Easter Island has more than 300 megalithic platforms, (5) ______ of which might
have been made by a separate community. The first of these are believed to have been
constructed in the 13th century.
It is thought the monuments represent ancestors and were linked to ritual activity,
forming a focal point for communities, (6) ______ the reason for their locations was previously
unsolved. (7) ______ studies have suggested the sites might have been chosen because of a link
to key resources, the team says the latest research is the first attempt to scrutinise such claims.
The team (8) ______ on the east of the island, where various resources have been well mapped,
and looked at the distribution of 93 megalithic platforms constructed (9) ______ European
sailors arrived later in the 18th century.
After finding (10) ______ link to the proximity of rock used for tools or for the
monuments, they looked at whether the statues were found near other important resources:
gardens spread with stones in which crops like sweet potatoes were grown, fishing sites, and
sources of freshwater.

(Adapted from The Guardian, Easter Island statues: mystery behind their location revealed)

Your answers:
1. 2. 3. 4. 5.

6. 7. 8. 9. 10.

Part 3. Read the following passage and decide which answer (A, B, C, or D) best fits each gap.
Write your answers in corresponding numbered boxes. (15 points)
ARTISANS AND INDUSTRIALIZATION
Before 1815 manufacturing in the United States had been done in homes or shops by
skilled artisans. As master craftworkers, they imparted the knowledge of their trades to
apprentices and journeymen. In addition, women often worked in their homes part-time, making
finished articles from raw material supplied by merchant capitalists. After 1815 this older form
of manufacturing began to give way to factories with machinery tended by unskilled or
semiskilled laborers. Cheap transportation networks, the rise of cities, and the availability of
capital and credit all stimulated the shift to factory production.
The creation of a labor force that was accustomed to working in factories did not occur
easily. Before the rise of the factory, artisans had worked within the home. Apprentices were
considered part of the family, and masters were responsible not only for teaching their
apprentices a trade but also for providing them some education and for supervising their
moral behavior. Journeymen knew that if they perfected their skill, they could become
respected master artisans with their own shops. Also, skilled artisans did not work by the clock,
at a steady pace, but rather in bursts of intense labor alternating with more leisurely time.
The factory changed that. Goods produced by factories were not as finished or elegant as
those done by hand, and pride in craftsmanship gave way to the pressure to increase rates of
productivity. The new methods of doing business involved a new and stricter sense of time.
Factory life necessitated a more regimented schedule, where work began at the sound of a bell
and workers kept machines going at a constant pace. At the same time, workers were required to
discard old habits, for industrialism demanded a worker who was alert, dependable, and self-
disciplined. Absenteeism and lateness hurt productivity and, since work was specialized,
disrupted the regular factory routine. Industrialization not only produced a fundamental change
in the way work was organized; it transformed the very nature of work.
The first generation to experience these changes did not adopt the new attitudes easily.
The factory clock became the symbol of the new work rules. One mill worker who finally quit
complained revealingly about "obedience to the ding-dong of the bell—just as though we are
so many living machines." With the loss of personal freedom also came the loss of standing in
the community. Unlike artisan workshops in which apprentices worked closely with the masters
supervising them, factories sharply separated workers from management. Few workers rose
through the ranks to supervisory positions, and even fewer could achieve the artisan's dream of
setting up one's own business. Even well-paid workers sensed their decline in status.
In this newly emerging economic order, workers sometimes organized to protect their
rights and traditional ways of life. Craftworkers such as carpenters, printers, and tailors formed
unions, and in 1834 individual unions came together in the National Trades' Union. The labor
movement gathered some momentum in the decade before the Panic of 1837, but in the
depression that followed, labor's strength collapsed. During hard times, few workers were willing
to strike or engage in collective action. And skilled craftworkers, who spearheaded the union
movement, did not feel a particularly strong bond with semiskilled factory workers and unskilled
laborers. More than a decade of agitation did finally bring a workday shortened to 10 hours to
most industries by the 1850's, and the courts also recognized workers' right to strike, but these
gains had little immediate impact.
Workers were united in resenting the industrial system and their loss of status, but they
were divided by ethnic and racial antagonisms, gender, conflicting religious perspectives,
occupational differences, political party loyalties, and disagreements over tactics. For them,
the factory and industrialism were not agents of opportunity but reminders of their loss of
independence and a measure of control over their lives. As United States society became more
specialized and differentiated, greater extremes of wealth began to appear. And as the new
markets created fortunes for the few, the factory system lowered the wages of workers by
dividing labor into smaller, less skilled tasks.

1. Which of the following can be inferred from the passage 1 about articles manufactured before
1815?
A. They were primarily produced by women.
B. They were generally produced in shops rather than in homes.
C. They were produced with more concern for quality than for speed of production.
D. They were produced mostly in large cities with extensive transportation networks.

2. Which of the sentences below best expresses the essential information in the bold sentence in
the passage 2?
"Apprentices were considered part of the family, and masters were responsible not only for
teaching their apprentices a trade but also for providing them some education and for
supervising their moral behavior."
A. Masters demanded moral behavior from apprentices but often treated them irresponsibly.
B. The responsibilities of the master to the apprentice went beyond the teaching of a trade.
C. Masters preferred to maintain the trade within the family by supervising and educating the
younger family members.
D. Masters who trained members of their own family as apprentices demanded excellence from
them.

3. The word "disrupted" in the passage 3 is closest in meaning to _____.


A. prolonged B. established C. followed D. upset
4. In paragraph 4, the author includes the quotation from a mill worker in order to _____.
A. support the idea that it was difficult for workers to adjust to working in factories
B. to show that workers sometimes quit because of the loud noise made by factory machinery
C. argue that clocks did not have a useful function in factories
D. emphasize that factories were most successful when workers revealed their complaints
5. All of the following are mentioned in paragraph 4 as consequences of the new system for
workers EXCEPT a loss of _____.
A. freedom
B. status in the community
С. opportunities for advancement
D. contact among workers who were not managers

6. The phrase "gathered some momentum" in the passage 5 is closest in meaning to _____.
A. made progress B. became active C. caused changes D. combined forces

7. Which of the following statements about the labor movement of the 1800's is supported by
paragraph 5?
A. It was successful during times of economic crisis.
B. Its primary purpose was to benefit unskilled laborers.
C. It was slow to improve conditions for workers.
D. It helped workers of all skill levels form a strong bond with each year

8. The author identifies "political party loyalties, and disagreements over tactics" in
paragraph 6 as two of several factors that _____.
A. encouraged workers to demand higher wages
B. created divisions among workers
C. caused work to become more specialized
D. increased workers' resentment of the industrial system

9. The word "them" in the passage 6 refers to _____.


A. workers B. political party loyalties
C. disagreements over tactics D. agents of opportunity
10. Which of the following square brackets [A], [B], [C] or [D] best indicates where in the first
paragraph the sentence "This new form of manufacturing depended on the movement of goods
to distant locations and a centrealized source of laborers." can be inserted?

Before 1815 manufacturing in the United States had been done in homes or shops by skilled
artisans. [A] As master craftworkers, they imparted the knowledge of their trades to apprentices
and journeymen. [B] In addition, women often worked in their homes part-time, making finished
articles from raw material supplied by merchant capitalists. [C] After 1815 this older form of
manufacturing began to give way to factories with machinery tended by unskilled or semiskilled
laborers. [D] Cheap transportation networks, the rise of cities, and the availability of capital and
credit all stimulated the shift to factory production.
A. [A] B. [B] C. [C] D. [D]
(Adapted from TOEFL)
Your answers:
1. 2. 3. 4. 5.

6. 7. 8. 9. 10.

Part 4: Read the passage and do the following tasks. (15 points)

A. Charles Bolden, NASA’s administrator, averred that the robotic vehicle Curiosity will ‘blaze
a trail for human footprints on Mars’. He could be right. But there is a gulf between what is
technically feasible and what is actually achieved.
B. Neil Armstrong made his ‘one small step’ on the Moon in 1969, only 12 years after Sputnik.
Had the pace set by John F. Kennedy’s Apollo programme been sustained there would
already be footprints on Mars. But that was driven by the urge to beat the Russians; there was
no motive to sustain such huge expenditure.
C. Scientific exploration has burgeoned too. In coming decades, the entire solar system will be
explored by flotillas of miniaturized unmanned craft. Robots will mine raw materials from
asteroids and fabricate large structures. The Hubble Telescope’s successors will further
expand our cosmic vision of galaxies and nebulae.
D. But what role will humans play? There is no denying that Curiosity may miss startling
discoveries no human geologist could overlook. But robotic techniques are advancing fast –
whereas the cost gap between manned and unmanned missions remains huge.
E. The main impediment to a manned NASA programme has always been that public and
political opinion constrains it into being too risk-averse. The space shuttle failed twice in 135
launches. Although astronauts or test pilots would willingly accept this risk level, the shuttle
had been promoted as safe for civilians. So, each failure caused a national trauma and was
followed by a hiatus in the programme while costly efforts were made – with very limited
effect – to reduce the risk still further.
F. Unless motivated by pure prestige, ambitious manned missions will be viable only if they are
cut-price ventures, accepting high risks – perhaps even ‘one-way tickets’. These may have to
be privately funded; no Western government agency would expose civilians to such hazards.
G. The SpaceX company, led by the entrepreneur Elon Musk, has successfully sent a payload
into orbit and docked with the Space Station. The involvement in space projects of Mr. Musk
and others in the high-tech community with credibility and resources is surely a positive step.
H. Richard Branson will soon be lobbing people into space to experience a few minutes of
weightlessness. Within a few years private companies will offer orbital flights. Maybe after
another decade the really wealthy will be able to take a week-long trip around the far side of
the Moon – voyaging farther from Earth than anyone has been before but avoiding the
greater risks of a Moon landing and blast-off.
I. The phrase ‘space tourism’ should, however, be avoided. It lulls people into believing that
such ventures are routine and low-risk. If that becomes the perception, the inevitable
accidents will be as traumatic as those of the space shuttle. Remember that nowhere in our
solar system offers an environment as clement even as the Antarctic or the top of Everest. It
is foolish to claim, as some do, that mass emigration into space offers escape from Earth’s
problems.
J. But I believe, and hope, that some people living now will walk on Mars. Moreover, a century
or two from now, small groups of intrepid adventurers may be living there or perhaps on
asteroids quite independently from Earth. Whatever ethical constraints we impose here on the
ground, we should surely wish such pioneers good luck in genetically modifying their
progeny to adapt to alien environments.
K. This might be the first step towards divergence into a new species: the beginning of the post-
human era. And machines of human intelligence could spread still farther. Whether the long-
range future lies with organic post- humans or intelligent machines is a matter for debate.
Either way, dramatic cultural and technological evolution will continue not only here on
Earth but far beyond.

Questions 1-7: Choose the correct headings for the following paragraphs. There are more
headings than necessary.
Example: Paragraph A ii
1. Paragraph C
2. Paragraph D
3. Paragraph E
4. Paragraph F
5. Paragraph G
6. Paragraph H

For questions 8-10, do the following statements agree with the information given?

TRUE if the statement agrees with the information


FALSE if the statement contradicts the information

NOT GIVEN if there is no information on this

7. The Americans had no reason to continue spending large amounts of money on their space
programme once they had won the race to the Moon.

8. One of the advantages of robots is that they notice unusual objects which human scientists
might not see.

9. It would be wrong for future space explorers to alter their children's genes to make it
possible for them to live on other planets.

10. Whatever the evolution of the species in the future, it should remain human.

D. WRITING (50 points)


Section 1. Rewrite the following sentences using the words given.
1. The teacher knew he hadn’t explained himself clearly enough when his student
completely misunderstood him. ACROSS
=> Had the teacher ___________________________________________________ the stick.
2. After hours of effort, Tina could not work out what the question was asking her to do.
HEAD
=> However_____________________________________________________ the question.
3. I felt vaguely that something was wrong, but what was it? AWRY
=> I felt at the back __________________________________________________________
4. So completely incompetent was that secretary that we lost heaps of invoices. RANK
=> Such was _______________________________________________________________
5. No real sportsman wants to be suspected of crowing over his opponent’s misfortunes.
SOMETHING
=> Exulting _________________________________________________________________

Section 2. Graph description (20 points)


The chart below shows global sales of the top five mobile phone brands between 2009 and 2013.
Summarize the information by selecting and reporting the main features and make comparisons
where relevant.
You should write at least 150 words.
Global mobile phone sales by brand
Section 3. Essay writing (30 points)
Write an essay of 200 - 250 words on the following topic.
Some people think scientific research should focus on solving world health problems. Others
think that there are more important issues. Discuss both views and give your opinion.

TEST 2
BẢNG TỪ, CỤM TỪ NÂNG CAO
get one’s wires crossed = hiểu sai, hay lầm lẫn về điều người khác nói.
Give (one) the lowdown" = Nói cho ai biết sự thật về điều gì.
Showdown=an important argument that is intended to end a disagreement that has existed for a
long time
know-how=practical knowledge and ability
look-out = a place from which to keep watch or view landscape./ sự canh phòng, xem chừng
gifted with= having (something) as a special ability or quality
Above board = thẳng thắn, không che đậy, không giấu diếm
'on the cards = gần như chắc chắn sẽ xảy ra/diễn ra.
be the bee's knees =to be excellent or of an extremely high standard
The salt of the earth = Muối của đất -> Cá nhân hoặc nhóm người được cho là tốt lành và cao
thượng trong xã hội.
Needle in a haystack = cây kim trong đống rơm. Cụm từ này có nghĩa là một thứ gì đó rất khó
tìm kiếm và định vị hoặc rất phức tạp và khó uốn nắn, cụm này thường đi cùng với động từ
"look for", "find". Thành ngữ tương đương trong tiếng Việt là "Mò kim đáy bể".
A drop in the ocean = một giọt nước trong biển cả -> từ này chỉ việc làm rất nhỏ bé khi đặt
trong tổng thể; 'muối bỏ bể'.
Burn the midnight oil = đốt đèn đêm khuya -> nghĩa là thức khuya làm việc, nghiên cứu, học
bài... (read, study, or work late into the night).
Be at pains to do sth = to try very hard to do something
well intentioned=wanting to have good effects, but sometimes having bad effects that were not
expected
In term of = liên quan đến, xét về mặt
In response to = để đáp lại” hay “để đối phó lại” một vấn đề hay một sự vật nào đó đang hoặc
sắp xảy ra.
in reply (to something)= as a way of replying to something
with respect to=about; concerning=về
If need be -> nghĩa là nếu nó cần thiết.
may/might as well = tốt hơn nên làm điều gì đó hơn là không làm.
Come what may -> bất kể chuyện gì có thể xảy ra
culminate in/with something =>if a process culminates in or with a particular event, it ends
with that event
incorrigible liar=kẻ nói dối cố hữu( không thay đổi)
lead someone up the garden path =to deceive someone = lừa gạt, lừa dối ai
Spate = sự dồn dập, ồ ạt => spate of sth = 1 loạt, 1 tràng sự việc xảy ra trong thời gian ngắn
( thường là tiêu cực )
hypocrisy = đạo đức giả
transience = sự ngắn ngủi, sự tạm thời, sự chóng tàn
demise = sự tan rã, sụp đổ ( 1 công ty ), cái chết
a close-call là thoát chết, là tránh khỏi được một nguy hiểm
be analogous to sth = tương đồng, có điểm chung với
cross-examine=to ask detailed questions of someone, especially a witness in a trial, in order to
discover if they have been telling the truth= kiểm tra chéo
figurehead= bù nhìn, vô giá trị ( vật )
get one’s wires crossed = hiểu sai, hay lầm lẫn về điều người khác nói.
Give (one) the lowdown" = Nói cho ai biết sự thật về điều gì.
Showdown=an important argument that is intended to end a disagreement that has existed for a
long time
know-how=practical knowledge and ability
look-out = a place from which to keep watch or view landscape./ sự canh phòng, xem chừng
gifted with= having (something) as a special ability or quality
Above board = thẳng thắn, không che đậy, không giấu diếm
'n the cards = gần như chắc chắn sẽ xảy ra/diễn ra.
be the bee's knees =to be excellent or of an extremely high standard
The salt of the earth = Muối của đất -> Cá nhân hoặc nhóm người được cho là tốt lành và cao
thượng trong xã hội.
Needle in a haystack = cây kim trong đống rơm. Cụm từ này có nghĩa là một thứ gì đó rất khó
tìm kiếm và định vị hoặc rất phức tạp và khó uốn nắn, cụm này thường đi cùng với động từ
"look for", "find". Thành ngữ tương đương trong tiếng Việt là "Mò kim đáy bể".
A drop in the ocean = một giọt nước trong biển cả -> từ này chỉ việc làm rất nhỏ bé khi đặt
trong tổng thể; 'muối bỏ bể'.
Burn the midnight oil = đốt đèn đêm khuya -> nghĩa là thức khuya làm việc, nghiên cứu, học
bài... (read, study, or work late into the night).
Be at pains to do sth = to try very hard to do something
well intentioned=wanting to have good effects, but sometimes having bad effects that were not
expected
In term of = liên quan đến, xét về mặt
In response to = để đáp lại” hay “để đối phó lại” một vấn đề hay một sự vật nào đó đang hoặc
sắp xảy ra.
in reply (to something)= as a way of replying to something
with respect to=about; concerning=về
If need be -> nghĩa là nếu nó cần thiết.
may/might as well = tốt hơn nên làm điều gì đó hơn là không làm.
Come what may -> bất kể chuyện gì có thể xảy ra
culminate in/with something =>if a process culminates in or with a particular event, it ends
with that event
incorrigible liar=kẻ nói dối cố hữu( không thay đổi)
lead someone up the garden path =to deceive someone = lừa gạt, lừa dối ai
Spate = sự dồn dập, ồ ạt => spate of sth = 1 loạt, 1 tràng sự việc xảy ra trong thời gian ngắn
( thường là tiêu cực )
hypocrisy = đạo đức giả
transience = sự ngắn ngủi, sự tạm thời, sự chóng tàn
demise = sự tan rã, sụp đổ ( 1 công ty ), cái chết
a close-call là thoát chết, là tránh khỏi được một nguy hiểm
be analogous to sth = tương đồng, có điểm chung với
cross-examine=to ask detailed questions of someone, especially a witness in a trial, in order to
discover if they have been telling the truth= kiểm tra chéo
figurehead= bù nhìn, vô giá trị ( vật )
onvert sth into sth = chuyển đổi, biến đổi cái gì thành cái gì
reduce the risk of sth= giảm nguy cơ của điều gì
fatality rate = tỷ lệ tử
side-effects= phản ứng, tác dụng phụ
Switch one’s attention = chuyển đổi sự chú ý
be associated with something => If problems or dangers are associated with a particular thing
or action, they are caused by it
- might as well: dùng đưa ra lời đề nghị không mấy vui vẻ (cũng nên)
- encore: màn biểu diễn cuối
put one’s finger on sth: hiểu, nhận ra lý do
hold down: giữ được (công việc)
- down to earth: thực tế
- get a kick out of sth: vui vẻ, thích thú

A. LISTENING (50 points):

Part 1: You will hear a conversation between Louise and the owner of the video library.
Listen and complete the form below. Write NO MORE THAN ONE WORD AND/OR A
NUMBER in each gap.
Your answers:
1. 2. 3.
4. 5.

Part 2. Listen and decide whether the following statements are True (T) or False (F). Write T
or F in the given boxes.

6. Hwange is a great national park located in Africa.

7. Two hundred elephants have died during the last two months.

8. Many subsistence farmers are struggling to survive in the park area.

9. The national park is ready to cope with an environmental crisis.

10. The water pipes in the area have been fixed for two hours.

Your answers:
6. 7. 8. 9. 10.

Part 3. You will hear a discussion in which two biologists, Ian Cartwright and Angela
Sharpe, talk about conservation and the public's perception of it. Choose the answer (A, B,
C or D) which best fits according to what you hear.
11. According to Ian, why aren't people easily convinced of the importance of protecting
endangered species?

A They find it difficult to understand the concept.

B They are presented with information that is too vague.

C They do not believe human activity causes extinction.

D They think that conservationists are exaggerating the situation.

12. Angela says that eco-tourism has been successful because

A it provides locals with a long-term source of income.

B it has been properly managed by governments in developing countries.

C it encourages people to have more respect for nature.

D it is affordable for a large number of people from developed countries.

13. The story Ian tells about how economists determined the value of the environment highlights
A what a complex subject economics can be.

B the ease with which false promises are made.

C why companies get away with polluting lakes.

D how ignorant people are of the role nature plays in their lives.

14. What explanation does Angie give for people being indifferent to the destruction of the
ecosystem?

A They believe scientists will fix the problem.

B The vast majority do not suffer too much when it happens.

C They consider the exploitation of environmental resources necessary.

D They think the cost of replacing unrecognised benefits has been overestimated.

15. The species Ian refers to

A indicate how fast an ecosystem is likely to collapse.


B only live in one specific ecosystem.

C are extremely sensitive to environmental change.

D appear to be in the greatest danger of extinction.

Your answers:
11. 12. 13. 14. 15.

Part 4: You will hear a talk on education. For question 16 to 25, fill in each blank with NO
MORE THAN THREE WORDS taken from the recording. You will hear the audio TWICE.
Write your answers in the space provided.

Education is so important because it is supposed to help us get prepared for the challenges in our
life. That’s why in most of schools of all kinds 16. ______ with challenges in life is widely
studied. However, things such as talent, energy, goodwill, and 17. ______ seem to disappear. In
order to improve education it is claimed that what we need is not money but that we must pay
more attention to the real purpose of education, which is aimed to help us with: working and
18______. To address these needs two crucial subjects must be included in the curriculum. The
first one is 19. ______ since we don’t fully understand how the economy works. To help to study
this subject, maths is taught to help students how to deal with money. It is hoped that students
can get more understanding the global economy together with other terms such as leadership,
marketing and competition, cash flow, and 20. ______. Secondly students really need to study
themselves since we usually misunderstand ourselves. They should be taught a number of
concepts and helped towards their personality maps so that they can understand, among with
other issues, what type of people they are 21. ______ to go out with.

It is essential that they should also be taught 22. ______ so that they can understand which job
they are fit for. Regarding the study of relationships, there must be lessons on techniques such as
23. ______ and on kindness and forgiveness as well.

Education must not be restricted to classrooms or schools. Other forms or fields such as media
and arts are also used to help students learn what they really need to. However, the real problem
is that we fail to identify the 24. ______ of the problems that we are now facing in education that
is we have got the 25. ______

Your answers
16. __________________________ 21. __________________________

17. __________________________ 22. __________________________


18. __________________________ 23. __________________________

19. __________________________ 24. __________________________

20. __________________________ 25. __________________________

B. LEXICO - GRAMMAR (40 points)

Part 1. Choose one of the words marked A, B, C, or D which best completes each of the
following sentences. Write your answers in the corresponding numbered boxes. (20 points)

1. I'm afraid we got our _______ crossed — I thought my husband would be picking up the
children and he thought I was doing it.

A. minds B. purposes C. wires D. fingers

2. The princess's nanny's autobiography really gives the _______ on life among the royals.

A. show-down B. know-how C. low-down D. look-out

3. As well as being a good cook, my grandmother was ________ with a fine voice.

A. competent B. capable C. gifted D. talented

4. _____talking of running for election again, after such a crushing defeat, is surely proof of his
resilience.

A. Should he be B. That he is C. Had he been D. That he


were

5. Nam, remember, although money is important, never do anything that is not ______.

A. above board B. under the sky C. within the square D. in the cards

6. Since Harry is the bee’s ______ in terms of Maths, it is no wonder so many friends ask him
for support in this subject.

A. knees B. wings C. arms D. legs


7. Searching for one man in this city is like looking for a _________.
A. salt of the earth B. sand in the desert
C. needle in a haystack D. drop in the ocean
8. Denise has been ___________ the midnight oil trying to finish this report, so she must be
exhausted.

A. lighting B. brightening C. burnt D. burning

9. I was at _____________ to make it clear that I wasn’t blaming either of them.

A. efforts B. trouble C. pains D. endeavours

10. Many young people feel a sense of __________ and don’t believe they have to work hard.

A. entitlement B. aspiration C. association D. dedication


11. ______, early approaches for coping with workplace stress dealt with the problem only after
its symptoms had appeared.

A. Although well-intending B. Although it is a good intention

C. Although a gook intention D. Although well-intended


12. If the size of the work force can be easily and rapidly altered ______ market fluctuations,
profits will be maximized.

A. in terms of B. in response to C. in reply to D. with respect to


13. ______, the strollers can take another road.

A. If need be B. When it must C. We might as well D. Come what


may
14. Global warming has progressed ______ glaciers everywhere are shrinking.

A. too much an extent that B. to such an extent that

C. enough an extent that D. so great an extent that


15. My arguments with the boss got worse and worse, and it all ______ in my deciding to change
jobs.

A. elaborated B. superseded C. regenerated D. culminated

16. That Mary is an _________ liar: you must take what she says with a small grain of salt.

A. incorrigible B. incurable C. irredeemable D. irremediable


17. It seems to me that we’ve been led up the _______ path. There’s no such address as the one
she gave us!
A. forest B. garden C. maze D. mountain

18. The renewed interest in Elizabethan times is evident in the _____ of new Hollywood films
set during that period.

A. spate B. hypocrisy C. transience D. demise

19. We weren't able to drive down the street because of a ______ in the middle of the
road.

A. broadening B. border C. barricade D. buttress


20. It was a close _______ but we just made it to the airport on time for our flight.

A. run B. drive C. call D. go

Your answers
1. 2. 3. 4. 5. 6. 7. 8. 9. 10.
11. 12. 13. 14. 15. 16. 17. 18. 19. 20.

II. Give the correct form of each word in brackets to complete the sentence.

1. When my favorite team lost the big game, I was ____________ and did not leave my seat for
an hour. (MUSE)

2. For users, they are still expensive ____________ features and come with their own set of
integration problems. (ADD)

3. This proposal was ____________ to the one we discussed at the last meeting. (ANALOGY)

4. The opposing counsel had a last opportunity to ________ the witness to address the facts
brought out in redirect examination. (EXAMINE)

5. The government announced that more than a million dollars will be ________ for COVID-19
vaccine research. (EAR)

6. The president of this company is merely a ________ the Chief Executive is the one who is
truly in control. (HEAD)
7. Barack Obama is the first President of the United States with __________ background. RACE

8. This statue __________ the soldiers who died in the war. MEMORY

9. The chairman of this company is merely a __________, the Chief Executive is the one who is
truly in control. HEAD

10. The headmaster decided to expel the rough boy from our school. His behaviour was really
____________ (OBJECT).

Your answers:
1. 6.
2. 7.
3. 8.
4. 9.
5. 10.

C. READING (60 POINTS)

Part 1. Fill each of the following numbered blanks with ONE suitable word.

Economic benefits of vaccination

The costs of vaccination programmes are (1) ________ outweighed by the economic benefits of
reducing illness, disability and premature death, according to a modelling study. Patenaude and
his team at Johns Hopkins University in Baltimore, Maryland generated estimates for the
economic cost of illnesses, disability and premature death that (2) ________ otherwise occur
without vaccination programmes in 94 low and middle-income countries, and compared these
with the overall cost of (3) ________ the programmes. They focused on vaccination programmes
targeting 10 infectious diseases, (4) ________measles, yellow fever and hepatitis B.

Using a model that considered treatment costs as well as lost wages and productivity due to
illness, the researchers found that the money saved through the vaccination programmes will be
approximately $682 billion for the period from 2011 to 2020 and a (5) ________ $829 billion
from 2021 to 2030. Patenaude claimed that they wanted to convert the benefits (6) ________
money so people could compare them with other types of investments a country or organisation
might be making – like in education or transport or other things. Also, the researchers (7)
________ their findings using another model, (8) ________ estimates the value of a saved life
using data on people’s willingness to spend money to (9) ________ their risk of death. Using this
model, they found that the estimated value of lives saved by the vaccination programmes will be
about 51 times their cost from 2011 to 2020 and 52 times their cost from 2021 to 2030. As stated
by Patenaude, measles vaccination provided the highest estimated return on investment as it had
a high case fatality (10) ________ and measles vaccines were extremely effective at reducing
mortality in young children.

Your answers:
6. 6.
7. 7.
8. 8.
9. 9.
10. 10.

Part 2: Read the following passage and choose the best option to fill in each gap.

Attention Deficiency Disorder (ADD) is a neurobiological problem that affects 3-5% of all
children. (1)_______include inattentiveness and having difficulty getting organized, as well as
easily becoming (2)________. Sometimes, ADD is accompanied by hyperactivity. In these
cases, the sufferer exhibits (3)________physical activity.

Psychostimulant drugs can be (4________to ADD sufferers to assist them with the
completion of the desired thought processes, although they might cause (5)______. Current
theory states that medication is the only (6)_______action that has a sound scientific basic. This
action should only be taken after an accurate diagnosis is made.

Children with ADD do not (7)______have trouble learning; their problem is that they
involuntarily (8)________their attention elsewhere. It is not only children that are (9)_______by
this condition. Failure to treat ADD can lead to lifelong emotional and behavioral problems.
Early diagnosis and treatment, however, are the key to successfully overcoming learning
difficulties (10)_______with ADD.

1. A. Symptoms B. Sights C. Signs D. Signals

2. A. distracted B. dim C. divulged D. unattended


3. A. excessive B. rich C. exaggeratedD. over

4. A. handed B. taken C. given D. drunk

5. A. disadvantages B. damage C. pain D. side-effects

6. A. cure B. health C. remedial D. medical

7. A. absolutely B. eternally C. necessarily D. always

8. A. switch B. carry C. move D. bring

9. A. worsened B. affected C. caused D. spoiled

10. A. combined B. associated C. made D. supplied

Your answers
1. 2. 3. 4. 5.
6. 7. 8. 9. 10.

Part 3: Read the following passage about the human immune system and choose the best
answer (A, B, C, or D) according to the text. Write your answers in the corresponding
numbered boxes provided.

The Human Immune System

The human immune system is composed of both an innate and an adaptive immune
system. First, humans have an innate immune system that is intrinsic in all organisms, and it
functions particularly through establishing biological barriers and creating biochemical
reactions that immediately respond with a maximal effort in order to destroy infectious
microbes. [A] Second, humans have an adaptive immune system, which can only be found in
vertebrates with jaws. [B] The adaptive immune system gains an immunological memory from
previously encountered germs, so it is able to prevent these specific microbes from causing
further infection. [C] With these dual capacities of fighting infection and acquiring resistance to
germs, humans can maximize their immunity. [D]

A person’s innate immune system has many complex barriers and biochemical reactions
designed to ward off infections. The most visible one is the skin, which keeps most bacteria,
fungi, and viruses from ever entering the body, but humans also have mucus, which traps germs
that reside in the body’s tissue. In addition to such biological material, there are other internal
barriers like gastric acids, tears, saliva, urine, and various chemicals that either destroy or flush
out germs. Even involuntary functions like sneezing and coughing are barriers that serve to
expel germs. Beyond these, there are biochemical reactions that come from leukocytes, which
are found in the blood. Leukocytes are white blood cells that effectively clear out cellular
debris, create inflammation near an infection, summon immune cells to the inflammation,
activate several other chemical reactions, and even destroy tumors. However, perhaps the most
important action these cells perform is activating a human’s adaptive immune system, which is
essential in not only curing current diseases but also preventing future infections.

With an adaptive immune system, cells learn how to best combat pathogens and develop
a higher resistance to them. Like the innate immune system, this involves chemical reactions
and cellular cooperation. Unlike the innate immune system, this system doesn’t respond very
quickly or with its full strength all at once. Instead, it uses its time and energy to provide cells
with an immunological memory to the pathogens they encounter, making them more resistant to
recurring infections (similarly to how a vaccination works). Certain white blood cells called T-
cells are the principal actors in this system; these identify “self” cells with the same DNA and
distinguish them from any foreign cells with different DNA. After this, they seek and destroy
these foreign cells, whether they are invading microbes or infected host cells. T-cells also
mediate the responses from the innate and adaptive immune systems so that the body can
effectively exterminate the infection.

After destroying infectious cells, the body uses B-cells to develop antibodies, or
specialized proteins that prevent future infections. A B-cell is designed to connect with an
individual type of antigen created by an infectious cell. The B-cell uses this antigen to produce
antibodies that seek out and neutralize infectious bacteria, fungi, and viruses. However, the
most important process comes after the infection disappears: these B-cells will duplicate, and
their progeny will manufacture the same antibodies. Thus, the body will constantly produce
antibodies that successfully fight off a specific infection, and the body can successfully fight off
any subsequent infections from this pathogen. In addition to this, B-cells also mark antigens for
leukocytes to attack, thus making them and microbes easier targets for the biochemical reaction.

An interesting feature of the human immune system is how it affects infants both before
and after birth. When babies are first born, they do not have very many previously formed
antibodies, so they have a greater risk of infection than adults do. However, they ward off many
infections by temporarily obtaining the mother’s antibodies from breast milk and nutrients
passed through the placenta. Also interesting is the very inception of the fetus among such an
aggressive immune system: somehow, the fetus, which doesn’t have its mother’s exact DNA, is
ignored by the mother’s T-cells and B-cells. Scientists currently have a few theories about this
phenomenon. For instance, the uterus may not be monitored by white blood cells, or it may
produce special proteins that suppress any local immune responses. Nonetheless, the fact that
the immune system restrains its programming for reproductive development continues to puzzle
many scientists.

1. The word intrinsic in the passage is closest in meaning to ________.

A. fundamental B. auxiliary C. detrimental D. extraordinary


2. Which of the following square brackets [A], [B], [C], or [D] best indicates where in the
paragraph the sentence ‘However, this particular action never changes to counter specific
threats of infection’ can be inserted?

A. [A] B. [B] C. [C] D. [D]

3. According to passage 2, which bodily fluid initiates biochemical reactions in a human’s


innate immune system?

A. urine B. blood C. saliva D. tears

4. The word mediate in the passage is closest in meaning to _________.

A. interrupt B. magnify C. contemplate D. coordinate

5. Based on the information in paragraph 3, what can be inferred about the adaptive immune
system?

A. Because it takes so long to act, it is less effective in purging infectious cells than the innate
immune system.

B. Even though it takes longer to act, it is more effective in long term immunity than the
innate immune system.

C. Because humans already have an innate immune system, this system is unnecessary and
only used as a substitute.

D. It works differently from the innate immune system, so the two are completely
independent of one another.

6. The word progeny in the passage is closest in meaning to _________.

A. willingness B. mechanism C. offspring D. mutation

7. According to paragraph 4, what do B-cells produce?

A. antigens B. antibodies C. leukocytes D. pathogens

8. The word inception in the passage is closest in meaning to __________.

A. conception B. invulnerability C. contamination D. consumption

9. According to paragraph 5, where do newborn babies get most of their antibodies?

A. from their own white blood cells B. from immune cells in the uterus
C. from mucus and other barriers D. from breast milk and the placenta

10. Based on the information in paragraph 5, what can be inferred about how the mother’s
immune system should scientifically be affecting the fetus?

A. It should protect the fetus from infection.

B. It should help develop cells in the fetus.

C. It should be attacking foreign fetal cells.

D. It should be exposing the fetus to microbes.

Your answers:
1. ________ 2. ________ 3. ________ 4. ________ 5. _______
6. ________ 7. ________ 8. ________ 9. ________ 10. _______

Part 4: Read the passage and do the tasks below

A. Genealogy, the study of tracing family connections and relationships through history – so
building a cohesive family tree, has become an increasingly popular hobby from non-specialist
enthusiasts over recent decades. The introduction of the Internet has, in many ways, spurred
interest levels since historical information has been made far more accessible than previously.
Experts warn, however, that sources obtained from the internet must be considered with caution
as they may often contain inaccuracies, often advising novice genealogists to join a family
history society where they are able to learn useful skills from experienced researchers.

B. Originally, prior to developing a more mainstream following, the practice of genealogy


focused on establishing the ancestral links of rulers and noblemen often with the purpose of
disputing or confirming the legitimacy of inherited rights to wealth or position. More recently,
genealogists are often interested in not only where and when previous generations of families
lived but also details of their lifestyle and motivations, interpreting the effects of law, political
restrictions, immigration and the social conditions on an individual’s or family’s behaviour at the
given time. Genealogy searches may also result in location of living relatives and consequently
family reunions, in some cases helping to reunite family members who had been separated in the
past due to fostering/adoptlon, migration or war.

C. In Australia, there has been a great deal of interest of late, from families wishing to trace their
links to the early settlers. As a result of the loss of the American colonies in the 1700s, Britain
was in need of an alternative destination for prisoners who could not be accommodated in the
country’s overcrowded penal facilities. In 1787, the ‘First Fleet’ which consisted of a flotilla of
ships carrying just over 1300 people (of which 753 were convicts or their children and the
remainder marines, officers and their family members) left Britain’s shores for Australia. On
January 26, 1788 – now celebrated as Australia Day – the fleet landed at Sydney Cove and the
first steps to European settlement began.

D. Genealogy research has led to a shift in attitudes towards convict heritage amongst
contemporary Australian society, as family members have been able to establish that their
ancestors were, in fact, not hardened and dangerous criminals, but had, in most cases, been
harshly punished for minor crimes inspired by desperation and dire economic circumstances. So
dramatic has the shift in attitudes been that having family connections to passengers on the ‘First
Fleet’ is considered nothing less than prestigious. Convicts Margaret Dawson and Elizabeth
Thakery were amongst the first European women to ever set foot on Australian soil. Details
about the former, whose initial death sentence passed for stealing clothes from her employer was
commuted to deportation, and the latter expelled for stealing handkerchiefs along with others of
similar fate are now available on the internet for eager descendants to track.

E. Although many of the deported convicts were forbidden to return to Britain, others such as
Dawson, were, in theory, expelled for a given term. In reality, however, the costs of attempting
to return to the mother country were well beyond the means of the majority. Genealogists now
attribute the successful early development of Australia to such ex-convicts who decided to
contribute fully to society once their sentence had been served. Many rewards were available to
prisoners who displayed exemplary behaviour, including land grants of 30 acres or more, tools
for developing and farming the land and access to convict labour. Genealogy studies also show
that many former prisoners went on to hold powerful positions in the newly forming Australia
society, examples being Francis Greenway – a British architect expelled on conviction of fraud –
who went on to design many of Sydney’s most prominent colonial buildings, and Alexander
Munro, transported after stealing cheese at the age of 15, who would later build Australia’s first
gas works and hold the position of Town Mayor.

F. In North America, the Mormon Church, headquartered in Salt Lake City, Utah, holds wo
major genealogical databases, the International Genealogical Index and the Ancestral File, which
contain records of hundreds of million individuals who lived between 1500 and 1900 in the
United States, Canada and Europe. Resources available to genealogy enthusiasts include the Salt
Lake City based Family History Library and more than 4000 branches where microfilms and
microfiches can be rented for research and the newer Family Search internet site which provides
open access to numerous databases and research sources. Such data sharing practices are central
and crucial to genealogical research and the internet has proven to be a major tool in facilitating
ease of transfer of information in formats suitable for use in forums and via email. The global
level of interest in and demand for such information has proven so intense, that traffic load on
release of sources such as Family Search and the British Census for 1901 led to temporary
collapse of the host servers.

G. Experts advise that reliability of sources used for genealogical research should be evaluated in
light of four factors which may influence their accuracy, these being the knowledge of the
informant, the bias and mental state of the informant, the passage of time and potential for
compilation error. First, genealogists should consider who the information was provided by and
what he or she could be ascertained to have known. For example, a census record alone is
considered unreliable as no named source for the information is likely to be found. A death
certificate signed by an identified doctor, however, can be accepted as more reliable. In the case
of bias or mental state, researchers are advised to consider that even when information is given
by what could be considered a reliable source, that there may have been motivation to be
untruthful – continuing to claim a government benefit or avoidance of taxation, for example.

H. Generally, data recorded at the same time or close to the event being researched is considered
to be more reliable than records written at a later point in time, as – while individuals may intend
to give a true representation of events – factual information may be misrepresented due to lapses
in memory and forgotten details. Finally, sources may be classified as either original or
derivative. The latter refers to photocopies, transcriptions, abstracts, translations, extractions, and
compilations and has more room for error due to possible misinterpretations, typing errors or loss
of additional and crucial parts of the original documentation.

Questions 1 – 5

Reading Passage has eight paragraphs A-H. Choose the correct heading for paragraphs B
and D-G from the list of headings below. Write the correct number i to ix in boxes 1 – 5 on
your answer sheet.

List of Headings

i. An Embarrassing Heritage
ii. Assessing Validity
iii. Diversity of Application
iv. Interpretation Errors
v. Past Usage
vi. Useful Sources
vii. Australasian Importance
viii. Changing Viewpoints
ix. Significant Roles

Example: Paragraph C; Answer: vii

1) Paragraph B ______
2) Paragraph D ______
3) Paragraph E ______
4) Paragraph F______
5) Paragraph G ______
Questions 6 – 8

Do the following statements agree with the information given in Reading Passage?
TRUE if the statement agrees with the information
FALSE if the statement contradicts the information
NOT GIVEN if there is no information on this

6) Early applications of genealogy focused on behaviour, movement and settlement of


populations.
7) Some ex-convicts chose to stay in Australia due to the opportunities it presented.
8) Overwhelming interest in obtaining genealogical information has led to technological
difficulties.

Questions 9-10

Choose the correct letter A, B, C or D


9) Why has recreational genealogy become more popular?
A. Because it is now a fashionable hobby.
B. Because more people wish to trace missing relatives.
C. Because there are less political barriers.
D. Because it is no longer requires so much effort.

10) Why does census information need to be approached with caution?


A. Because it cannot easily be attributed to a particular individual.
B. Because it is often not validated by a physician.
C. Because administration practices in the past were unreliable.
D. Because informants may not have been truthful due to financial motivations.

D. WRITING (50 POINTS)


Part 1. Rewrite the following sentences using the words given
1. People can only enter the stadium if they have a pre-booked ticket. RESTRICTED
=> Entrance ___________________________________ possession of a pre-booked ticket.
2. There’s no point staying now that the band have played their last song. ENCORE
=> We might as_____________________________________________________________
3. It doesn’t matter how hard I try, I can’t remember where I’ve seen that guy before.
FINGER
=> Try
_____________________________________________________________________
4. The loss of his job made Phil more realistic. EARTH
=> Failure to hold ___________________________________________________________
5. Children enjoy unwrapping their Christmas presents. KICK
=> Children ________________________________________________________________
Part 2.The bar chart shows the percentage of school children learning to play different
musical instrument in 2005, 2010 and 2015.

Summarize the information by selecting and reporting the main features, and make comparisons
where relevant.

Write at least 150 words

Part 3. Write an essay on the following topics (at least 250 words).
Every year large numbers of people migrate from one country to another for different reasons.
Discuss the advantages and disadvantages of migration for the individual and for society as a
whole.

TEST 3

BẢNG TỪ, CỤM TỪ NÂNG CAO


have a ring to it = To sound appealing
"At (one's) behest" có behest là mệnh lệnh -> cụm từ này nghĩa là thực hiện theo lệnh/ý muốn
của ai.
on the part of someone/on someone's part=done or experienced by someone
as a consequence (of something)/in consequence (of something) =as a result of something =
nhue là kết quả, hậu quả của điều gì
subsequent to = at a time later or more recent than
Impair = làm yếu, làm hao mòn, thiệt hại
Dwindle = làm giảm về số lượng, kích thước, sức mạnh
Decrease = make or become smaller or fewer in size, amount, intensity, or degree.
Contract = decrease in size, number, or range.
Look/feel like death warmed over/up = trông ốm yếu, kiệt sức.
Be like a bear with a sore head' (như con gấu bị đau đầu) nghĩa là ở trong tâm trạng bực dọc,
khó chịu mà có thể đối xử tệ với người khác hoặc than vãn suốt ngày. (cau cau có có, gắt như
mắm tôm)
A wet blanket= để chỉ một người phá hoại niềm vui của người khác vì anh ta âu sầu hay bi
quan./ người làm cụt hứng
A dead duck = người hoặc vật vô dụng/vô vọng. Nghĩa khác là một người sắp gặp rắc rối to vì
hành động của mình.
rule out: loại trừ
Pull one’s weight = hoàn thành bổn phẩn, làm tròn trách nhiệm của bản thân
Take sth with a pinch of salt" = một tí muối -> nghĩa là không hoàn toàn tin vào những tin đồn
hoặc thông tin không chính xác.
In full swing = sôi nổi, náo nhiệt
Over the moon = vui sướng, hạn phúc = on cloud nine
up in the air =>If a matter is up in the air, it is uncertain, often because other matters have to
be decided first:
under a cloud (of suspicion) (bị đám mây nghi ngờ che phủ), mọi người nghĩ họ đã làm gì sai
hoặc dính líu đến việc gì đó trái pháp luật.
"A blazing row" có từ "blazing" là cháy sáng, nóng rực -> cuộc cãi vã, tranh cãi to và dữ dội.
Take (someone or something) by storm -> Chế ngự, áp đảo và tấn công ai/cái gì một cách ào ạt
và đột ngột.
Left somebody to their own devices" có device là thiết bị, phương tiện -> cụm từ này nghĩa là
để mặc ai tự xoay xở (không bị điều khiển, không cần sự giúp đỡ).
unanimous = nhất trí, hoàn toàn đồng ý
United = thống nhất, liên hợp
Undoubted = không nghi ngờ gì nữa, chắc chắn
Undivided= còn nguyên, không phân chia
"White elephant" = voi trắng -> vật vô dụng, gây phiền toái, tốn rất nhiều tiền nhưng không
thể bỏ được.
Wild goose chase là một sự đeo đuổi viển vông, một sự tìm tòi vô hy vọng.
Fat cat = mèo béo -> một người giàu có và quyền lực, thường có liên quan đến kinh doanh
hoặc chính trị. Tiếng Việt hay gọi là nhà tài phiệt.
The Black Sheep of the Family = cừu đen trong gia đình. Thông thường, một đàn cừu sẽ có
màu trắng nên khi nói ai đó là "black sheep" sẽ có ý mô tả họ là một thành viên đơn lẻ hoặc tai
tiếng nằm trong một nhóm hoặc là những đứa trẻ khác biệt trong một gia đình hoặc những
nhân tố nổi loạn trong một tập thể.
Act as a deterrent to sb = như 1 sự răn đe đối với ai
spell out =giải thích rõ ràng
stand up = đứng lên, thất hứa
take in = tiếp thu kiến thức, lừa dối
Measure out =Đo lường hoặc cân đong lượng cần thiết
Foot (meet) the bill nghĩa là thanh toán hóa đơn (to pay for something, to pay money owed).
Be pregnant with = mang thai, có con với ai
Introspection = sự tự quan sát, đánh giá điều gì
Procrastination = sự trì hoãn, dời lại
Diligence=the quality of working carefully and with a lot of effort= sự siêng năng, cần mẫn
Perfectionism= cầu toàn
have a ring to it = To sound appealing
"At (one's) behest" có behest là mệnh lệnh -> cụm từ này nghĩa là thực hiện theo lệnh/ý muốn
của ai.
on the part of someone/on someone's part=done or experienced by someone
as a consequence (of something)/in consequence (of something) =as a result of something =
nhue là kết quả, hậu quả của điều gì
subsequent to = at a time later or more recent than
Impair = làm yếu, làm hao mòn, thiệt hại
Dwindle = làm giảm về số lượng, kích thước, sức mạnh
Decrease = make or become smaller or fewer in size, amount, intensity, or degree.
Contract = decrease in size, number, or range.
Look/feel like death warmed over/up = trông ốm yếu, kiệt sức.
Be like a bear with a sore head' (như con gấu bị đau đầu) nghĩa là ở trong tâm trạng bực dọc,
khó chịu mà có thể đối xử tệ với người khác hoặc than vãn suốt ngày. (cau cau có có, gắt như
mắm tôm)
A wet blanket= để chỉ một người phá hoại niềm vui của người khác vì anh ta âu sầu hay bi
quan./ người làm cụt hứng
A dead duck = người hoặc vật vô dụng/vô vọng. Nghĩa khác là một người sắp gặp rắc rối to vì
hành động của mình.
rule out: loại trừ
Pull one’s weight = hoàn thành bổn phẩn, làm tròn trách nhiệm của bản thân
Take sth with a pinch of salt" = một tí muối -> nghĩa là không hoàn toàn tin vào những tin đồn
hoặc thông tin không chính xác.
In full swing = sôi nổi, náo nhiệt
Over the moon = vui sướng, hạn phúc = on cloud nine
up in the air =>If a matter is up in the air, it is uncertain, often because other matters have to
be decided first:
under a cloud (of suspicion) (bị đám mây nghi ngờ che phủ), mọi người nghĩ họ đã làm gì sai
hoặc dính líu đến việc gì đó trái pháp luật.
"A blazing row" có từ "blazing" là cháy sáng, nóng rực -> cuộc cãi vã, tranh cãi to và dữ dội.
Take (someone or something) by storm -> Chế ngự, áp đảo và tấn công ai/cái gì một cách ào ạt
và đột ngột.
Left somebody to their own devices" có device là thiết bị, phương tiện -> cụm từ này nghĩa là
để mặc ai tự xoay xở (không bị điều khiển, không cần sự giúp đỡ).
unanimous = nhất trí, hoàn toàn đồng ý
United = thống nhất, liên hợp
Undoubted = không nghi ngờ gì nữa, chắc chắn
Undivided= còn nguyên, không phân chia
"White elephant" = voi trắng -> vật vô dụng, gây phiền toái, tốn rất nhiều tiền nhưng không
thể bỏ được.
Wild goose chase là một sự đeo đuổi viển vông, một sự tìm tòi vô hy vọng.
Fat cat = mèo béo -> một người giàu có và quyền lực, thường có liên quan đến kinh doanh
hoặc chính trị. Tiếng Việt hay gọi là nhà tài phiệt.
The Black Sheep of the Family = cừu đen trong gia đình. Thông thường, một đàn cừu sẽ có
màu trắng nên khi nói ai đó là "black sheep" sẽ có ý mô tả họ là một thành viên đơn lẻ hoặc tai
tiếng nằm trong một nhóm hoặc là những đứa trẻ khác biệt trong một gia đình hoặc những
nhân tố nổi loạn trong một tập thể.
Act as a deterrent to sb = như 1 sự răn đe đối với ai
spell out =giải thích rõ ràng stand up = đứng lên, thất hứa
take in = tiếp thu kiến thức, lừa dối Measure out =Đo lường hoặc cân đong lượng cần
thiết
Foot (meet) the bill nghĩa là thanh toán hóa đơn (to pay for something, to pay money owed).
Be pregnant with = mang thai, có con với ai
Introspection = sự tự quan sát, đánh giá điều gì
Procrastination = sự trì hoãn, dời lại
Diligence=the quality of working carefully and with a lot of effort= sự siêng năng, cần mẫn
Perfectionism= cầu toàn
To be entitled to =Để chỉ ai đó “có quyền” hoặc “được hưởng
permit/allow + tân ngữ + to V= cho phép ai đó làm gì
be/get sucked in(to)to be unable to stop yourself from getting involved in something bad
To be/get involved in + something =Dùng để mô tả ai đó trực tiếp tham gia một công việc, sự
kiện hay một hành động nào đấy
A degree of sth = (an) amount or level of something
Immersion in sth = sự ngâm mình, sự đắm chìm
be cured with sth = chữa bằng cái gì
impact on sb/sth = ảnh hưởng, tác động lên ai/ cái gì
ineefctive at doing sth = kém hiệu quả trong làm việc gì
conduct oneself: cư xử
- result in: dẫn đến kết quả
little does/did/do sb know: không biết
- lie in store/ ahead: điều gì chờ đợi
put down roots: có cuộc sống ổn định
have a flair for sth: có tài năng trong việc gì
the odds be that: khả năng cao là

A. LISTENING (50 points)


Part 1. Complete the notes below. For questions 1-5, write NO MORE THAN TWO WORDS
AND/OR A NUMBER for each answer. (10 points)
The Official Guide to IELTS Test 3

First name: Harry


Last name: 1. ______
Date of Birth: Day: 11th; Month: December, Year: 2. ______
Type of Membership: 3. ______
Activities: Badminton and 4. ______
Payment details: Total: £450
To be paid 5. ______

Your answers:
1. 2. 3. 4. 5.

Part 2. You will hear an interview in which a deep-sea map-maker called Sally Gordon and a
marine biologist called Mark Tomkins are talking about making maps of the ocean floor. For
questions 11-15, decide whether the statement is TRUE (T) or FALSE (F). (10 points)
Cambridge English Advanced 3_Test 1

6. Sally felt excited at the prospect of making further discoveries when she had completed her first
mapping expedition.
7. Mark compares the ocean floor to the planets to emphasize how it is overexploited.
8. Sally feel optimistic about attitudes towards deep-sea exploration.
9. When talking about the territorial ambitions of some island nations, Mark reveals his concern about
the potential consequences.
10. Sally and Mark predict that future developments in deep-sea exploration result in a change in human
behaviour.
Your answers:
6. 7. 8. 9. 10.

Part 3. You will hear part of an interview with the astronaut Charles Duke, who is talking
about his trip to the moon. For questions 6-10, choose the answer (A, B, C or D) which fits
best according to what you hear. (10 points)
https://engexam.info/cae-listening-practice-tests-printfriendly/

11. How did Charles feel about space travel as a boy?


A. He thought it was unlikely to happen
B. He regarded it as more than science fiction
C. He was fascinated by the idea of it
D. He showed no particular interest in it

12. What did Charles consider to be the hardest part of the training?
A. feeling trapped in the heavy spacesuit
B. endlessly practising the lunar surface landing
C. constantly being afraid of making a mistake
D. being unable to move his arms and hands

13. What was Charles’s reaction when he first found out he was going to the moon?
A. He realised he had to be cautious
B. He felt proud to be given the opportunity
C. He tried to control his excitement
D. He reflected on his chances of survival.

14. What feature of the moon made the greatest impact on Charles?
A. the brightness of the moon
B. the vastness of the sky
C. the loneliness of the place
D. the absence of any stars

15 What does Charles feel was the most memorable part of his mission?
A. nearly falling into a crater
B. walking on the moon’s surface
C. seeing things never seen before
D. holding a piece of the moon

Your answers:
11. 12. 13. 14. 15.

Part 4. You will hear part of a scientific television programme for young people in which the
speaker explains what meteors' are. For questions 16-25, complete the notes. Write NO
MORE THAN THREE WORDS for each answer. (20 points)
Successful FCE Practice Tests – Test 8

'Meteors' is another name for 16. _______________


To help explain meteors, planet Earth is compared to a 17.__________
You can think of meteors as a group of 18.____________
In reality, meteors are very small chunks of 19.________________
The circular path the Earth travels around the Sun is called its 20.__________________
When Earth comes close to a meteor, the meteor is pulled 21.______________by gravity.
A meteor travels very fast - a hundred times faster than 22._______________
Due to the speed it travels through the air, the meteor becomes 23._________________
Because of the heat, the meteor becomes less hard, 24._________________ and then burns.
We are lucky that most meteors burn up and never 25._____________________

Your answers:

16. 21.

17. 22.

18. 23.

19. 24.

20. 25.

SECTION B. LEXICO- GRAMMAR (40 points)


Part 1. Choose the best option A, B, C, or D to complete the following sentences and write
your answers in the corresponding numbered boxes. (20 points)

1. The way the film eventually got made _____ , like the story itself, a certain ring of destiny to
it.
A. looks B. has C. indicates D. feels

2. The investigation was instigated ______ the Prime Minister.


A. on the part of B. consequence of C. subsequent to D. at the behest of

3. Do not read in such dim light; it will _________ your eyesight.


A.impair B. dwindle C. decrease D. contract
4. Did you see Jonathan this morning? He looked like ________. It must have been quite a party
last night.
A. a wet blanket B. a dead duck C. death warmed up D. a bear with a sore
head
5. The thick fog ____ out any possibility of our plane taking off before morning.
A. ruled B. struck C. stamped D. crossed
6. Everybody was busy with the spring cleaning, except Stanley, who always refused to pull his
_________.
A. socks B. weight C. fingers D. share
7. “Don’t look so worried! You should take the boss’s remarks with a ______ of salt.”
A. teaspoon B. pinch C. grain D. dose
8. The party was already _________ by the time we arrived. Everyone was singing and dancing.
A. in full swing B. up in the air C. over the moon D. under the cloud
9. It was so embarrassing. We were in the middle of a crowded restaurant when they suddenly
had a ____________ row.
A. blazing B. heated C. stormy D. smouldering
10. The young rookie scored over 20 goals in his first year, taking the whole league by
__________.
A. force B. example C. storm D. assault
11. David’s hardworking when supervised; left to his own ______, he becomes lazy.
A. means B. instruments C. tools D. devices
12. The vote on the anti-bullying policy was ______ and it will be put into effect immediately.
A. unanimous B. united C. undoubted D. undivided
13. The sports complex is likely to become a ______ after the championships are over.
A. white elephant B. wild goose C. fat cat D. black sheep
14. The installation of CCTVs across the city will hopefully act as a strong ______ to anyone
tempted to commit vandalism.
A. constraint B restriction C deterrent D boundary
15. _______ martial arts he now has considerably more free time to dedicate to his new business
venture.
A Dropping B. Dropped C. Having dropped D. Having been
dropped
16. He went to great lengths to_______ the details of the intricate plans to his co-workers.
A. take in B. spell out C. stand up D. measure
out
17. In the end, we decided to ____ the bill for the party.
A. head B. leg C. arm D. foot
18. Mary is rumored to be pregnant _____ Jay’s child.
A. to B. by C. with D. within
19. Exercise can be classified as active or passive with the former _____ effort and the latter the
use of machines or training assistants.
A. involves physical B. physics is involved C. involving physical D. physically
involved
20. Jackson needs to put limits to his _____ otherwise, nothing will ever be good enough for
him.
A. introspection B. procrastination C. diligence D. perfectionism

Your answers:
1. 2. 3. 4. 5.

6. 7. 8. 9. 10.

11. 12. 13. 14. 15.

16. 17. 18. 19. 20.

Part 2. Write the correct form of the words given in the brackets. Write your answers in the
corresponding numbered boxes. (10 points)
21. They are composed of algae and fungi which ____________ to satisfy the needs of the
lichens.
(UNITY)
22. His behaviour in his father's presence caused his ____________ and his sister ended up
inheriting the whole family fortune. (INHERIT)
23. The ____________ of any manned mission to the planets will be increased if a secure
fuel supply can be found beforehand. (SURVIVAL)
24. There was a heavy ______ yesterday afternoon which completely ruined the church
Garden Party. (POUR)
25. A strong Scots background and thirty years at Carlyle Rural School had made her an
expert ____________. (DISCIPLINE)
26. There was a distinct danger that the second day of the trail was going to be a/an ________
disaster. (MITIGATE)
27. When going trekking, I tend to stagger along in silence, ____________ of energy being my
main priority. (CONSERVE)
28. When we finally reached our camp, I was overcome by relief and ______________joy. I
might have missed a lot, but I had reached my destination. (ADULTERATE)
29. I was well-aware of the ______________ of my eyes-to-the-ground climbing technique.
(ABSURD)
30. His visit to De Gaulle may or may not lay_____________for a Franco-American
rapprochement. (GROUND)
Your answers:
21. 22. 23. 24. 25.

26. 27. 28. 29. 30.

C. READING (60 points)


Part 1. Read the following passage and decide which answer (A, B, C, or D) best fits each gap.
Write your answers in corresponding numbered boxes. (15 points)

Gap years are quite common in many parts of the world and most young people, upon
leaving high school, feel (1)____ to one. It's plain to see how the idea would be (2)______;
taking a year off from studies to travel the world and consider your future sound like bliss.
Adverts for gap years contain (3)_____ , that read ‘The best year of my life' and ‘Total
adventure, Totally rewarding', and offer the newly (4 ______ student the opportunity to learn
more about themselves while learning about the world.
Of course, a gap year shouldn't be (5)________ as just a time to party, and as attractive as it
may sound, one must not get (6) ______ in by that notion. One way to make the most of this time
is to get involved (7) some inspiring voluntary work abroad. There is more than a (8)_______ of
truth in the idea that (9)_______ in a new culture will teach you more about yourself than any
classroom ever would. It will allow you to reach a level of emotional (10)______ that will stay
with you for a lifetime.
1. A permitted B. entitled C. designated D. allowed
2 A teasing B. touting C. tempting D. taunting
3. A editions B. billboards C. jingles D. captions
4. A emancipated B. liberated c. independent D. sovereign
5. A dismissed B. denied C. denounced D. dissuaded
6. A pulled B. sucked C. forced D. swept
7. A in B. on C. at D. about
8. A mark B. grade C. degree D. notch
9. A immersion B. diversion C. compulsion D. emersion
10. A wisdom B. ripeness C. adulthood D. maturity

Your answers:
1. 2. 3. 4. 5.
6. 7. 8. 9. 10.

Part 2. Read the following passage and fill in the blank with ONE suitable word. Write your
answers in corresponding numbered boxes. (15 points)
Although there are many strains of antibiotic bacteria now present in hospital wards, antibiotics
have effectively served (11)______ original purpose over the course of the past eighty years.
They have been able to treat the infections of countless individuals and saved millions of lives.
Antibiotics have changed the way in (12)______ many common diseases are viewed.
(13)______ infected with bacterial pneumonia, for instance, is no longer considered fatal.
Rather, it is viewed as a mundane infection which can be cured (14)______ a simple course of
antibiotics. The number of antibiotics available for use has also affected their impact (15)______
society. Even if one antibiotic is ineffective (16)______ treating a disease, there are, for most
common infections, a host of (17)______ drugs that can be used to effectively cure the disease.
The development of antibiotics over the past eighty years has changed the relationship between
humans and disease. Antibiotics have given humans the power to fight back effectively
(18)______ microorganisms in a way that (19)______ have been considered impossible just a
century (20)_____.

Your answers:
11. 12. 13. 14. 15.
16. 17. 18. 19. 20.

Part 3. Read the following passage and circle the best answer to each of the following
questions. Write your answers in corresponding numbered boxes. (15 points)
Successful FCE Tests – Test 7
The 20th century was a time of remarkable change. In less than 100 years, the population went
from around 2 billion to close to 6; almost treble the number of people living in the world today
as did ten or so decades ago. Not only have our numbers exploded, but our lives have become
more intertwined than ever. For most of human history, different communities which existed
lived in their own very small worlds inside of a bigger world they knew little about. The only
world that mattered was the one you could see in your immediate surroundings. Compared with
today, when even the poorest parts of sub-Saharan Africa can boast 43 television sets per
thousand people. The world view is no longer limited to the horizon; it stretches across the
planet. The global village is here. Now, let's see how it came about.

The lessons of two world wars in quick succession signalled the dawning of a new age.
Statesmen and women saw that the way forward lay in bringing the world closer. World War
Three was to be avoided at all costs. It was believed that by making nations more interdependent,
the risk of conflict would be lessened as it would be in nobody's interest to go to war.

That desire to see the nations of the world united gave birth to the United Nations (U.N.). The
idea was to share power, responsibility and decision-making for world affairs equally between all
members of the new global village, so it is the nearest thing we have ever had to a world
government. The U.N. brings together officials from 185 member states to preserve world peace
and prevent conflict, but the dream never quite became a reality as this body has very little 'real'
power - it just does a lot of talking. Not long after the U.N. was founded, Europe started to play
with the idea of uniting its own continent. After all, it was internal conflict being the main cause
of both world wars.
Line 21--- Then, in 1957, the idea took shape; it started as the European Coal and Steel
Community with six member states. Today, we know it as the European Union (E.U.)- 27
countries, called member states, united in one large free trade area and committed to supporting
each other to make Europe a safer, more secure and more prosperous place. 15 of those members
have gone a step further and created a single currency. The system is hardly perfect, but at least
the members are working together and not trying to destroy each other anymore.

But, for all the political movement that took place, there was a revolution more powerful, yet
more simple, that changed the world - and that was the dawn of the information age. First the
television brought people from opposite sides of the globe into contact; then the Internet makes
the world our living room. Technology was the most powerful tool for uniting people in the last
century, and the first to create a truly global community. Now we can communicate with people
from different 'tribes' in an instant; debate with, learn from, understand, chat with them. But for
all the change, have we made the world any better? There's still a huge gap between the richest
and the poorest nations and misunderstanding and conflict. We may be closer and live in a global
village; but there's still a lot more to do.

21 .The number of people living in the world ______


A. has almost trebled since a decade ago.
B. has more than trebled in just under 100 years.
C. has risen to more than 6 million.
D. rose tremendously during the twentieth century.

22. What does the writer mean by saying communities used to live in worlds inside of a
bigger world?
A. In the past people knew little about faraway places.
B. In the past people only cared about themselves.
C. Most people didn't travel very much in the past.
D. Most people cared about what was happening in the bigger world.

23. What changed after the experience of two world wars?


A. Politicians felt determined to prevent another world war.
B. Information technology brought the world closer together.
C. Nobody was interested in conflict anymore.
D. Nations wanted to become more independent.

24. What is suggested about the United Nations?


A. It keeps the world peaceful and conflict-free.
B. It will become a global government.
C. It doesn't have a lot of meaningful influence.
D. It is controlled by a few big powers.

25. What does the phrase 'took shape' mean in the context of paragraph 4, line 21?
A. succeeded
B. developed
C. concluded
D. changed

26. The E.U. is now comprised of ______


A. 6 member states.
B. 15 member states.
C. 27 member states.
D. 15 member and 27 associate states.

27. The arrival of new technology and the information age _______
A. seemed unimportant compared to the political changes taking place.
B. had a strong impact on the opposite side of the globe.
C. brought people together in a way that politicians could not.
D. saw people use the internet a lot in their living rooms.

28. What does the writer's tone in the final paragraph suggest?
A. He is satisfied with what has been achieved.
B. He is critical and pessimistic about the future.
C. He is confused and upset.
D. He is realistic about the situation.

29. What does the author mean by “the Internet makes the world our living room”?
A. people can access huge sources of information thanks to the Internet
B. people can enjoy the convenience brought about by the Internet
C. people can watch a lot of TV shows thanks to Internet connection
D. people can purchase furnitures for their homes on the Internet

30. What is the best title for the passage?


A. How the U.N. and E.U. came into being
B. A century of significant changes
C. Technology is the foundation for growth
D. The problem of overpopulation

Your answers:
21. 22. 23. 24. 25.
26. 27. 28. 29. 30.

Part 4: Read the following passage and choose the best answer to each of the following
questions. Write your answers in corresponding numbered boxes. (15 points)
A
One misguided legacy of over a hundred years of writing on bilingualism1 is that children’s .
intelligence will suffer if they are bilingual. Some of the earliest research into bilingualism
examined whether bilingual children were ahead or behind monolingual2 children on IQ tests.
From the 1920s through to the 1960s, the tendency was to find monolingual children ahead of
bilinguals on IQ tests. The conclusion was that bilingual children were mentally confused.
Having two languages in the brain, it was said, disrupted effective thinking. It was argued that
having one well-developed language was superior to having two half-developed languages.
B

The idea that bilinguals may have a lower IQ still exists among many people, particularly
monolinguals. However, we now know that this early research was misconceived and incorrect.
First, such research often gave bilinguals an IQ test in their weaker language – usually English.
Had bilinguals been tested in Welsh or Spanish or Hebrew, a different result may have been
found. The testing of bilinguals was thus unfair. Second, like was not compared with like.
Bilinguals tended to come from, for example, impoverished New York or rural Welsh
backgrounds. The monolinguals tended to come from more middle class, urban families.
Working class bilinguals were often compared with middle class monolinguals. So the results
were more likely to be due to social class differences than language differences. The comparison
of monolinguals and bilinguals was unfair.
C
The most recent research from Canada, the United States and Wales suggests that bilinguals are,
at least, equal to monolinguals on IQ tests. When bilinguals have two well- developed languages
(in the research literature called balanced bilinguals), bilinguals tend to show a slight superiority
in IQ tests compared with monolinguals. This is the received psychological wisdom of the
moment and is good news for raising bilingual children. Take, for example, a child who can
operate in either language in the curriculum in the school. That child is likely to be ahead on IQ
tests compared with similar (same gender, social class and age) monolinguals. Far from making
people mentally confused, bilingualism is now associated with a mild degree of intellectual
superiority.
D
One note of caution needs to be sounded. IQ tests probably do not measure intelligence. IQ tests
measure a small sample of the broadest concept of intelligence. IQ tests are simply paper and
pencil tests where only ’right and wrong ’answers are allowed. Is all intelligence summed up in
such right and wrong, pencil and paper tests? Isn’t there a wider variety of intelligences that are
important in everyday functioning and everyday life?
E
Many questions need answering. Do wc only define an intelligent person as somebody who
obtains a high score on an IQ test? Are the only intelligent people those who belong to high IQ
organisations such as MENSA? Is there social intelligence, musical intelligence, military
intelligence, marketing intelligence, motoring intelligence, political intelligence? Are all, or
indeed any, of these forms of intelligence measured by a simple pencil and paper IQ test which
demands a single, acceptable, correct solution to each question? Defining what constitutes
intelligent behaviour requires a personal value judgement as to what type of behaviour, and what
kind of person is of more worth.
F
The current state of psychological wisdom about bilingual children is that, where two languages
are relatively well developed, bilinguals have thinking advantages over monolinguals.Take an
example. A child is asked a simple question: How many uses can you think offer a brick? Some
children give two or three answers only. They can think of building walls, building a house and
perhaps that is all. Another child scribbles away, pouring out ideas one after the other: blocking
up a rabbit hole, breaking a window, using as a bird bath, as a plumb line, as an abstract
sculpture in an art exhibition.
G
Research across different continents of the world shows that bilinguals tend to be more fluent,
flexible, original and elaborate in their answers to this type of open-ended question. The person
who can think of a few answers tends to be termed a convergent thinker.They converge onto a
few acceptable conventional answers. People who think of lots of different uses for unusual
items (e.g. a brick, tin can, cardboard box) are called divergers. Divergers like a variety of
answers to a question and are imaginative and fluent in their thinking.
H
There are other dimensions in thinking where approximately ’balanced’ bilinguals may have
temporary and occasionally permanent advantages over monolinguals: increased sensitivity to
communication, a slightly speedier movement through the stages of cognitive development, and
being less fixed on the sounds of words and more centred on the meaning of words. Such ability
to move away from the sound of words and fix on the meaning of words tends to be a
(temporary) advantage for bilinguals around the ages four to six. This advantage may mean an
initial head start in learning to read and learning to think about language

For questions 31-36, choose the correct heading for paragraphs B-G from the list of headings
below.

List of Headings
i No single definition of intelligence
ii Faulty testing, wrong conclusion
iii Welsh research supports IQ testing
iv Beware: inadequate for selling intelligence
v International research supports bilingualism
vi Current thought on the advantage bilinguals have
vii Early beliefs regarding bilingualism
viii Monolinguals ahead of their bilingual peers
ix Exemplifying the bilingual advantage

Example: Paragraph A: vii


31. Paragraph B
32. Paragraph C
33. Paragraph D
34. Paragraph E
35. Paragraph F
36. Paragraph G
For questions 37-40, do the following statements agree with the information given?
TRUE if the statement agrees with the information
FALSE if the statement contradicts the information
NOT GIVEN if there is no information on this
37 Balanced bilinguals have more permanent than temporary advantages over monolinguals.
38 Often bilinguals concentrate more on the way a word sounds than on its meaning.
39 Monolinguals learn to speak at a younger age than bilinguals.
40 Bilinguals just starting school might pick up certain skills faster than monolinguals.
Your answers:
31. 32. 33. 34. 35. 36. 37. 38. 39. 40.

D. WRITING (50 points)


Part 1. Rewrite the following sentences using the words given
1. His behavior at the conference gave him the bad reputation he now has. CONDUCTED
=> The way _____________________________________ in the bad reputation he now has.
2. She does not have the faintest idea about what will happen to her when she takes the
test. STORE
=> Little____________________________________________________________________
3. When they got to Texas, they settled successfully and built a life. ROOTS
=> When they arrived________________________________________________________
4. Thanks to her good upbringing, she is really good at languages. FLAIR
=> If it ____________________________________________________________________
5. Daniel stands a very good chance of debuting with his teammates. ODDS
=> The ____________________________________________________________________

Part 2. Provided below is a table demonstrating information about the subway systems in six
cities.
In about ____ words, summarize the information by selecting and reporting the main
features, and make comparisons where relevant. (20 points)
Part 3. Essay writing. (30 points)
Write an essay of about 250 words to express your opinion on the following topic:
Parents should get punishment in some ways if their children break the law.
Do you agree with this opinion?
Use your own knowledge and experience to support your arguments with examples and relevant
evidence

TEST 4
BẢNG TỪ, CỤM TỪ NÂNG CAO

So as to /in oder to + to V = to V = in order for sb/sth to do sth = để làm gì


- put/ tighten the screws on sb: gây áp lực lên ai để buộc họ phải làm gì
- get into gear/ get sth into gear: bắt đầu hoạt động/ làm việc một cách nghiêm túc
- put one over on sb: thuyết phục ai tin điều gì là đúng
- wipe sth off the map: xóa bỏ/ phá hủy hoàn toà
Cấu trúc đảo ngữ; Not only + be/axiliary + S + adj/n/v, but + S + also + V = Không những…
mà còn
In the light of = because of = bởi vì, vì cân nhắc cái gì
be at/in/to the forefront (of something) =to be in a leading position in an important activity that
is trying to achieve something or develop new ideas
on the verge of: sắp, gần, suýt, trên bờ vực ( tuyệt chủng, phá sản …)
in accordance with : in a way that agrees with or follows (something, such as a rule or request)
Intriguingly= in a manner that arouses one's curiosity or interest; fascinatingly.= hấp dẫn, gây
tò mò
Intrusively = làm phiền, quấy rầy, làm cho ai cảm thấy khó chịu
intrinsically = in an essential or natural way.= về cơ bản, bản chất
intrepidly= gan dạ, dũng cảm
Grudgingly agree = miễn cưỡng đồng ý
ill-mannered= không lịch sự, bất lương, thô lỗ
immaterial= phi vật chất, không quan trọng
impertinent=not showing proper respect; rude.= xấc xược
inapposite = không phù hợp, không thích hợp
hit the headlines=to appear in the news suddenly or receive a lot of attention in news reports =
được phổ biến rộng rãi
gimmick ( it is public knowledge that new magazines often use free gifts or other gimmicks to
get people to buy them) =quảng cáo lừa người
snares = bẫy lưới thường dùng để bắt động vật
plot = âm mưu, mưu đồ
scam = sự lừa đảo, không trung thực
'there are plenty more fish in the sea', điều đó có nghĩa là có nhiều lựa chọn khác trong một
tình huống. Câu này thường được nói để an ủi ai đó khi vừa trải qua tan vỡ quan hệ tình cảm
(biển còn nhiều cá).
excitable: dễ hưng phấn, dễ bị kich động =responding rather too readily to something new or
stimulating; too easily excited.
Admonish sb to do sth = to advise someone to do something= khuyên ai đó nên làm gì
in the red: mắc nợ, nợ tiền, nợ nần, không có khả năng thanh toán
to be in the pink: (từ lóng) rất sung sức, rất khoẻ.
out of the blue= bất thình lình, bất ngờ
over the moon= rất vui sướng, hạnh phúc với điều gì
So as to /in oder to + to V = to V = in order for sb/sth to do sth = để làm gì
- put/ tighten the screws on sb: gây áp lực lên ai để buộc họ phải làm gì
- get into gear/ get sth into gear: bắt đầu hoạt động/ làm việc một cách nghiêm túc
- put one over on sb: thuyết phục ai tin điều gì là đúng
- wipe sth off the map: xóa bỏ/ phá hủy hoàn toà
Cấu trúc đảo ngữ; Not only + be/axiliary + S + adj/n/v, but + S + also + V = Không những…
mà còn
In the light of = because of = bởi vì, vì cân nhắc cái gì
be at/in/to the forefront (of something) =to be in a leading position in an important activity that
is trying to achieve something or develop new ideas
on the verge of: sắp, gần, suýt, trên bờ vực ( tuyệt chủng, phá sản …)
in accordance with : in a way that agrees with or follows (something, such as a rule or request)
Intriguingly= in a manner that arouses one's curiosity or interest; fascinatingly.= hấp dẫn, gây
tò mò
Intrusively = làm phiền, quấy rầy, làm cho ai cảm thấy khó chịu
intrinsically = in an essential or natural way.= về cơ bản, bản chất
intrepidly= gan dạ, dũng cảm
Grudgingly agree = miễn cưỡng đồng ý
ill-mannered= không lịch sự, bất lương, thô lỗ
immaterial= phi vật chất, không quan trọng
impertinent=not showing proper respect; rude.= xấc xược
inapposite = không phù hợp, không thích hợp
hit the headlines=to appear in the news suddenly or receive a lot of attention in news reports =
được phổ biến rộng rãi
gimmick ( it is public knowledge that new magazines often use free gifts or other gimmicks to
get people to buy them) =quảng cáo lừa người
snares = bẫy lưới thường dùng để bắt động vật
plot = âm mưu, mưu đồ
scam = sự lừa đảo, không trung thực
'there are plenty more fish in the sea', điều đó có nghĩa là có nhiều lựa chọn khác trong một
tình huống. Câu này thường được nói để an ủi ai đó khi vừa trải qua tan vỡ quan hệ tình cảm
(biển còn nhiều cá).
excitable: dễ hưng phấn, dễ bị kich động =responding rather too readily to something new or
stimulating; too easily excited.
Admonish sb to do sth = to advise someone to do something= khuyên ai đó nên làm gì
in the red: mắc nợ, nợ tiền, nợ nần, không có khả năng thanh toán
to be in the pink: (từ lóng) rất sung sức, rất khoẻ.
out of the blue= bất thình lình, bất ngờ
over the moon= rất vui sướng, hạnh phúc với điều gì
irrevocable = not able to be changed, reversed, or recovered; final.= không thể thay đổi
differentiate = recognize or ascertain what makes (someone or something) different.= phân
biệt
self-evaluation= tự đánh giá
personified =represent (a quality or concept) by a figure in human form.= nhân hóa
unshrinkable = không thể co lại, rút lại
outspoken = frank in stating one's opinions, especially if they are critical or controversial.
depauperate = Làm nghèo đi, bần cùng hoá. Làm mất sức, làm suy yếu
impoverished= made weaker or worse in quality, very poor
foolhardy = dại dột, liều lĩnh 1 cách ngu ngốc
over-manned= having more employees than are needed
So as to /in oder to + to V = to V = in order for sb/sth to do sth = để làm gì
- put/ tighten the screws on sb: gây áp lực lên ai để buộc họ phải làm gì
- get into gear/ get sth into gear: bắt đầu hoạt động/ làm việc một cách nghiêm túc
- put one over on sb: thuyết phục ai tin điều gì là đúng
- wipe sth off the map: xóa bỏ/ phá hủy hoàn toà
Cấu trúc đảo ngữ; Not only + be/axiliary + S + adj/n/v, but + S + also + V = Không những…
mà còn
In the light of = because of = bởi vì, vì cân nhắc cái gì
be at/in/to the forefront (of something) =to be in a leading position in an important activity that
is trying to achieve something or develop new ideas
on the verge of: sắp, gần, suýt, trên bờ vực ( tuyệt chủng, phá sản …)
in accordance with : in a way that agrees with or follows (something, such as a rule or request)
Intriguingly= in a manner that arouses one's curiosity or interest; fascinatingly.= hấp dẫn, gây
tò mò
Intrusively = làm phiền, quấy rầy, làm cho ai cảm thấy khó chịu
intrinsically = in an essential or natural way.= về cơ bản, bản chất
intrepidly= gan dạ, dũng cảm
Grudgingly agree = miễn cưỡng đồng ý
ill-mannered= không lịch sự, bất lương, thô lỗ
immaterial= phi vật chất, không quan trọng
impertinent=not showing proper respect; rude.= xấc xược
inapposite = không phù hợp, không thích hợp
hit the headlines=to appear in the news suddenly or receive a lot of attention in news reports =
được phổ biến rộng rãi
gimmick ( it is public knowledge that new magazines often use free gifts or other gimmicks to
get people to buy them) =quảng cáo lừa người
snares = bẫy lưới thường dùng để bắt động vật
plot = âm mưu, mưu đồ
scam = sự lừa đảo, không trung thực
'there are plenty more fish in the sea', điều đó có nghĩa là có nhiều lựa chọn khác trong một
tình huống. Câu này thường được nói để an ủi ai đó khi vừa trải qua tan vỡ quan hệ tình cảm
(biển còn nhiều cá).
excitable: dễ hưng phấn, dễ bị kich động =responding rather too readily to something new or
stimulating; too easily excited.
Admonish sb to do sth = to advise someone to do something= khuyên ai đó nên làm gì
in the red: mắc nợ, nợ tiền, nợ nần, không có khả năng thanh toán
to be in the pink: (từ lóng) rất sung sức, rất khoẻ.
out of the blue= bất thình lình, bất ngờ
over the moon= rất vui sướng, hạnh phúc với điều gì
irrevocable = not able to be changed, reversed, or recovered; final.= không thể thay đổi
differentiate = recognize or ascertain what makes (someone or something) different.= phân
biệt
self-evaluation= tự đánh giá
personified =represent (a quality or concept) by a figure in human form.= nhân hóa
unshrinkable = không thể co lại, rút lại
outspoken = frank in stating one's opinions, especially if they are critical or controversial.
depauperate = Làm nghèo đi, bần cùng hoá. Làm mất sức, làm suy yếu
impoverished= made weaker or worse in quality, very poor
foolhardy = dại dột, liều lĩnh 1 cách ngu ngốc
over-manned= having more employees than are needed
Shed = (often used in newspapers) to get rid of something you do not need or want// to lose a
covering, such as leaves, hair, or skin, because it falls off naturally, or to drop something in a
natural way or by accident => shed pounds = giảm cân
Scrape = to remove an unwanted covering or a top layer from something, especially using a
sharp edge or something rough//to succeed in getting or achieving something, but with
difficulty or by a very small amount
Shred = xé ra từng miếng, từng mảnh nhỏ
Crack = break or cause to break without a complete separation of the parts.= làm nứt
Run a risk of doing sth = tự dẫn thân đến với sự nguy hiểm và dẫn đến kết cục rất xấu/kết quả
thất bại.
Pile on the pounds" nghĩa là tăng cân (nhanh).
Crave for sth = khao khát, mong ước thứ gì
resist the urge to do sth = chống, cưỡng lại sự thôi thúc làm gì
pride yourself on sth/doing sth=to feel satisfaction at a quality or skill that you have= cảm thấy
tự hào về điều gì
be known as sb/sth = được biết đến như là ai/ cái gì
Go off without a hitch" có hitch là bế tắc, khó khăn -> cụm từ này nghĩa là diễn ra tốt đẹp,
suôn sẻ, trôi chảy.
by profession = as your profession or job
on occasion =sometimes but not often
to do one's best: làm hết sức mình.
- on the verge of: sắp
- be kept in the dark: không biết, không được nói cho biết
- be prone to: dễ bị ảnh hưởng xấu bởi
- be deluged with sth: nhận được rất nhiều

I. LISTENING (50 points)


PART 1: Complete the notes below. Write NO MORE THAN THREE WORDS AND/OR
A NUMBER for each answer. (10 points)
Example
Company name: Burnham Coaches
Name of client: (1) ______________________
Address: Down Language School
Down House
(2) ______________________
Brighton
Contact number: the school secretary to be contacted on 01273 512634
Pick up time: 7:30
Return time: (3) ______________________
Purpose of Hire: Visit to Stonehenge and Bath
Question 4 and 5
Answer the questions below.
Write NO MORE THAN THREE WORDS for each answer.
4. What does the cost of hiring the coach include?
_____________________________________________________
5. Who does the customer have to speak to before confirming the booking?
_____________________________________________________
Your answers:

1. 2. 3. 4. 5.

PART 2: Listen to the recording and decide whether the statements are True (T) or False
(F). You will listen TWICE. (10 points)
1. The Flirtey delivery service is still waiting for government approval.
2. The drone uses GPRS to fly to the customer’s location.
3. The drone can deliver hot food such as pizza.
4. The company hopes to complete local deliveries with 10 minutes.
5. The drone are powered by solar panels.
Your answers:

1. 2. 3. 4. 5.

PART 3: You will ear part a radio programme in which two people, Sally White and
Martin Jones, are discussing the popularity of audio books (books recorded on tape), and
the problems involved with abridging books before taping them. From question 1 – 5,
choose the correct answer A, B, C, or D. You will hear the recording TWICE. (10 points)
1. Sally believes that most people listen to audio books
A. When they’re doing housework
B. before they go to sleep
C. On the way to work
D. when they’re with their children

2. Sally feels that the main advantage of audio books is that they
A. encourage children to read more
B. are read by experienced actors
C. make more books accessible to children
D. save parents from having to read to children
3. Martin says that in the USA there is a demand for audio books because people there
A. were the first to obtain audio books
В. have to drive long distances
C. feel that they do not have time to read books
D. are used to listening to the spoken word on the radio

4. According to Sally, successful abridgements depend on


A. the length of the original
B. their closeness to the original
C. the style of the author
D. the type of story
5. Books are not commisioned specifically for the audio market because
A. writers are too busy working for the BBC.
B. such books have failed in the past.
C. there are not enough people buying audio books.
D. people only want familiar stories.

Your answers:
1. 2. 3. 4. 5.

PART 4: Complete the summary with one word or number from the lecture in each gap.
(20 points)
The lecture covers some of the latest research and the main arguments involved in the Nature
versus nurture debate; or how much your personality is determined by genetics, and how much
by your (1) ______________________.
Some scientists believe that genes don’t determine by personality directly, but rather provide (2)
______________________. It is discovered that 35,000 genes each of us carries affect
personality, such as impulsivity, (3) ______________________ and addition to some substances.
For instance, a malfunction in the speech gene FOXP 2 can lead to (4)
______________________. On the other hand, scientists who believe that (5)
______________________ shapes personality disagree on who influences children more:
parents or peers and friends. Children acquire their parent’s habits by (6)
______________________, while the need to be part of a group comes from a primitive (7)
______________________.
The latest research suggests that both factors have influence on an individual to approximately
the same extent: (8) ______________________ % by nature, (9) ______________________ by
nurture.
From the research, we'll be in a better position to (10) ______________________ how to change
someone's personality.
Your answers:
1. 2. 3. 4. 5.

6. 7. 8. 9. 10.

III. LEXICO – GRAMMAR (40 points)

PART 1: Choose the word / phrase which best complete each sentence. (20 points)
1. You story is interesting and lively, but it contains several historical inaccuracies. For example,
your hero___________ have offered Miss Swinton shelter under his umbrella, as they weren’t
invented until a hundred years later.
A. couldn’t B. might not C. shouldn’t D. wouldn’t
2. The coach worked long and hard into the night ___________ the team’s strategy for the next
game.
A. for preparing B. for the preparation of
C. to prepare D. in order for proper preparation of
3. The government needs to ______ businesses that have been trying to evade the tax.
A. put the screws on B. get into gear C. put one over on D. wipe off the map
4. Not only ___________ more brittle than hard maples, but they are also less able to withstand
high winds.
A. soft maples are B. they are soft maples C. are soft maples D. soft maples
5. ___________ that dynamic, departmental recommendations against using pepper spray on
nonviolent arrestees may be needed.
A. in light of B. At the forefront of C. On the verge of D. In accordance with
6. It is mandatory that smoking in public ___________.
A. prohibited B. prohibit C. be prohibited D. is prohibited
7. You may borrow as many books as you like, provided you show them to ___________ is at
the desk.
A. whoever B. who C. whom D. anyone
8. My dog’s being very difficult at the moment; I don’t know what the trouble is but he just
___________ not eat.
A. shall B. will C. may D. has
9. The Red Cross is ___________ an international aid organization.
A. intriguingly B. intrusively C. intrinsically D. intrepidly
10. ___________ Jim’s support, I wouldn’t have got the job.
A. As for B. But for C. Except for D. Just for
11. ___________ the initial fears of the Commander-in-Chief, the campaign was a resounding
success.
A. Despite B. Although C. Besides D. Whereas
12. My sister showed great ___________ in selling her house when she did because soon
afterwards the market fell dramatically.
A. premonition B. forecasting C. prediction D. foresight
13. She ___________ agreed to go with him to the football match although she had no interest in
the game at all.
A. apologetically B. shamefacedly C. grudgingly D. discreetly
14. You shouldn’t have been ___________ to your elders by raising those matters.
A. ill-mannered B. immaterial C. impertinent D. inapposite
15. Once the story ___________ the headlines, everyone was talking about it.

A. crashed B. struck C. smashed D. hit

16. It is public knowledge that new magazines often use free gifts or other ___________ to get
people to buy them.
A. gimmicks B. snares C. plots D. scams
17. I know you're upset about breaking up with Tony but there are plenty more ___________.
A. horses in the stable B. cows in the field C. tigers in the zoo D. fish in the sea
18. Lauren is often labelled easy-going as she tends to appear mild and relaxed rather than tense
and ___________.
A. sullen B. likeable C. humorous D. excitable
19. Gabriel’s mentor ___________ him to change his dishonest ways before it was too late.
A. dislodged B. notified C. admonished D. informed
20. It was such a shock to receive a letter like that ___________.
A. in the red B. out of the blue C. in the pink D. over the moon
Your answers:

1. 2. 3. 4. 5. 6. 7. 8. 9. 10.

11. 12. 13. 14. 15. 16. 17. 18. 19. 20.

PART 2: WORD FORM Supply the correct form of the word provided in blankets in each
sentence. (10 points)
1. You won’t persuade him to change his mind. His decision is ___________ (REVOKE)
2. Politeness is one thing. Real kindness is another. You must learn to ___________ between the
two. (DIFFERENT)
3. In Scotland there is greater emphasis on ___________ by individual school. (VALUE)
4. The sun and the moon are often ___________ in poetry. (PERSON)
5. Many fabrics are specially treated so as to be ___________ (SHRINK)
6. Jim is one of the most ___________ members of the committee. (SPEAK)
7. The fauna is becoming comparatively ___________ due to the isolation, young geological age
of
the island and forestry activities within habitats. (PAUPER)
8. Heavy rain and excessive use have the soil____________. (POVERTY)
9. He made a ____________ attempt to climb the tree to recover his kite. (FOOL)
10. Management decided the office was ____________ and sacked three junior typists. (MAN)

Your answers:

1. 2. 3. 4. 5.

6. 7. 8. 9. 10.

III. READING (60 points)


PART 1: Read the following passage and decide which answer (A, B, C, or D) best fits each
gap. Write your answers in corresponding numbered boxes. (15 points)
THE MEDITERRANEAN WAY

There is more evidence proving the Mediterranean diet to be heart healthy and most likely
helpful in (1)_____________ pounds and keeping them off. Although Mediterranean cuisine is
not technically low in fat, the type of (2) _____________acids it does contain, such as those that
come from nuts and olive oil, is a much healthier form of (3) _____________ fat than what
exists in other foods. In a recent study, participants who were overweight and suffering from (4)
_____________were placed on either a regular low-fat diet or a Mediterranean diet. The ones on
the latter diet fared better than their counterparts in terms of health risks. The results showed that
it is not just a matter of eating low-fat (5) _____________ of food, but it is the addition of nuts
and olive oil that improves the indicators of good health. The ones who followed the regular low-
fat diet also (6) _____________ the risk of returning to old habits that caused them to (7)
_____________on the pounds in the first place. A Mediterranean diet is easier to follow because
it is filled with healthy yet (8) _____________ foods such as fish, walnuts and fresh fruit and
vegetables. Those who follow the diet are less likely to suffer from (9) _____________for less
healthier foods, thus making it easier to (10) _____________ the urge to eat meat and high-fat
dairy products. Ultimately, the Mediterranean diet
contains all the nutrients needed for lifelong health, making it a diet people can live on for their
entire lives.
1. A. scraping B. shredding C. shedding D. cracking
2. A. fattening B. fat C. fatty D. fattened
3. A. diet B. dieting C. dietician D. dietary
4. A. obesity B. disorder C. malnutrition D. indigestion
5. A. toppings B. servings C. coverings D. flavourings
6. A. packed B. ran C. played D. quenched
7. A. stuff B. stack C. place D. pile
8. A. sharp B. bitter C. bland D. appetising
9. A. cravings B. passions C. urges D. appetites
10. A. whip B. resist C. polish D. feed
Your answers:
1. 2. 3. 4. 5.
6. 7. 8. 9. 10.

PART 2: Read the text below and think of the word which best fits each space. Use only
ONE word in each space. Write your answers in the corresponding numbered boxes. (15
points)
THEATRICAL GHOSTS

Britain has many haunted theatres. The Theatre Royal in Drury Lane, London, prides
(1)_____________on being regarded as the most haunted theatre (2) ______________ showbiz.
The most legendary ghostly inhabitant is an 18th-century gentleman known
(3)______________the ‘Man in Grey’. Spotted by dozens of cleaners over the years, a sighting
of him is usually considered to be good luck and indicates the show will go (4) ______________
a hitch. It is thought that he is associated with a skeleton uncovered by workmen in the theatre in
1843. The skeleton was found with a knife in (5) ______________ ribs.

There is also the ghost of an Irish man Charles Macklin, an actor (6) ______________
profession. He stabbed a fellow actor, Thomas Hallam, in 1735. Macklin’s ghost has (7)
______________seen in a backstage corridor. Joseph Grimaldi was a famous clown who
performed at the Theatre Royal for many years. When he died in 1837, he left a request that he
be buried near the theatre. Since then, his ghost appears during shows (8) ______________
occasion, sometimes sitting in the audience, smiling encouragement up at performers on the
stage. Every once in a while, performers (9) ______________ felt his presence on stage with
them. Some have spoken of feeling his hands guiding them to a better spot on stage, or even
kicking them in the rear if they weren’t (10) ______________ their best!
Your answers:
1. 2. 3. 4. 5.
6. 7. 8. 9. 10.

PART 3: Read the following passage and choose the best answer to each of the following
questions. Write your answers in corresponding numbered boxes. (15 points)

The tide means the rising and falling of ocean levels that are affected by the moon, the sun, and
the earth’s own rotation. Although the sun gives about 200 times the gravitational force on the
earth as the moon does, the moon exerts more than twice the tidal force. As the closer
astronomical body, the moon has greater influence on the earth’s tides. Although other factors do
affect the tide, the positions of these two bodies have by far the greatest determinant roles.

It is easy to imagine that the ocean level closest to the moon bulges upward. However, a bulge
occurs on the opposite side of the earth as well. [A] As the moon’s tidal force counteracts the
earth’s gravitational force, water bulges on both sides. Because there are two major oceanic
bulges, there are two high tides in a daily cycle. [B] A lunar day, not a solar day, must be used in
this calculation. It takes the earth 24 hours and 48 minutes to make one rotation. As with a solar
day, the earth rotates once in 24 hours. [C] Thus, high tides generally occur every 12 hours and
24 minutes, an interval of half a lunar day. [D] Exact intervals will vary depending on local land
features.
The sun has a similar but less visible effect on water levels. Oceanic bulges occur on the sides of
the earth. While these bulges resulting from the sun’s tidal forces are smaller than the greatest
swells caused by the moon, they can amplify the moon’s effect. Furthermore, the bulges in water
levels caused by the sun can make predicting the tides tricky, since both the sun and the moon
must be taken into consideration.

When the moon and the sun are aligned – that is, when both are on the same side of the earth or
on opposite sides of the earth – the oceanic bulges are at their greatest size. Known as spring
tide, this powerful phenomenon results in abnormally high tides, abnormally low tides, and
abnormally strong tidal currents. In general, seven days after the spring tide comes, the neap
tide comes. The results are weaker tidal currents and less variation in water levels between
high tide and low tide.

Because the moon and the sun are major factors in tide formation, researchers have tried to
investigate these factors under assumptions of the earth with no land masses, smooth ocean
floors, constant distances from the moon and the sun, and an unchanging environment. Even
under these simplified imaginary conditions, it has been difficult to construct complete and
conclusive models of tidal phenomena. With the additional factors of huge land masses, wildly
variable underwater terrain, the elliptical orbits of the moon and the earth, and climate change,
tidal analysis become increasingly more complex.

Land masses block tidal ebbs and flows, affecting the timing, volume, and strength of the tides.
Submarine terrain also has great effects on tides and their movements and variation. As for the
principal astronomical bodies, one must remember that the moon’s orbit around the earth and the
earth’s orbit around the sun are elliptical. Therefore, tidal forces vary with changing relative
distances between the earth and the moon and between the earth and the sun. Finally, weather
and climate change always pose challenges to tidal prediction. Now, with global warming, rising
ocean levels, and weather phenomena, study of the tides is becoming more challenging than ever
before.

1. According to the passage, all of the following are true of tides EXCEPT
A. The earth’s rotation affects water depth.
B. Solar gravitational force is the biggest influence on tidal flow.
C. The moon is a more important factor than the sun.
D. Tidal force is related to distance from the earth.

2. The word “counteracts” in paragraph 2 is closest in meaning to ___________.


A. stops B. intensifies C. offsets D. resists

3. Look at the four squares [A], [B], [C], [D] that indicate where the following sentence could be
added to the passage.
However, the moon is constantly moving around the earth, so an additional 48 minutes is
required for the moon to complete the rotation.
Where would the sentence best fit? Choose the square where the sentence should be added to the
passage.
A. [A] B. [B] C. [C] D. [D]

4. The word “they” in paragraph 3 refers to ___________.


A. bulges B. forces C. swells D. bodies

5. The word “abnormally” has been repeated three times in order to ___________.
A. emphasize the power of spring tides
B. accentuate unpredictability of tidal movement
C. note how infrequently spring tides occur
D. highlight the complexities of tidal formation

6. Which of the following best expresses the essential information in the bold sentence in
paragraph 4?
A. Neap tide features weaker water currents and less difference between high and low
tide levels.
B. Lower current speeds result in little change in water levels between high tide and low
tide.
C. As a result, neap tide is difficult to detect due to little tidal variation and slow-moving
tidal currents.
D. The effects of these tidal bulges are less water movement between tides.

7. In the passage, what does the author say about simplified conditions?
A. They are unnecessary for a complex understanding of tides.
B. They have been inadequate factors in tidal models.
C. Even they have not helped scientists make models of tides.
D. They were created to study major factors in tidal behavior.

8. Which of the following is NOT mentioned in the passage as something that complicates study
of the tides?
A. Land formations change the way tidal waters move.
B. Underwater land formations affect changes in tidal levels.
C. The orbit between the sun and the earth affects tidal phenomena.
D. Increasing water depths may be causing unusual weather.

9. The word “volume” in paragraph 6 is closest in meaning to ___________.


A. duration B. sound C. size D. direction

10. An introductory sentence for a brief summary of the passage is provided below.
“Several factors make the tide a complicated phenomenon.”
Which of the following DOES NOT belong in the summary?
A. The moon is the major cause of high and low tides.
B. The sun exerts much more gravitational force than the moon does.
C. The moon and the sun create conditions for spring tide and neap tide.
D. Land formations, orbital irregularities, and climate change complicate tidal study.
Your answers:
1. 2. 3. 4. 5.
6. 7. 8. 9. 10.

PART 4: Read the passage including seven paragraphs and do the following tasks. (10 pts)
Task 1. The text has seven paragraphs, A-G
Choose the correct heading for each paragraph from the list of headings below.
Write the correct number, i-viii, in boxes 1-5 below.
List of Headings
i Different accounts of the same journey
ii Bingham gains support
iii A common belief
iv The aim of the trip
v A dramatic description
vi A new route
vii Bingham publishes his theory
viii Bingham’s lack of enthusiasm

Paragraphs Your answers:


Paragraph A iv
1. Paragraph B ………….
2. Paragraph C ………….
3. Paragraph D ………….
4. Paragraph E ………….
5. Paragraph F ………….
Paragraph G iii

The Lost City


An explorer’s encounter with the ruined city of Machu Picchu, the most famous icon of the
Inca civilisation
A
When the US explorer and academic Hiram Bingham arrived in South America in 1911, he was
ready for what was to be the greatest achievement of his life: the exploration of the remote
hinterland to the west of Cusco, the old capital of the Inca empire in the Andes mountains of
Peru. His goal was to locate the remains of a city called Vitcos, the last capital of the Inca
civilisation. Cusco lies on a high plateau at an elevation of more than 3,000 metres, and
Bingham’s plan was to descend from this plateau along the valley of the Urubamba river, which
takes a circuitous route down to the Amazon and passes through an area of dramatic canyons and
mountain ranges.

B
When Bingham and his team set off down the Urubamba in late July, they had an advantage over
travellers who had preceded them: a track had recently been blasted down the valley canyon to
enable rubber to be brought up by mules from the jungle. Almost all previous travellers had left
the river at Ollantaytambo and taken a high pass across mountains to rejoin the river lower down,
thereby cutting a substantial corner, but also therefore never passing through the area around
Machu Picchu.

C
On 24 July they were a few days into their descent of the valley. The day began slowly, with
Bingham trying to arrange sufficient mules for the next stage of the trek. His companions
showed no interest in accompanying him up the nearby hill to see some ruins that a local farmer,
Melchor Arteaga, had told them about the night before. The morning was dull and damp, and
Bingham also seems to have been less than keen on the prospect of climbing the hill. In his book
Lost City of the Incas, he relates that he made the ascent without having the least expectation that
he would find anything at the top.

D
Bingham writes about the approach in vivid style in his book. First, as he climbs up the hill, he
describes the ever-present possibility of deadly snakes, “capable of making considerable springs
when in pursuit of their prey”; not that he sees any. Then there’s a sense of mounting discovery
as he comes across great sweeps of terraces, then a mausoleum, followed by monumental
staircases and, finally, the grand ceremonial buildings of Machu Picchu. “It seemed like an
unbelievable dream … the sight held me spellbound …” he wrote.

E
We should remember, however, that Lost City of the Incas is a work of hindsight, not written
until 1948, many years after his journey. His journal entries of the time reveal a much more
gradual appreciation of his achievement. He spent the afternoon at the ruins noting down the
dimensions of some of the buildings, then descended and rejoined his companions, to whom he
seems to have said little about his discovery. At this stage, Bingham didn’t realise the extent or
the importance of the site, nor did he realise what use he could make of the discovery.

F
However, soon after returning it occurred to him that he could make a name for himself from this
discovery. When he came to write the National Geographic magazine article that broke the story
to the world in April 1913, he knew he had to produce a big idea. He wondered whether it could
have been the birthplace of the very first Inca, Manco the Great, and whether it could also have
been what chroniclers described as “the last city of the Incas”. This term refers to Cilcabamba,
the settlement where the Incas had fled from Spanish invaders in the 1530s. Bingham made
desperate attempts to prove this belief for nearly 40 years. Sadly, his vision of the site as both the
beginning and end of the Inca civilisation, while a magnificent one, is inaccurate. We now know
that Vilcabamba actually lies 65 kilometres away in the depths of the jungle.

G
One question that has perplexed visitors, historians and archaeologists alike ever since Bingham,
is why the site seems to have been abandoned before the Spanish Conquest. There are no
references to it by any of the Spanish chroniclers – and if they had known of its existence so
close to Cusco they would certainly have come in search of gold. An idea which has gained wide
acceptance over the past few years is that Machu Picchu was a moya, a country estate built by an
Inca emperor to escape the cold winters of Cusco, where the elite could enjoy monumental
architectrue and spectacular views. Furthermore, the particular architecture of Machu Picchu
suggests that it was constructed at the time of the greatest of all the Incas, the emperor Pachacuti
(c.1438-71). By custom, Pachacuti’s descendants built other similar estates for their own use,
and so Machu Picchu would have been abandoned after his death, some 50 years before the
Spanish Conquest.

Task 2. Do the following statements agree with the information given in the passage?
Write
True if the statement agrees with the information
False if the statement contradicts the information
Not Given if there is no information on this

6. Bingham went to South America in search of an Inca city.


7. Bingham chose a particular route down the Urubamba valley because it was the most common
route used by travellers.
8. Bingham returned to Machu Picchu in order to find evidence to support his theory.
Task 3. Complete the sentences below.
Choose ONE WORD ONLY from the passage for each answer.
9. The track that took Bingham down the Urubamba valley had been created for the
transportation of __________.
10. Bingham found out about the ruins of Machu Picchu from a __________ in the Urubamba
valley.
Your answers:
1. …………… 2. …………… 3. …………… 4. …………… 5. ……………
6. …………… 7. …………… 8. …………… 9. …………… 10. ………..…

IV. WRITING (50 points)


PART 1: Rewrite the following sentences using the words given
1. There is grave concern about confirmed cases of cholera that originated in the
makeshift shelters. ARE
=> Of _____________________________________________________________________
2. Simon is rumoured to be about to become a Sky News presenter. VERGE
=> Rumour__________________________________________________________________
3. She knew nothing about the party that they were planning. DARK
=> She was _________________________________________________________________
4. She may run into additional problems if she doesn't have any proper insurance. PRONE
=> She ___________________________________________ she has any proper insurance.
5. The company received an enormous number of calls responding to the advert.
DELUGED
=> The company ________________________________________response to the advert.

Part 2. GRAPH DESCRIPTION (20 pts)


The chart below gives information on the percentage of British people taking holidays at
home, abroad, or both, or no holidays for the years 2014 and 2015.
Summarise the information by selecting and reporting the main features and make comparisons
where relevant.
Write at least 150 words.

PART 3: ESSAY WRITING (30 points)


As mass communication and transport continue to grow, societies are becoming more and
more alike leading to a phenomenon known as globalization. Some people fear that
globalization will inevitably lead to the total loss of cultural identity.
To what extent do you agree or disagree with this statement?
Give reasons for your answer and include any relevant examples from your own knowledge or
experience. Write at least 250 words.

TEST 5
BẢNG TỪ, CỤM TỪ NÂNG CAO

not get/have a wink of sleep; not sleep a wink =không chợp mắt được, không ngủ tí nào

Distinctively =in a way that is easy to recognize because of being different from other things
Decisively =quickly, effectively, and confidently
Flatly=>If you flatly deny, refuse, or disagree with something or someone, you do it
completely or in a very clear and firm way
Totally= completely; absolutely= hoàn toàn
Aplenty = dồi dào, vô số Inexhaustibly = vô tận, vô hạn

Profusely = nhiều, vô khối superabundant= thừa thãi, rất dồi dào

ups and downs = thăng trầm trong cuộc sống

A school of fish = a large number of fish or other sea creatures swimming in a group
A flock of sth =a large group of people, usually people of a particular type => a flock of
tourists//a group of sheep, goats, or birds => a flock of sheep/goats/geese
A colony of => a group of animals, insects, or plants of the same type that live together=> a
colony of ants/termites/bacteria
A pack of = a group of animals, such as dogs, that live and/or hunt together => a pack of
wolves
Be an integral part of= là một phần quan trọng của điều gì
Inward = on or towards the inside// toward the inside of something, or toward your self, your
mind, or your spirit
Inset = (of an object) having something positioned within it
Internal = trong thâm tâm, trong cơ thể, trí óc
save your breath = used to say that it is not worth talking to someone because they will not
listen to you = đừng thừa hơi, phí lời vời người không đáng.
intervene: xen vào, can thiệp => mang nghĩa tích cự
interact: ảnh hưởng lẫn nhau
interfer...with: can thiệp vào, quấy rầy, gây trở ngại => mang nghĩa tiêu cực
intercept:chắn, chặn đứng
controversy surrounding/ over sth = tranh cãi xung quanh cái gì
Out of this world = bên ngoài thế giới này -> nghĩa là tuyệt vời, không còn lời để diễn tả.
Above the law = bên trên luật pháp -> ý nói việc không cần phải tuân thủ luật pháp.
Take in = tiếp thu kiến thức, lừa dối, làm quần áo nhỏ hơn
Put off = trì hoãn, ngưng thích điều gì
Give up = ngừng làm gì, đầu hàng, ngừng cố gắng // Cắt đứt mối quan hệ với
Put down = ngừng giữ/ giết con gì vì nó bị ốm
At a loose end = rảnh rỗi
low-end =Low-end products are the cheapest in a group of products//intended for people who
do not have a lot of money to spend
at a dead end = a situation that is unlikely to be successful or to make any progress= tại điểm
bế tắc, không thể tiến bộ hơn được nữa
At (one's) wit's end -> Vô phương kế, khi không biết cách làm gì thì sẽ gây ra tình trạng ức
chế và tức giận.
-In view of (something) = trong cái nhìn của -> bởi vì cái gì, xem xét cái gì.
-within/in sight of something =in the area where you can see something / in a position where
you will soon be able to get something or achieve something
Be in the mood for/to (something) = khao khát, mong muốn điều gì/làm gì. Ngược lại "be in
no mood for (something) = cảm thấy chán nản, phiền phức phải làm gì.
To be in agreement with sb = Đồng ý với ai, đồng tính với ai về điều gì
In sequence = theo thứ tự, trật tự
By accident = without intending to, or without being intended= ngẫu nhiên
at random: ngẫu nhiên, tình cờ, hú hoạ.
on impact =at the moment when one thing hits another
Conjure up =Gợi lên cái gì đó (như một bức tranh) trong tâm trí//Tạo hoặc làm cái gì đột ngột,
không báo trước

to square the circle (biến tròn thành vuông) có nghĩa là tìm giải pháp tốt cho một vấn đề dường
như không thể thực hiện được, đặc biệt khi những người liên quan có yêu cầu hay ý kiến khác
nhau.
Chew the fat = quây quần tán dóc.
wave/show/fly the flag = to show support for the country, group, or organization that you
belong to
Put the lid on = đỉnh cao của, là cực điểm của; gây ra sự thất bại của cái gì, ngăn cản sự gia
tăng/phát triển của cái gì.
Stock up = To incite or intensify negative emotions or reactions to something.
flash up =To flash very brightly and suddenly. // Of a computer notification or message, to
appear on the screen very suddenly.
Prognosticate = báo trước, tiên đoán các sự việc sẽ xảy ra trong tương lai
Sidetrack = cause (someone) to be distracted from an immediate or important issue.
at stake: thua, đang lâm nguy, đang bị đe doạ
at close quarters/range =(from) a short distance away
fair-weather friend =someone who is a good friend when it is easy to be one and who stops
being one when you are having problems
all-round =used to say that a person has many different types of skills and abilities
thought-provoking =making you think a lot about a subject
rely on sb: tin tưởng vào
(to) show reaction to sth: phản ứng lại
good at (adj): giỏi cái gì
Intrusion into = xâm phạm ( thường là đời tư của ai )
Secretive = (of a person or an organization) inclined to conceal feelings and intentions or not
to disclose information.
Reticent = kín đáo, ít nói, thận trọng
Confidential =thông tin mật, chuyện riêng tư
Classified = phân loại
An end in itself" = Một sự kết thúc bên trong chính nó -> Điều mà tự nó đã là quan trọng,
không liên quan đến sự quan trọng của những điều khác hay không mang lại một giá trị lớn lao
cho điều khác nữa.
No/small/little wonder = Dùng khi muốn diễn đạt một điều gì đó không gây ngạc nhiên và
đương nhiên là như vậy.

- hold one’s ground: giữ vững vị trí


- be (put) down to: nguyên do là
- many a/ an: biết bao nhiêu
- be thrown on the scrap heap: bị vứt bỏ - takeover (n): sự mua lại, tiếp quản
- the only fly in the ointment: con sâu làm rầu nồi canh

A. LISTENING (50 points):


Part 1. Complete the notes below. Write NO MORE THAN THREE WORDS AND/ OR A
NUMBER for each answer. (10 points)

ENQUIRY ABOUT BOOKCASES


Number of bookcases available: two
Both bookcases
Width: (1) _______________________________
Made of: (2) _______________________________
First bookcase
Cost: (3) _______________________________
Colour: (4) _______________________________
Number of shelves: six (four are (5) _______________________________)
Your answers:
1. 2. 3. 4. 5.
Part 2. Listen to part of a tutorial between two students and their tutor. The students are doing
a research project to do with computer use. Listen and decide whether the following sentences
are True (T) or False (F). Write your answers in the corresponding numbered boxes provided.
1. Sami and Irene decided to do a survey about access to computer facilities because no one has
investigated it before.
2. Sami and Irene had problems with the reading for their project because not much had been
written about the topic.
3. Sami and Irene get the main data in their survey from observation of students.
4. The tutor suggests that one problem with the survey was limitation in the number of students
involved.
5. 77% of students surveyed thought that a booking system would be the best solution.
Your answers:
1. 2. 3. 4. 5.
Part 3. You will hear a radio interview with the gardening experts Jed and Helena Stone.
Choose the answer A, B, C, or D which fits best according to what you hear. (10 pts)
1. How does Helena feel about the use of Jed's name for their joint business?
A. Occasionally frustrated that her contribution goes unnoticed.
B. Amused that they have a name people tend to remember.
C. Appreciative of the respect that the name has brought her.
D. Irritated by the fact that Jed is more of a celebrity than she is.
2. What is Jed's attitude to his public profile?
A. He likes the fact that complete strangers often want to talk to him.
B. He's unhappy that it prevents him from doing everyday activities.
C. He enjoys it more now than he did when he was younger.
D. He's proud of the way it reflects his achievements.
3. How did Helena feel about her work on The Travel Show?
A. She would have enjoyed it more in different circumstances.
B. It was convenient for her to be away from the house then.
C. It was a welcome alternative to manual work.
D. She felt obliged to do it at that particular time.
4. What explanation does Helena give for the name of the garden?
A. It provided a useful framework for the project.
B. It was a response to the bright colours they wanted there.
C. It allowed them to experiment with a wide range of options.
D. It was meant to inspire them to embrace unconventional ideas.
5. Jed says that, for him, the name 'jewel garden' is ______.
A. a reminder of the value of creativity
B. an appropriate one for something so beautiful
C. a positive way of combining both past and present
D. a way of explaining his philosophy of design to people
Your answers:
1. 2. 3. 4. 5.

Part 4. Listen to part of a talk and complete the sentences with NO MORE THAN THREE
WORDS taken from the recording for each answer in the space provided. (20 points)
There really is no single definition of "business attire" these days. The era of "Big
Business" with fine clothes and tailored suits is largely gone, except in
(1)____________________________. It is easily for people to toss out all the starched,
buttoned-up conformity of (2)____________________________. The questioned posed is what
we use to replace office clothes. While bankers are (3)____________, techy-types in all
industries tend to dress down. The issues associated with (4)_____________ isn’t new in this
21st century though it seems to be more casual. Interestingly, in the prewar period, half of men’s
suit were (5)____________________________. From 1940s to 1950s, all styles remain the
same, except the fact that men’s suits change from (6)_____________________________ to
skinny. Between the time of 1960s and 1970s, traditional men’s suit remains unchanged,
however a new type of suit called the (7) ____________________________ shirts or Nehru
jackets were introduced.
Thanks to Silicon Valley, a term called “business casual” was used in the 1980s. Originally, it
referred to khaki pants, sensible shoes and (8)____________________________. However, this
term today is rather ambiguous. The Atlantic grappled with this by defining the casual workplace
basing on (9)____________________________ business culture instead of a "process-oriented"
culture obsessed with employees' looks. Mark Zuckerberg's gray T-shirt and gray hoodie is one
exception to casual workplace. Some business are still creased and pleated and tailor. We are left
to suss out the suitability of (10) ____________________________ and Allbirds, which don’t
match with shoulder pads. Final words given after all is “Dressing for success is as hard to define
as business casual”.

Your answers:
1. 6.
2. 7.
3. 8.
4. 9.
5. 10.

B. LEXICO - GRAMMAR (40 points)


Part 1. Choose one of the words marked A, B, C, or D which best completes each of the
following sentences. Write your answers in the corresponding numbered boxes. (20 points)
1. I never get a _________ of sleep after watching a horror film.
A. wink B. blink C. night D. ounce
2. The child sat in the middle of the floor and _________ refused to move.
A. distinctively B. decisively C. flatly D. totally
3. Little did I imagine The Amazing Race would entail long-winded journeys and ups and
downs _________.
A. aplenty B. inexhaustibly C. profusely D. superabundant
4. Ants are social insects which form small to large_________.
A. schools B. flocks C. colonies D. packs
5. Having our heart broken is an _________ part of growing up.
A. inward B. inset C. integral D. internal
6. You may as well as your _________ because Tom never listens to anyone.
A. breath B. iron C. impression D. nature
7. I couldn’t believe it when Marcy accused me of _________in her relationship with Joe.
A. intervening B. interacting C. interfering D. intercepting
8. There has been a lot of _________ surrounding the government’s proposed scheme.
A. controversy B. consent C. conformity D. consequence
9. The stage designed was out of _________ , but unfortunately the acting was not so impressive.
A. moon B. planet C. world D. earth
10. We all have to follow the rules, and none of us is _________the law.
A. beyond B. over C. above D. onto
11. Don’t be _________ by false advertisements. If something looks too good to be true, it
probably is.
A. putting off B. given up C. taken in D. put down
12. _________ talking of running for election again, after such a crushing defeat, is surely proof
of his resilience.
A. Should he be B. That he is C. Had he been D. That he were
13. I was at a_________ end on Sunday morning so I decided to bathe the puppy.
A. loose B. low C. dead D. wits’
14. In _______ of the increasing violence in our towns the President has decided to impose a
curfew.
A. sight B. view C. mood D. agreement
15. _________ 30,000 people are thought to have attended the concert.
A. As much as B. As many as C. Much less than D. As little as
16. I recommend reading the books _________, starting with the very first.
A. by accident B. at random C. in sequence D. on impact
17. The word “gossip” conjures _________ an image of a group of people huddled in a corner
sharing some scandalous news about neighbor or colleague.
A. up B. out C. cover D. in
18. The government is trying to _________ when it says it will spend more on health service
without raising taxes.
A. chew the fat B. wave the flag C. put the lid D. square the circle
19. Unanswered, the demands for nuclear deterrent have_________ fears of civil war.
A. flashed up B. prognosticated C. sidetracked D. stoked up
20. It is important to make sure the right decision because there is a lot at _________ .
A. range B. stake C. chance D. expectation

Your answers:
1. 5. 9. 13. 17.
2. 6. 10. 14. 18.
3. 7. 11. 15. 19.
4. 8. 12. 16. 20.

Part 2. Give the correct form of the words in brackets. Write your answers in the
corresponding numbered boxes. (10 points)
1. Not only is smoking unhealthy but it’s also __________ SOCIETY.
2. At first, I thought I could count on Jim, but then I discovered that he’s just a __________
friend. WEATHER
3. Now that this has been fully understood, the industy is __________ fighting online for the
loyalty of fans, but it may already be a lost cause. LATE
4. At all events, it was this group of the __________ that gave the first successful impetus to the
Revolution. POSSESS
5. All the talk about global warming really has some __________ for me after being flooded
last year. RESONATE
6. All human beings need some degree of physical exertion to keep fit, and inline skating is a
great __________ source of exercise. ROUND
7. This invention is attributed to Charles Barbier, who was __________ officer. Art
8. The computer age created an __________ and continuing explosion in the amount of Braille
published and read in nearly every country throughout the world. PRECEDENT
9. The Palme d’Or was awarded to the director of that movie because of his having created a
__________ film. THINK
10. Excuses like poor connections and __________ , or the inability to reach someone just
don’t work when even the most inaccessible places have wireless coverage. COMMUNICATE
Your answers:
1. 2. 3. 4. 5.
6. 7. 8. 9. 10.

C. READING (60 points)


Part 1. Read the following passage and decide which answer (A, B, C, or D) best fits each gap.
(15 points)
Interpreting the feelings of other people is not always easy, as we all know, and we (61)
_____ as much on what they seem to be telling us, as on the actual words they say. Facial
expression and tone of voice are obvious ways of showing our (62) _____ to something, and it
may well be that we (63) _____ express views that we are trying to hide. The art of being (64)
_____ lies in picking up these signals, realising what the other person is trying to say, and acting
so that they are not embarrassed in any way. For example, we may understand that they are in
fact reluctant to answer our question, and so we stop pressing them. Body movements in general
may also (65) _____ feelings, and interviewers often pay particular attention to the way a
candidate for a job walks into the room and sits down. However, it is not difficult to present the
right kind of (66) _____ while what many employers want to know relates to the candidate's
character traits, and (67) _____ stability. This raises the awkward question of whether job
candidates should be asked to complete psychological tests, and the further problem of whether
such tests actually produce (68) _____ results. For many people, being asked to take part in such
a test would be an objectionable (69) _____ into their private lives.
After all, a prospective employer would hardly ask a candidate to run a hundred metres, or
expect his or her family doctor to provide (70) _____ medical information. Quite apart from this
problem, can such tests predict whether a person is likely to be a conscientious employee or a
valued colleague?
61. A. estimate B. rely C. reckon D. trust
62. A. view B. feeling C. notion D. reaction
63. A. unconsciuosly B. rarely C. unaware D. cannot
64. A. good at B. humble C. tactful D. successful
65. A. have B. indicate C. contain D. infer
66. A. appearance B. candidate C. manners D. introduction
67. A. similar B. physical C. psychological D. relevant
68. A. faithful B. regular C. reliable D. predictable
69. A. invasion B. intrusion C. infringement D. interference
70. A. classified B. secretive C. reticent D. confidential

Your answers:
1. 2. 3. 4. 5.
6. 7. 8. 9. 10.

Part 2. Read the text below and think of the word which best fits each space. Use only one
word in each space. (15 points)
THE CULT OF CELEBRITY
Once, children had ambitions to be doctors, explorers, sportsmen, artists or scientists.
Now taking their lead from TV, they just "want to be famous". Fame is no longer a reward for
gallant service or great, perhaps even selfless endeavor. It is an end in (1) ______, and the sooner
it can be achieved, the sooner the lonely bedroom mirror can be replaced by the TV camera and
flash gun, the better. Celebrity is the profession (2 ______ the moment, a vainglorious vocation
which, (3) ______ some 18th- century royal court, seem to exist largely so that the rest of us
might watch and be amazed (4) ______ its members live out their lives in public, like self-
regarding members of some glittering soap opera.
Today, (5) ______ everyone can be famous. Never has fame (6) ______ more
democratic, more ordinary, more achievable. (7)______ wonder it's modern ambition. It's easy to
see why people crave celebrity, (8) ______ generations reared on the instant fame offered by
television want to step out of the limousine (9) ______ the flashlights bouncing around them.
Who doesn't want to be the center of attention at some time in their lives?
Modern celebrity, peopled by the largely vain and vacuous, fills a need in our lives. It
peoples talks shows, sells goods and newspapers and rewards the famous for, well, (10)
______famous.

Your answers:
1. 2. 3. 4. 5.
6. 7. 8. 9. 10.

Part 3. Read an extract from an article and choose the answer A, B, C or D that fits best
according to the text. Write your answers in the corresponding numbered boxes provided. (15
points)
Using video gaming in education
It has become conventional wisdom that spending too much time playing video games has a
detrimental effect on children’s studies and their social development. However, some
educationalists are now questioning this theory and are using video games as effective
educational tools thus bridging the gap between recreational and educational activities.
Due to the sophisticated nature of today’s games, teachers are able to justify the inclusion of
video and online games for many pedagogical reasons. There may, for example, be sociological,
psychological, and ethical implications built into the gameplay. Harvey Edwards, who teaches IT
classes in London, was one such educator who decided to use video games in his lessons. To do
this, he chose Minecraft, an online game in which players create and develop imaginary worlds.
He was somewhat uneasy about attempting such an unconventional approach, not because of
some students’ unfamiliarity with the game but rather due to them not being able to make sense
of what he was trying to do with it. He worried that it might interfere with his learners’ focus, but
he couldn’t have been more surprised by the results.
Minecraft is an example of a ‘sandbox game’, in which gamers roam around and change a virtual
world at will. Instead of having to pass through numbered levels to reach certain places, there’s
full access from start to finish. The original version can be adapted to control which characters
and content are left in. Each student can then be allocated tasks – such as house-building,
locating items or problem-solving – which they must complete within the game. Elements of
more general skills can be subtly incorporated into the lessons, such as online politeness and
safety, teamwork and resolving differences. Edwards feels that presenting such lessons in the
context of a game students probably already know and enjoy enables him to connect with them at
greater depth, and in more motivational ways.
Bolstered by his success, Edwards introduced his approach to another school nearby. He recalls
that the first couple of sessions didn’t live up to his expectations. Those who had played
Minecraft before were keen for others to adopt their own style of play. Unsurprisingly, this
assortment of styles and opinions as to how the game should proceed were far from harmonious.
However, the sessions rapidly transformed into something more cohesive, with the learners
driving the change. With minimal teacher input, they set about choosing leaders and established
several teams, each with its own clearly-defined role. These teams, now party to clear common
goals, willingly cooperated to ensure that their newborn world flourished, even when faced with
the toughest of challenges.
‘Human’ inhabitants in a Minecraft ‘society’ are very primitive and wander around the
imaginary world, waiting for guidance from players. [A] This dynamic bears a resemblance to
traditional education, an observation highlighted by Martina Williams, one of the leaders of the
group. [B] ‘Through the game, we were no longer passive learners in the classroom, being told
what and how to learn, but active participants in our own society. [C] The leaders, meanwhile,
had a vision for their virtual world as a whole, encouraging everyone to play their part in
achieving the group’s goals. [D] Through creating their own characters and using these to build
their own ‘world’, students will have gained some experiential understanding of societal
structure and how communities work.
But not everyone is convinced by video games’ potential academic value. While many
progressive commentators cite extensive evidence to maintain that video games encourage
collaboration and build problem-solving skills, more traditional factions continue to insist they
are a distraction that do not merit inclusion in any curriculum. Even less evangelical cynics, who
may grudgingly acknowledge games have some educational benefit, assert that this is only the
case in the hands of creative educators. However, the accusation most often levelled at video
games is that they detract from the social aspect of the classroom, particularly taking part in
discussions. Dr Helen Conway, an educational researcher, argues that video games can be used
to promote social activities. ‘Students become animated talking about the game and how to
improve their gameplaying and problem-solving skills,’ she says. ‘I find it strange, this image
that many people have,’ Conway says. ‘Children are often totally detached from their peers when
undertaking more traditional activities, like reading books, but we never suggest that books are
harmful because they’re a solitary experience.

1. The first time Edwards used a game in his classes, he was______________


A. convinced that learners would realise why he wanted them to play it.
B. convinced that learners would see the reasons for playing it.
C. anxious that he had chosen the wrong one for learners to play.
D. sure that his reasons for getting learners to play it were valid.
2. The writer suggests that Minecraft is a good choice of educational game because______
A. any number of learners can use it simultaneously.
B. teachers can remove any inappropriate material.
C. gamers can create educative tasks whilst playing it.
D. players can develop their skills in a step-by-step way.
3. Which of the following words in the fourth paragraph is used to convey a feeling of
approval?
A. keen B. harmonious C. driving D. newborn
4. In the fifth paragraph, the writer draws a comparison between a Minecraft ‘society’
and_______________
A. relationships within the group as they played.
B. the way in which countries organize themselves.
C. typical students in a school environment.
D. how leadership operates in different situations.
5. In the sixth paragraph, the writer feels that critics of video games in education_________
A. are unwilling to admit that using them in class has benefits.
B. make accurate observations about teachers who use them.
C. use flawed research to support their objections to using them.
D. acknowledge the drawbacks of more traditional teaching methods.
6. The words ‘this image’ in the sixth paragraph refer to______________
A. people who criticize gaming in education. B. students discussing a game in a group.
C. a group of students reading individually. D. a solitary player absorbed in a game.
7. Where does this sentence belong to in the fifth paragraph?
Each group member had ideas as to how their function should develop.
A. [A] B. [B] C. [C] D. [D]
8. The word ‘subtly’ in the third paragraph is closest in meaning to __________.
A. intricately B. ingeniously C. ingenuously D. haphazardly
9. The word ‘grudgingly’ in the sixth paragraph is closest in meaning to __________.
A. gleefully B. vivaciously C. genially D. reluctantly
10. Which of the following best describes the author's attitude towards the application of
gaming into education?
A. supportive B. neutral C. cynical D. satirical

Your answers:
1. 2. 3. 4. 5.
6. 7. 8. 9. 10.

Part 4. For questions 1-10, read the following passage and do the tasks that follow. (15 points)
Follow your nose
A Aromatherapy is the most widely used complementary therapy in the National Health
Service, and doctors use it most often for treating dementia. For elderly patients who have
difficulty interacting verbally, and to whom conventional medicine has little to offer,
aromatherapy can bring benefits in terms of better sleep, improved motivation, and less disturbed
behaviour. So the thinking goes. But last year, a systematic review of health care databases found
almost no evidence that aromatherapy is effective in the treatment of dementia. Other findings
suggest that aromatherapy works only if you believe it will. In fact, the only research that has
unequivocally shown it to have an effect has been carried out on animals.
B Behavioural studies have consistently shown that odours elicit emotional memories far
more readily than other sensory cues. And earlier this year, Rachel Herz, of Brown University in
Providence, Rhode Island, and colleagues peered into people’s heads using functional Magnetic
Resonance Imaging (fMRI) to corroborate that. They scanned the brains of five women while
they either looked at a photo of a bottle of perfume that evoked a pleasant memory for them, or
smelled that perfume. One woman, for instance, remembered how as a child living in Paris—she
would watch with excitement as her mother dressed to go out and sprayed herself with that
perfume. The women themselves described the perfume as far more evocative than the photo,
and Herz and co-workers found that the scent did indeed activate the amygdala and other brain
regions associated with emotion processing far more strongly than the photograph. But the
interesting thing was that the memory itself was no better recalled by the odour than by the
picture. “People don’t remember any more detail or with any more clarity when the memory is
recalled with an odour,” she says. “However, with the odour, you have this intense emotional
feeling that’s really visceral.”
C That’s hardly surprising, Herz thinks, given how the brain has evolved. “The way I like to
think about it is that emotion and olfaction are essentially the same things,” she says. “The part
of the brain that controls emotion literally grew out of the part of the brain that controls smell.”
That, she says, probably explains why memories for odours that are associated with intense
emotions are so strongly entrenched in us, because the smell was initially a survival skill: a
signal to approach or to avoid.
D Eric Vermetten, a psychiatrist at the University of Utrecht in the Netherlands, says that
doctors have long known about the potential of smells to act as traumatic reminders, but the
evidence has been largely anecdotal. Last year, he and others set out to document it by describing
three cases of post-traumatic stress disorder (PTSD) in which patients reported either that a
certain smell triggered their flashbacks, or that smell was a feature of the flashback itself. The
researchers concluded that odours could be made use of in exposure therapy, or for
reconditioning patients’ fear responses.
E After Vermetten presented his findings at a conference, doctors in the audience told him
how they had turned this association around and put it to good use. PTSD patients often undergo
group therapy, but the therapy itself can expose them to traumatic reminders. “Some clinicians
put a strip of vanilla or a strong, pleasant, everyday odorant such as coffee under their patients’
noses, so that they have this continuous olfactory stimulation,” says Vermetten. So armed, the
patients seem to be better protected against flashbacks. It’s purely anecdotal, and nobody knows
what’s happening in the brain, says Vermetten, but it’s possible that the neural pathways by
which the odour elicits the pleasant, everyday memory override the fear-conditioned neural
pathways that respond to verbal cues.
F According to Herz, the therapeutic potential of odours could lie in their very unreliability.
She has shown with her perfume-bottle experiment that they don’t guarantee any better recall,
even if the memories they elicit feel more real. And there’s plenty of research to show that our
noses can be tricked, because being predominantly visual and verbal creatures, we put more faith
in those other modalities. In 2001, for instance, Gil Morrot, of the National Institute for
Agronomic Research in Montpellier, tricked 54 oenology students by secretly colouring a white
wine with an odourless red dye just before they were asked to describe the odours of a range of
red and white wines. The students described the coloured wine using terms typically reserved for
red wines. What’s more, just like experts, they used terms alluding to the wine’s redness and
darkness—visual rather than olfactory qualities. Smell, the researchers concluded, cannot be
separated from the other senses.
G Last July, Jay Gottfried and Ray Dolan of the Wellcome Department of Imaging
Neuroscience in London took that research a step further when they tested people’s response
times in naming an odour, either when presented with an image that was associated with the
odour or one that was not. So, they asked them to sniff vanilla and simultaneously showed them
either a picture of ice cream or of cheese, while scanning their brains in a fMRI machine. People
named the smells faster when the picture showed something semantically related to them, and
when that happened, a structure called the hippocampus was strongly activated. The researchers’
interpretation was that the hippocampus plays a role in integrating information from the senses—
information that the brain then uses to decide what it is perceiving.
Questions 1-7
The passage has seven paragraphs, A-G.
Choose the correct heading for paragraph A-G and from the list of headings below.
Write the correct number, i-x, in boxes 1-6.
List of Headings
i Remembering the past more clearly
ii Bringing back painful memories
iii Originally an alarm signal
iv The physical effects of scent versus image
v Checking unreliable evidence
vi Reinforcing one sense with another
vii Protection against reliving the past
viii The overriding power of sight and sound
ix Conflicting views

Example Paragraph A : ix
1 Paragraph B
2 Paragraph C
3 Paragraph D
4 Paragraph E
5 Paragraph F
6 Paragraph G
Your answers
1. 2. 3. 4. 5. 6.
Questions 7-10
Look at the following findings and the list of researchers
Match each finding with the correct researcher, A-D.
Write the correct letter, A-D, in boxes 7-10.
NB You may use any letter more than once.
7. Smell can trigger images of horrible events.
8. Memory cannot get sharpened by smell.
9. When people are given an odour and a picture of something to learn, they will respond more
quickly in naming the smell because the stimulus is stronger when two or more senses are
involved.
10. It is impossible to isolate smell from visual cues.

A Rachel Hertz
B Eric Vermetten
C Gil Morrot
D Jay Gottfried and Ray Dolan
Your answers
7. 8. 9. 10.

D. WRITING (50 points)


Part 1.Rewrite the following sentences using the words given
1. He maintained his position against his adversary. GROUND
=> He held __________________________________________________________________
2. Maria has never forgotten to do his homework. REMEMBER
=> Mandy __________________________________________________________________
3. Jane didn’t get the job because she wasn’t experienced enough. DOWN
=> Jane’s ___________________________________________________________________
4. Apparently, a lot of employees will be made redundant when the 21st Century Fox is
taken over. HEAP
=> Apparently, many an ______________________________________________________
5. I like everything about my new job except for the fact that I have to work every other
weekend. FLY
=> I like everything about my new job, ___________________________________________

Part 2: (20 points)


The graph below shows the number of tourists visiting a particular Caribbean Island between
2010-2017. Summarize the information by selecting and reporting the main features, and
make comparisons where relevant. Write at least 150 words.

Part 3. Essay writing. (30 points)


Write an essay of about 250 words to express your opinion on the following topic:
Some people say that subjects like arts, music, drama and creative writing are more beneficial
to children and therefore they need more of these subjects to be included in the timetable.
Do you agree or disagree?
Use your own knowledge and experience to support your arguments with examples and relevant
evidence.

TEST 6
I. LISTENING (50 POINTS)
Part 1. For questions 1-5, listen to the recording and complete the form below. Write NO
MORE THAN TWO WORDS AND/OR A NUMBER for each answer. Write your answers
in the corresponding numbered boxes provided. (10 pts)

Your answers:
1. 4.
2. 5.
3.

Part 2. You will hear a radio interview with the gardening experts Jed and Helena Stone.
For questions 6-10, choose the answer (А, В, C or D) which fits best according to what you
hear. (10 pts)

6. How does Helena feel about the use of Jed’s name for their joint business?

A. occasionally frustrated that her contribution goes unnoticed


B. amused that they have a name people tend to remember

C. appreciative of the respect that the name has brought her

D. irritated by the fact that Jed is more of a celebrity than she is

7. What is Jed’s attitude to his public profile?

A. He likes the fact that complete strangers often want to talk to him

B. He’s unhappy that it prevents him doing everyday activities

C. He enjoys it more now than he did when he was younger

D. He’s proud of the way it reflects his achievements

8. How did Helena feel about her work on The Travel Show?

A She would have enjoyed it more in different circumstances

B It was convenient for her to be away from the house then

C It was a welcome alternative to manual work

D She felt obliged to do it at that particular time

9. What gave Jed the incentive to make a jewel garden?

A. He wanted to realise a long-held ambition

B. He had led people to believe that it already existed

C. He wanted to show pictures of it at a gardening event

D. He was inspired by the illustrations at a talk he attended

10. Jed says that, for him, the name ‘jewel garden’ is

A. a reminder of the value of creativity

B. an appropriate one for something so beautiful

C. a positive way of combining both past and present

D. a way of explaining his philosophy of design to people


Your answer:
6. 7. 8. 9. 10.

Part 3. For questions 11-15, listen to an explanation about 5G and decide these statements
are TRUE (T) or FALSE (F). Write your answers in the corresponding numbered boxes
provided. (10 pts)

11. 5G is a defined network which can replace cables completely by operating on the cloud only.

12. With the help of 5G, the Internet of Things is likely to grow threefold by 2025.

13. Until 2025, the majority of people will start to use 5G, which will have surpassed 3G and 4G
by then.

14. In the UK, 3G and 4G networks were quite cheap because they could be operated on the
country’s radio spectrum.

15. It is predicted that by 2025 almost a quarter of all mobile connections in the U.S. will be 5G.

Your answer:
11. 12. 13. 14. 15.

Part 4. You will hear a radio programme about the deforestation in South America’s
Amazon. For questions 16-25, complete the missing information with a word or short
phrase (no more than three words). (20 pts)

16. An increase in agriculture and ______ and the building of roads and dams are blamed to
bring about the deforestation.

17. ______ methods at the pre-Columbian period could offer valuable lessons for today.

18. A research was conducted on a coastal wetland area where ancient ______ and canals remain
unchanged.

19. It was erroneously believed that a great deal of fire was used by pre-Columbian farmers to
manage ______.

20. The result of the study showed that raised-field farmers ______ to improve agriculture
production.

21. Periods without fires during the time when land was out of use in farming were valuable for
rebuilding ______ matter and preserving soil structure.
22. This fire-free method contributed to turning the seasonally ______, or grassland, into
productive cropland.

23. Moreover, this large, cultivated elevation provided better drainage and soil aeration and also
held ______ during the dry season.

24. However, this fire-free method would have been ______ with ninety-five percent of the
native people died from the diseases spread by the Europeans.

25. ______ method imposed a threat to the rainforest by the European colonizers.

Your answers:
16. 21.
17. 22.
18. 23.
19. 24.
20. 25.

II. LEXICO - GRAMMAR

Part 1. For questions 26-45, choose one of the words marked A, B, C, or D which best
completes each of the following sentences. Write your answers in the corresponding
numbered boxes. (20 pts)

26. You can't believe a word that woman says. She is a ____________ liar.

A. dedicated B. devoted C. committed D. compulsive

27. When you come down the hill, do drive slowly because it is not _________ obvious where
the turning is.

A. immediately B. directly C. instantaneously D. quite

28. The driver consumed enough alcohol to make him ____________.

A. pass off B. pass out C. pass for D. pass up

29. Sarah and I ____________ reserved the room in the same hotel. She was very surprised to
see me there.

A. practically B. intentionally C. deliberately D. coincidentally

30. It was a fair result because we played the game ____________ the rules.
A. on account of B. due to C. according to D. apart from

31. Fiona didn’t lie but she did give rather ____________ a picture of the situation.

A. defaced B. distorted C. disfigured D. disguised

32. As Michelle told me the story, tears ____________ up in her eyes.

A. welled B. raised C. filled D. mounted

33. His past behaviour had a definite _________ on what the judges decided.

A. bearing B. weight C. decision D. conclusion

34. On cloudy nights it is not possible to see the stars with ____________ eye.

A. naked B. bare C. flesh D. pure

35. He seemed rather ____________ .Was he upset about something?

A. unsocial B. apathetic C. passive D. subdued

36. The museum is just a short walk from here. ______ you go, it will take you no more than ten
minutes.

A. No matter how B. Wherever C. No matter the way D. Whichever the way

37. We took the shortcut round the market ______ late for class this morning.

A. so that we will not be B. lest we be not

C. for fear that we should be D. in order that we not be

38. Susana is managing to attend night school without allowing ______ with her day job.

A. that it interferes B. interfering C. the interference D. it to interfere

39. The lecture was so lengthy that the students started to ______ and daydream.

A. come off B. put off C. turn off D. get off

40. My parents always had a happily _______ attitude to my staying out late in the evening.

A. cold- blooded B. long- suffering C. easy- going D. thick- skinned


41. I am in a _______ as how to use this CD Rom.

A. loss B. difficulty C. quandary D. mind- game

42. Many of Lan’s friends like Korean soap operas, but she finds it not to her ______.

A. Likes B. liking C. likeness D. likelihood

43. “If they asked you to help them move next week, ______?”

A. would you B. didn’t they C. did you D. would they

44. Some teenagers just simply run ______ to their parents’ expectations as a way to express
their identity.

A. against B. opposite C. contrary D. counter

45. He was _______ devastated by the news.

A. utterly B. extremely C. deeply D. immensely

Your answers:
26. 30. 34. 38. 42.

27. 31. 35. 39. 43.

28. 32. 36. 40. 44.

29. 33. 37. 41. 45.

Part 2. For questions 56-65, fill each gap with the correct form of the words in brackets.
Write your answer in the boxes provided. (10 pts)

A. Put the correct form of the words in brackets.

56. The careless driver was __________________about his bad behavior, which made all the
witnesses fly into a rage. (APOLOGIZE)

57. The pregnant woman is looking at the collection of __________________clothes.


(MATERRIAL)

58. There was a/an______________change in the tone of her voice. Almost nobody could
recognize it. (PERCEIVE)

59. The slight __________________in his left hand was corrected by surgery. (FORM)
60. We are trying to create our own computerized __________________. (DATA)

61. In my opinion, this book is just __________________rubbish. (INTELLECT)

62. The company has established total __________________over its rivals. (SUPREME)

63. Bard is a very good employee, and is very __________________ (CONSCIENCE)

64. He achieved __________________for failing a drug test after winning an Olympic final.
(NOTORIOUS)

65. Before enrolling on a course, you should first ensure that it has been
__________________by an officially recognized body. (VALID)

Your answers:
56. 57. 58. 59. 60.
61. 62. 63. 64. 65.

III. READING (60 POINTS)

Part 1. For questions 66-75, read the text below and decide which answer (A, B, C or D)
best fits each gap. Write your answers in the corresponding numbered boxes provided. (15
pts)

A desert is a special region where only certain kinds of plants and animals can survive.
All deserts have very little water. This means that only animals and plants that can (66)
_____water for long periods of time can exist in the desert.

Plants in the deserts are particularly (67) _____to the dry and hot environment. One well-
known desert plant is the cactus. (68) _____many desert plants, this plant has very tiny leaves.
As plants lose most of their water through their leaves, the small leaves of the cactus help to cut
down water evaporation. There are some desert plants that do not have leaves (69) _____.

Some desert plants survive by avoiding the dry season (70) _____. During the dry season,
this plant remains a seed and does not (71) _____from the soil at all. When the rains come, this
seed would grow very quickly into a plant. It would bloom rapidly and then (72) _____its seeds
before the dry season returns.

Desert animals have also learnt to adapt well to life in this region. The camel, for
example, (73) _____well in the desert because water can be (74) _____in its body. Other desert
animals include rodents such as mice. These animals need very little water as they can get all the
water they (75) _____from their food.
66. A. do without B. keep off C. stay away D. give up

67. A. convenient B. adjusted C. regulated D. adapted

68. A. As B. Similar C. Like D. Just as

69. A. at all B. as well C. in all D. either

70. A. totally B. entirely C. altogether D. wholly

71. A. rise B. arise C. awake D. emerge

72. A. scatter B. throw C. fling D. cast

73. A. exists B. survives C. subsists D. remains

74. A. stocked B. kept C. stored D. contained

75. A. demand B. require C. request D. ask for

Your answers
66. 67. 68. 69. 70.
71. 72. 73. 74. 75.

Part 2. For questions 76-85, fill each gap in the passage below with ONE appropriate word.
Write your answers in the corresponding numbered boxes provided. (15 pts)

Ten thousand years ago, as the last ice age (76) ______ to a close, sea levels around the
world were far lower than they are today. Much of the land under both the North Sea to the east
of Britain and the English Channel which now (77) ______France and Britain was part of a huge
region of forests and grassy plains, (78) ______herds of horses and reindeer roamed free and
people lived in villages by the lakes and rivers. Then the climate gradually became warmer (a
phenomenon certainly not confined to our own age!) and the water (79) ______ in glaciers and
ice caps was released. This ancient land was submerged in the resulting deluge and all that
remains to tell us that it was once lush and verdant – and inhabited – is the occasional stone tool,
harpoon or mammoth tusk brought up from the seabed by fishing boats.

Now the development of advanced sonar technology, known as bathymetry, is (80)


______ it possible to study this flooded landscape in extraordinary detail. A special echo sounder
is fixed to the bottom of a survey vessel, and it makes wide sweeps across the seabed. (81)
_______ previous technology has only been able to produce two-dimensional images,
bathymetry can now deploy computers, satellite-positioning equipment and special software to
create accurate and remarkably detailed maps. For the first time, an ancient riverbed leaps out of
the three-dimensional image, complete with rocky ledges (82) ______ up from the bottom of the
valley. The sites of prehistoric settlements can now be pinpointed, and it is also possible to see in
stunning detail the sunken shipwrecks that litter this part of the seabed.

According to archaeologist Dr Linda Andrews, this technological development is of huge


(83) ______ . ‘We now have the ability to map the seabed as accurately as we can map dry land,’
she says. She is, (84) ______, scathing about the scale of financial support for such projects. ‘We
have better images of Mars and Venus than of two-thirds of our own planet! Britain is an
interesting case. It’s been a maritime nation for much of its history, and the sea has had (85)
______ a massive influence on it, and in view of this, it’s an absolute scandal that we know so
little about the area just off the country’s shores!’
Your answers:
76. 77. 78. 79. 80.
81. 82. 83. 84. 85.

Part 3. For questions 86-95, read the following passage and choose the best answer to each
question. Write your answers in the corresponding numbered boxes provided. (15pts)

The Formation of the Rocky Mountains

The Rocky Mountains of North American extend 5,000 kilometers from New Mexico all the way
up through Canada. Elevations along the range are about 1,500 meters along the lower plains to
4,399 meters at the highest peak, and widths range from 120 to 650 kilometers. The natural
beauty, abundant wildlife, and fresh water of the ranges have attracted human inhabitants for the
last 10,000 to 12,000 years.

The history of the Rocky Mountains in the pre-Cambrian era, a half-billion years ago. While this
is long before the Rocky Mountains themselves began forming, their hard core rocks – consisting
of granites, schists, gneisses, quartzites, and slates – were produced in ancient ranges. Erosion
eventually leveled these mountain ranges, and during the Paleozoic and Mesozoic Eras, about 75
to 540 million years ago, the ocean invaded the land and deposited sediments some 20,000 feet
deep. They included layers of sandstones, shales, and limestones.

At the close of the Mesozoic Era, during the Cretaceous period about 75 million years ago, the
growth of the Rockies began. There was a tremendous squeezing that uplifted the region in a
great series of folds, like wrinkles in a carpet. After the arching, erosion carved away at the
mountains. Some 10,000 feet of sedimentary rock were washed off the top of the arch, exposing
the hard rock core. The erosional resistance of these hard, crystalline rocks led to the formation
of the high peaks that still exist today. On the flanks of the core the sedimentary beds sloped
outward. Great quantities of sand and clay were spread out on the bordering plains and plateaus.
This was only one of the cycles of upheaval and erosion that occurred in the region.
Near the end of the Eocene period, about 40 million years ago, the Rockies again rose several
thousand feet. Volcanoes erupted, most extensively in the Yellowstone Plateau and the Absaroka
Range. As the mountains were formed, streams eroded their sides, and thousands of feet of
sediment spread out on plains and plateaus. Just before the Pleistocene period, about one million
years ago, the region again uplifted. But once again, the mountains began to be worn away as
soon as they rose. Streams flowed faster and began to cut canyons, and rivers ate deep gorges
through the ranges. The most recent geological event of note was the “Ice Age” during the
Pleistocene Epoch, 1 million to 10,000 years ago. The high peaks of the Sangre de Cristo
Mountains supported numerous small glaciers and snows accumulated on the sides of the
mountains. These glaciers carved a typical collection of alpine landforms, such as cirques, horns,
arêtes, and cols. Lower down in the glaciated valleys, various kinds of till and stratified
sediments accumulated to form Moraines. Most of the glacial deposits and landforms present
today date from the last glacial phase, known in the Rocky Mountains as the Pinedale Glaciation
of Pinedale Stage. During this stage, over 90% of the Yellowstone National Park was covered in
ice. The glaciated terrains formed and moved down the valleys, thereby further eroding the
mountains into bold and dramatic forms.

There was even a “little ice age” from about 1550 to 1860 – a few centuries of glacial advance –
that made its mark on the mountains recently. For example, the Agassiz and Jackson glaciers in
Glacier National Park reached their most forward positions by around 1860. The incessant
sculpturing of the Rockies by rain, wind, and ice continues even today.

(Source: Cracking the TOEFL iBT 2019 Edition)

86. The purpose of paragraph 1 is to demonstrate that

A. the Rocky Mountains have not been fully explored until recently

B. most of the Rocky Mountains are not very high

C. there are many types of mountains in the Rocky Mountains

D. human inhabitants are destroying the natural beauty of the Rocky Mountains

87. The word They in the passage refers to

A. sediments B. eras C. years D. mountain ranges

88. According to the passage, all of the following types of rock would be found at the core of the
Rocky Mountains EXCEPT

A. gneiss B. cirque C. slate D. granite

89. What happened when oceans covered the Rocky Mountain region?
A. The mountains were washed away. B. Quartzite rocks were formed.

C. Erosion shifted the rocks. D. Deep sediment covered the land.

90. The word “ate” in the passage is closest in meaning to

A. raised B. buried C. erased D. dug

91. The upward growth of the Rocky Mountains began to occur for the first time during which
period?

A. Pre-Cambrian B. Paleozoic C. Eocene D. Cretaceous

92. The plains and plateaus that surround the Rocky Mountains were covered with

A. material washed down from the mountains

B. hard core rocks such as slate

C. densely packed volcanic deposits

D. soil rich in organic matter

93. The passage indicates that some of the most dramatic-looking parts of the Rocky Mountains
were formed by

A. volcanic eruptions B. rivers C. glaciers D. the ocean

94. Which of the following best describes the organization of the passage as a whole?

A. A mountain range serves to illustrate a widespread geological process.

B. The history of a geological feature is discussed in chronological order.

C. Two time periods in the history of a mountain range are contrasted.

D. The effects of a number of geological periods on North America are compared.

95. The phrase “of note” in the passage is closest in meaning to

A. catastrophic B. distinct C. important D. ancient

Your answer
86. 87. 88. 89. 90.
91. 92. 93. 94. 95.
Part 4. For questions 96-105, read the text and do the tasks followed. Write your answers in
the corresponding numbered boxes provided. (15 pts)

Leisure time

A. A raft of forecasts has been made in the recent decade, predicting the decline in the number of
working hours coupled with a consequent increase in leisure time. It was estimated that the
leisure revolution would take place by the turn of the last cent my with hours devoted to work
railing to 25-30 per week, This reduction hits failed to materialize, but the revolu¬tion has,
nonetheless, arrived.

B. Over the past 30 to 41 years, spending on leisure has witnessed a strong increase, Accord¬ing
to the annual family expenditure survey published in 1935 by the Office for National Statistics,
the average household in the United Kingdom spent more on leisure than food, housing and
transport for the very first time, and the trend is also set to continue up¬wards well into the
present century.

C. The survey, based on a sample of 6,500 households showed, that the days are long gone when
the average family struggled to buy basic foods. As recently as 1969, family spending on food
was approximately one third compared to 17% now. Twelve years later, there was a noticeable
shift towards leisure with the percentage of household spending on leisure increasing to 9%, and
that on food declining to 26%.

D. The average household income in the UK in 1999 was £460 per week before tax, and average
spending was £352.20. Of the latter sum, £59.70 was spent on leisure and £58.90 on food. On
holidays alone, family expenditure was 6%, while in 1969 the proportion spent on holidays was
just 2%. And whereas the richest 10% lashed out 20% of their income in 1999 on leisure, the
poorest spent 12%.

E. Among the professional and managerial classes, working hours have increased and, overall in
the economy, record numbers of people are in employment. As people work more, the appetite
for leisure activities has grown to compensate for the greater stress in life. The past 5 years alone
have seen the leisure business expand by 25% with a change in emphasis to short domestic
weekend breaks and long-haul short breaks to exotic destinations in place of long holidays. In the
future, it is expected that people will jump from one leisure activity to another in complexes
catering for everyone’s needs with gyms, cinemas, cafes, restaurants, bars and internet facilities
all under one roof. The leisure complexes of today will expand to house all the leisure facilities
required for the leisure age.

F. Other factors fueling demand for leisure activities are rising prosperity, increasing longevity
and a more active elderly population. Hence, at the forefront of leisure spending are not just
young or professional classes. The 1999 family expenditure survey showed that the 64 to 75-
year-old group spend a higher proportion of their income on leisure than any other age group.
The strength of the “grey pound” now means that elderly people are able to command more
respect and, thus, attention in the leisure market.

G. And the future? It is anticipated that, in the years to come, leisure spending will account for
between a third to a half of all household spending. Whilst it is difficult to give exact figures, the
leisure industry will certainly experience a long period of sustained growth. Working hours are
not expected to decrease, partly because the 24-hour society will need to be serviced; and
secondly, because more people will be needed to keep the service/leisure industries running.

H. In the coming decades, the pace of change will accelerate, generating greater wealth at a
faster rate than ever before. Surveys show that this is already happening in many parts of Europe.
The south-east of England, for example, is now supposedly the richest area in the EEC. The
“leisure pound” is one of the driving forces behind this surge. But, sadly, it does not look as if we
will have the long leisure hours that we had all been promised.

Questions 29-35

The reading passage has 8 paragraphs. Choose the most suitable heading for each paragraph
from the list of headings below.

Write the appropriate numbers (i-xiv) in Boxes 1-7 on your answer sheet. One of the headings
has been done for you as an example. You may use any heading more than once. There are
more headings than paragraphs, so you will not use all of them,

96. Paragraph A

97. Paragraph B

98. Paragraph C

Example: Paragraph D Answer: iv

99. Paragraph E

100. Paragraph F

101. Paragraph G

102. Paragraph H

List of headings

i. Leisure spending goes up strongly


ii. Decreasing unemployment

iii. False forecasts

iv. Spending trends – leisure v food

v. More affordable food

vi. Leisure as an answer to stress

vii. Looking forward

viii. The leisure revolution – working hours reduced to 25

ix. The “grey pound” soars

x. Rising expenditure

xi. The elderly leisure market

xii. National Statisticians

xiii. Work, stress, and leisure all on the up

xiv. Money yes, leisure time no

Questions 8 - 10

Do the statements below agree with the information in the reading passage? In Boxes 8 -10,
write:

YES, if the statement agrees with the information in the passage

NO, if the statement contradicts the information in the passage

NOT GIVEN, if there is no information about the statement in the passage

Example: In recent decades, an increase in working hours was predicted.

Answer: No.

103. At the turn of the last century, weekly work hours dropped to 25.

104. Spending on leisure has gone up over the past three decades.
105. 24-hour society will have a negative effect on people’s attitudes to work.

Your answer
96. 97. 98. 99. 100.
101. 102. 103. 104. 105.

IV. WRITING (50 POINTS)

Part 1.Rewrite the following sentences using the words given

1. The heavy downpour brought their picnic to an abrupt end. CUT

=> They had to _____________________________________________________________

2. It has warmed up considerably compared to yesterday. NEARLY

=> It ______________________________________________________________________

3. I don't like either candidate but Michael Jones is not quite as bad. LESSER

=> Neither candidate is in _____________________________________________________

4. Tom and Nick started to hate each other after getting into a fierce fight. LOST

=> There has been no ________________________________________________________

5. Two committee members have nominated John for the post of treasurer. FORWARD

=> John has ________________________________________________________________

Part 2. The graph below shows the demand for electricity in England during typical days in
autumn and spring. The pie chart shows how electricity is used in an average English
home. Summarize the information by selecting and reporting the main features, and make
comparisons where relevant. (20 pts)
Part 3. Write an essay of 200-250 words about this topic. (30 pts)

Young people are increasingly choosing to eliminate meat from their diets and become
vegetarians.

Do you think this is a good idea? Does it bring more benefits or drawbacks?

TEST 7

I. LISTENING (50 POINTS)

Part 1. You are going to hear a person calling an accommodation agency about an
apartment she wants to rent. Listen and complete the form. Write no more than THREE
WORDS AND/ OR NUMBER for each answer (10 pts)

GOOD MOVES ACCOMODATION AGENCY


Call taken by: Ben

Name: (1) ……………………..

Heard about us from: family member

Type of accommodation preferred: (2)……………………..

Preferred location: Want to be close to (3)……………………..

Price: Maximum: (4) £…………………….. per person (including bills)

Additional notes:

I suggested flat 3 at (5)…………………….. Road in Bampton.

I will send further details to customer by emails.

Part 2. You will hear two psychologists talking about modern childhood. For questions 1 –
5, choose the answer (A, B, C, or D) which fits best according to what you hear. (10 pts)

1. What does Daniel imply about past images of childhood?

A. They are entirely fictional.

B. They all show the misfortunes of childhood.

C. They are diverse.

D. They represent the innocence of childhood.

2. When mentioning the children throwing bags on the bus-stop, Louise is _____.

A. critical B. amused C. angry D. sarcastic

3. According to Daniel, ______.


A. children are failing to learn adequate social skills

B. children do not eat a balanced diet

C. children are becoming involved in political scandals

D. children are far more sociable than they used to be

4. Louise believes that ________.

A. parents are no longer interested in their children

B. children should study harder to pass school exams

C. modern life has a negative effect on children

D. most parents are emotionally unstable

5. Daniel implies that _______.

A. children would be happier if their parents taught them at home

B. machines are more of a menace to children than people are

C. teachers aren’t helping children to be competitive enough

D. most teenage problems stem from an unbalanced diet

Part 3. Listen to the extract of a television travel program, and then decide whether each of
the following statements is true or false. (10 pts)
Statements T F
1. A British passport holder has to pay $10 for a visa at the border of Guatemala.
2. A new limit of seven days will be imposed on tourist visas to visit Burma.
3. Tourists arriving in Burma will not be allowed to visit the capital, Rangoon.
4. At the moment, the only place you can obtain a visa to visit Burma is in Bangkok.
5. Not all resorts on the Costa del Sol will be offering reductions for children next year.

Part 4. You will hear someone giving a talk about careers in the fire service. For questions 1-
10, complete the sentences with a word or short phrase. Write NO MORE THAN THREE
WORDS for each answer in the space provided. (20 pts)

Careers in the Fire Service

1. Like all her colleagues, Debbie is involved in ________________ as well as dealing with
emergencies.

2. Debbie explains that most emergency calls turn to be ________________.

3. Debbie’s first experience of a big fire was when a ________________ near London caught
light.

4. At her first big fire, Debbie was positioned on a piece of equipment called a
________________

5. After a major fire, the firefighters meet for what’s known as a________________.

6. Debbie is especially useful when someone is needed to work in a ___________ in an


emergency.

7. To become a firefighter, Debbie had to undergo tests of fitness, _____________ and strength.

8. Debbie blames an unfair____________ for preserving women from joining the service in the
past.

9. There are no allowances made for female applicants; it’s all down to their ___________ in the
tests.

10. Finally, Debbie reminds us that the job is not as_____________ as it might appear on TV.
II. LEXICO-GRAMMAR (40 POINTS)

Part 1. Choose the best answer (A, B, C or D) to complete each sentence below. (20 pts)
1. She used her weekly column in the local newspaper as a ______ for her political views.
A. means B. vehicle C. vessel D. passage
2. ______ to have stolen credit cards, he has been investigated for days.
A. Suspecting B. Having suspected C. Suspected D. To be suspected
3. ______ as taste is really a composite sense made up of both taste and smell.
A. That we refer to B. What we refer to
C. To which we refer D. What do we refer to
4. Writing rhymes for birthday cards is really easy. It’s money for old ______.
A. boot B. rubbish C. bread D. rope
5. We all decreed that ______.
A. there be an end to their quarrel B. their quarrel should put an end to
C. they ended their quarrel then D. their quarrel be coming to an end.
6. The president was eventually ________ by a military coup.
A. disbarred B. supplanted C. deposed D. subverted
7. Poor Mary, all her colleagues teased her; she was the _______ of all their jokes.
A. hubbub B. butt C. bulk D. brunt
8. The firefighters fought the blaze while the crowd _________.
A. walked out B. kicked back C. sit about D. looked on
9. We do not have a secretary _________, but we do have a student who comes in to do a
bit of filing.
A. as such B. the least bit C. whatsoever D. little more
10. Derek had no experience of white-water canoeing, so it was extremely _______ of him
to try and shoot the rapids.
A. treacherous B. intrepid C. perilous D. foolhardy
11. Their children have been working their _____ to the bone so they need to take a vacation this
year.
A. fingers B. arms C. brains D. nails

12. I _____ him that he wasn’t telling the truth but he insisted he was.
A. brought it up B. laid it off C. set it on D. put it to

13. She's such a ______ of strength that everyone relies on her in a crisis.

A. tower B. pillar C. post D. support

14. It is far too easy to lay the blame ______ on the shoulders of the management.

A. flatly B. willingly C. squarely D. perfectly

15. Going to the unemployment office and having to wait there for hours is a ______ experience.

A. soul-destroying B. heart-stopping C. power-sharing D. thought-provoking


16. While most people regard Bragg as _________, Gaunt is aware that Bragg has a kind of
silent intelligence, and that he may in fact be one of the cleverest men in the regiment.
A. slow off the mark B. all brawn and no brains
C. at the bottom of the ladder D. larger than life
17. The main cause of the strike was the management’s refusal to give further consideration
to the question of pay ________.
A. nuances B. discrepancies C. differences D. differentials
18. It seems too good to be true. Are you sure this guy’s on the _________?
A. flat B. wagon C. level D. town

19. I am fully prepared for my interview and I am confident that I can answer any questions they
may care to _______ me.

A. throw at B. drop on C. slide to D. roll to

20. Who needs more than the _________necessities of life?


A. plain B. pure C. bare D. sole

Part 2. Supply the correct form of the words given in brackets to complete each of the
sentences below. (10 pts)
1. I am sorry to have bothered you - I was under the (APPREHEND) __________ that you
wanted me to call you.

2. I’ve never met such a strong man. His energy seems (EXHAUST) __________.

3. We should arrive two days early in order to (CLIMATE) __________.

4. Leaving the orphanage, we felt (ACCOUNT) __________ cheerful.

5. Each classroom in my school has been (FIT) __________ with a high-quality projector.

6. Yesterday Tom made a (HARD) __________ speech in the press.

7. The (ELECTRIC) __________ of agriculture is very necessary for all developing countries.

8. He also initially said reservations had been made by his assistants and that his accountant may
have (ERROR) ______ booked some personal expenses as work-related.

9. Enslaved families and their lawyers spoke publicly against the (HUMAN) ______ of slavery.

10. City officials quickly tried to (SOCIAL) ______ that explosion from any broad terrorism
plot.

III. READING (60 POINTS)

Part 1: Read the passage and decide which answer (A, B, C, or D) best fits each gap. (15 pts)

NOTHING IS IMPOSSIBLE

Law firm Matthews and Reynolds is a (1) _____ example of a business using art to revamp its
public image. The firm hired an advertising agency called Eyeopener to carry out a rebranding
(2) _____ and gave the agency (3) _____ rein to take the company by the scruff of the neck and
effect a major makeover. The firm wanted (4) _____, contemporary imagery which would
symbolise an innovative, forward-thinking business.
(5) _____, the firm now has a new logo, and all its advertising material features clever modern
images which are (6) _____ on the eye. Director Alan Ross comments: ‘The images Eyeopener
(7) _____ say a lot about our approach, size and experience. And we were delighted with the
advertising campaign they subsequently (8) _____ using a stylish, sophisticated approach with a
touch of humour here and there.’

Public response to the rebranding has been excellent, and what appeared to be a (9) _____ old
law firm has been given a new lease of (10) _____ as an adventurous and confident concern.

1. A. bright B. polished C. shining D. glossy


2. A. routine B. exercise C. transaction D. function
3. A. extra B. complete C. wide D. free
4. A. smart B. clumsy C. blurry D. waxy
5. A. In the end B. After all C. As a result D. In total
6. A. easy B. attractive C. delightful D. agreeable
7. A. stood up for B. came up with C. got through to D. fell back on
8. A. portrayed B. devised C. imagined D. drafted
9. A. dusty B. tedious C. murky D. monotonous
10. A. fortune B. energy C. time D. life

Part 2. For questions 1-10, fill each of the following numbered blanks with ONE suitable
word and write your answers in the corresponding numbered boxes provided. (15 pts)

The psychology of retailing has come to rely on highly sophisticated techniques. Over
and (1) ________ the design of the shops and the packaging of the merchandise, clever
positioning of goods also ensures that the natural flow of people takes them to (2) _________
and every section in a shop. Customers are led gently, but at the same time with deadly accuracy,
towards the merchandise in (3) ________ a way as to maximise sales.

Manufacturers compete for the right to have their products displayed at the most effective
level. In supermarkets, there is a crucial section in the tiers of vertical shelving somewhere
between waist height and eye (4)________, where we are most likely to take note of a brand. In
the old days, when we went into a shop, we (5) ________our way up to the counter, behind
which would be the shopkeeper and virtually all of the merchandise, and were served (6) ______
what we wanted. Those days are (7) _______ and truly over.

Today, we are used to serving (8) _______ in supermarkets; products are laid before us as
enticingly as possible, and impulse purchases are encouraged (9) _______ a major part of the
exercise. As a result of this, we, as shoppers, have to keep our (10) ________ about us to resist
the retailers’ ploys.

Part 3. Read the following passages and choose the best answer to each question. Write your
answers in the space provided. (15 pts)

Very few people, groups, or governments oppose globalization in its entirely. Instead,
critics of globalization believe aspects of the way globalization operates should be changed. The
debate over globalization is about what the best rules are for governing the global economy so
that its advantages can grow while its problems can be solved.

On one side of this debate are those who stress the benefits of removing barriers to
international trade and investment, and allowing capital to be allocated more efficiently and
giving consumers greater freedom of choice. With free-market globalization, investment funds
can move unimpeded from where they are plentiful (the rich countries) to where they are most
needed (the developing countries). Consumers can benefit from cheaper products because
reduced tariffs make goods produced at low cost from faraway places cheaper to buy. Producers
of goods gain by selling to a wider market. More competition keeps sellers on their toes and
allows ideas and new technology to spread and benefit others.

On the other side of the debate are critics who see neoliberal policies as producing greater
poverty, inequality, social conflict, cultural destruction, and environmental damage. They say
that the most developed nations- the United States, Germany, and Japan- succeeded not because
of free trade but because of protectionism and subsidies. They argue that the more recently
successful economies of South Korea, Taiwan, and China all had strong state-led development
strategies that did not follow neoliberalism. These critics think that government encouragement
of “infant industries”- that is, industries that are just beginning to develop- enables a country to
become internationally competitive.
Furthermore, those who criticize the Washington Consensus suggest that the inflow and
outflow of money from speculative investors must be limited to prevent bubbles. These bubbles
are characterized by the rapid inflow of foreign funds that bid up domestic stock markets and
property values. When the economy cannot sustain such expectations, the bubbles burst as
investors panic and pull their money out of the country. These bubbles have happened repeatedly
as liberalization has allowed speculation of this sort to get out hand, such as in Indonesia,
Malaysia, and Thailand in 1997 and since then in Argentina, Russia, and Turkey. According to
critics, a strong active government is needed to assure stability and economic development.

Protests by what is called the antiglobalization movement are seldom directed against
globalization itself but rather against abuses that harm the rights of workers and the environment.
The question raised by nongovernmental organizations and protesters at WTO and IMF
gathering is whether globalization will result in a rise of living standards or a race to the bottom
as competition takes the form of lowering living standards and undermining environmental
regulation.

1. The passage makes it clear that most of those who oppose globalization __________.
A. blame it for everything B. only blame part of it
C. support neoliberal policies D. ignore the rights of workers

2. The word “unimpeded” in paragraph 2 is closest in meaning to __________.


A. rapid B. unchallenged C. free D. limitless

3. It is stated in the passage that supporters of globalization emphasise the benefits of


__________.
A. limiting consumers’ freedom of B. imposing trade barriers
choice
C. increasing investment in rich countries D. lifting trade barriers

4. According to critics of globalization, one reason the United States, Germany, and Japan
succeeded is that they __________.
A. encouraged free trade in their B. increased tariffs on goods and services
economies
C. sold their goods to smaller markets D. supported domestic industries

5. Which of the following statements would critics of globalization most probably agree with?

A. It is important to remove barriers to international trade.

B. “Infant industries” should not be encouraged.

C. Neoliberal policies do more harm than good.

D. South Korea, Taiwan, and China all followed neoliberalism.

6. The phrase “infant industries” in paragraph 3 mostly means __________.


A. newly successful industries B. state-led industries
C. industries resulting from neoliberal D. industries that are not yet developed
policies

7. According to critics of globalization, the rapid inflow of foreign funds must be controlled to
__________.
A. prevent bubbles that may burst B. support “infant industries”
C. avoid outflow of foreign funds D. prevent overseas investment

8. Protests by the so-called antiglobalization movement are usually directed against __________.

A. globalization itself

B. abuses that harm the rights of workers and the environment

C. the question raised by nongovernmental organizations

D. a rise of living standards

9. Which of the following is NOT mentioned in the passage?


A. Critics of globalization think neoliberal policies are producing more property.

B. The antiglobalization movement is set up to stop globalization.

C. Some Asian countries had strong state-led development strategies.

D. Critics of globalization believe the way it operates should be changed.

10. As described in the passage, what is the debate over globalization mostly about?

A. How to govern the global economy most successfully

B. How to stop globalization and its effects

C. Ideas and technology to spread globalization

D. How to hold protests against neoliberal policies

Part 4. Read the text below carefully and then do the tasks that follow. (15 pts)

Why being bored is stimulating – and useful, too

This most common of emotions is turning out to be more interesting than we thought

A We all know how it feels – it’s impossible to keep your mind on anything, time stretches
out, and all the things you could do seem equally unlikely to make you feel better. But defining
boredom so that it can be studied in the lab has proved difficult. For a start, it can include a lot of
other mental states, such as frustration, apathy, depression and indifference. There isn’t even
agreement over whether boredom is always a low-energy, flat kind of emotion or whether feeling
agitated and restless counts as boredom, too. In his book, Boredom: A Lively History, Peter
Toohey at the University of Calgary, Canada, compares it to disgust – an emotion that motivates
us to stay away from certain situations. ‘If disgust protects humans from infection, boredom may
protect them from “infectious” social situations,’ he suggests.
B By asking people about their experiences of boredom, Thomas Goetz and his team at the
University of Konstanz in Germany have recently identified five distinct types: indifferent,
calibrating, searching, reactant and apathetic. These can be plotted on two axes – one running left
to right, which measures low to high arousal, and the other from top to bottom, which measures
how positive or negative the feeling is. Intriguingly, Goetz has found that while people
experience all kinds of boredom, they tend to specialise in one. Of the five types, the most
damaging is ‘reactant’ boredom with its explosive combination of high arousal and negative
emotion. The most useful is what Goetz calls ‘indifferent’ boredom: someone isn’t engaged in
anything satisfying but still feels relaxed and calm. However, it remains to be seen whether there
are any character traits that predict the kind of boredom each of us might be prone to.

C Psychologist Sandi Mann at the University of Central Lancashire, UK, goes further. ‘All
emotions are there for a reason, including boredom,’ she says. Mann has found that being bored
makes us more creative. ‘We’re all afraid of being bored but in actual fact it can lead to all kinds
of amazing things,’ she says. In experiments published last year, Mann found that people who
had been made to feel bored by copying numbers out of the phone book for 15 minutes came up
with more creative ideas about how to use a polystyrene cup than a control group. Mann
concluded that a passive, boring activity is best for creativity because it allows the mind to
wander. In fact, she goes so far as to suggest that we should seek out more boredom in our lives.

D Psychologist John Eastwood at York University in Toronto, Canada, isn’t convinced. ‘If you
are in a state of mind-wandering you are not bored,’ he says. ‘In my view, by definition boredom
is an undesirable state.’ That doesn’t necessarily mean that it isn’t adaptive, he adds. ‘Pain is
adaptive – if we didn’t have physical pain, bad things would happen to us. Does that mean that
we should actively cause pain? No. But even if boredom has evolved to help us survive, it can
still be toxic if allowed to fester.’ For Eastwood, the central feature of boredom is a failure to put
our ‘attention system’ into gear. This causes an inability to focus on anything, which makes time
seem to go painfully slowly. What’s more, your efforts to improve the situation can end up
making you feel worse. ‘People try to connect with the world and if they are not successful
there’s that frustration and irritability,’ he says. Perhaps most worryingly, says Eastwood,
repeatedly failing to engage attention can lead to state where we don’t know what to do any
more, and no longer care.
E Eastwood’s team is now trying to explore why the attention system fails. It’s early days but
they think that at least some of it comes down to personality. Boredom proneness has been
linked with a variety of traits. People who are motivated by pleasure seem to suffer particularly
badly. Other personality traits, such as curiosity, are associated with a high boredom threshold.
More evidence that boredom has detrimental effects comes from studies of people who are more
or less prone to boredom. It seems those who bore easily face poorer prospects in education, their
career and even life in general. But of course, boredom itself cannot kill – it’s the things we do to
deal with it that may put us in danger. What can we do to alleviate it before it comes to that?
Goetz’s group has one suggestion. Working with teenagers, they found that those who
‘approach’ a boring situation – in other words, see that it’s boring and get stuck in anyway –
report less boredom than those who try to avoid it by using snacks, TV or social media for
distraction.

F Psychologist Francoise Wemelsfelder speculates that our over-connected lifestyles might even
be a new source of boredom. ‘In modern human society there is a lot of overstimulation but still
a lot of problems finding meaning,’ she says. So instead of seeking yet more mental stimulation,
perhaps we should leave our phones alone, and use boredom to motivate us to engage with the
world in a more meaningful way.

Questions 14-19

Reading Passage 2 has six paragraphs, A-F

Choose the correct heading for each paragraph from the list of headings below.

Write the correct number, i-viii, in boxes 14-19 on your answer sheet.

List of Headings

i The productive outcomes that may result from boredom

ii What teachers can do to prevent boredom

iii A new explanation and a new cure for boredom


iv Problems with a scientific approach to boredom

v A potential danger arising from boredom

vi Creating a system of classification for feelings of boredom

vii Age groups most affected by boredom

viii Identifying those most affected by boredom

1 Paragraph A

2 Paragraph B

3 Paragraph C

4 Paragraph D

5 Paragraph E

6 Paragraph F

Questions 7-10 Look at the following people (Questions 7-10) and the list of ideas below.

Match each person with the correct idea, A-E. Write the correct letter, A-E, in boxes 7-10
on your answer sheet.

7 Peter Toohey

8 Thomas Goetz

9 John Eastwood

10 Francoise Wemelsfelder

List of Ideas
A The way we live today may encourage boredom.

B One sort of boredom is worse than all the others.

C Levels of boredom may fall in the future.

D Trying to cope with boredom can increase its negative effects.

E Boredom may encourage us to avoid an unpleasant experience.

IV. WRITING (50 POINTS)

Part 1.Rewrite the following sentences using the words given

61. Your encouragement helped to make things less grievous after such a heavy loss.

CUSHION

=> It was ___________________________________________________________________

62. The government recommends a balance of reward and punishment when dealing with

young offenders. STICK

=> The government favours ___________________________________________________

63. It was such bad coffee that he couldn’t drink it even a little. TAKE

=> So bad __________________________________________________________________

64. She wants nothing less than to get that job. It would be a dream come true. SET

=> She has __________________________________________________________________

5. After the expedition, I realized that she was not an efficient researcher. SHAKES

=> It dawned__ ______________________________________________________________


Part 2. The following graphs give information about book-buying over 30 years in New York.
Summarize the information by selecting and reporting the main features, and make
comparisons where relevant. You should write between 130 and 160 words. (20 pts)

Types of books bought per person per year

Part 3. Essay writing (30 pts)

As some dominant languages are more widely spoken all over the world, many minority
languages are dying out. Some people think that it is unnecessary to save these languages. To
what extent do you agree with this perspective?

Write an essay of about 250 words to express your viewpoint. Support your opinion by using
relevant examples and explanations.

TEST 8

I: LISTENING (50 points)

Part 1: Based on the talk you hear, complete the table below. Write NO MORE THAN
THREE WORDS OR A NUMBER for each answer. (10 points)
THE BRITISH LIBRARY
Commenced operation 1st July 1973
Four main constituent parts
The Library of the British Museum

Patent Office Library

1. _____________________________

The British Library Document Supply Centre


The British Museum founded in
1753
The British Library receives a copy of
many books

journals, magazines and newspapers

2. _____________________________

Famous users of the 3. maps


____________________________
Vladimir Lenin, Karl Max, Charles Dickens, George
The British Library Document Supply Bernard Shaw, and Virginia Woolf
Centre has almost 5.000.000

The British Library’s website

4. _____________________________

5. www.__________________________

Your answer:

1. __________ 2. __________ 3. ___________ 4. ___________ 5. __________


Part 2: Listen to a report about how European countries are dealing with the coronavirus
pandemic and decide whether these statements are True or False. Write your answers to the
space provided. (10 points)

1. Under the full lockdown in Italy, all grocery stores must be shut down.

2. There is a ban on entry for people from Austria into Italy.

3. Angela Merkel warned that Coronavirus could infect up to 17%of Germany's population.

4. Public gatherings still take place in the UK.

5. According to the professor, although the UK is taking more drastic measures than Italy, its
effectiveness is open to question.
Your answer:

1. __________ 2. __________ 3. ___________ 4. ___________ 5. __________

Part 3. You will hear an interview with a woman called Emma Stoneham, who

works as a manager in the horse racing industry. Choose the answer (A, B, C or D)

which fits best according to what you hear. (10 points)

1. What does Emma say about her qualifications?

A. She chose to study certain subjects against her father's wishes.


B. She decided to do courses that dealt directly with horse racing.
C. She did a postgraduate course that few people knew about.

D. She was fortunate to be accepted on a specialist course.

2. How does Emma feel about what she calls the 'big names' in horse racing?
A. She resents having to plan her events to suit them.

B. She respects the contribution they make to the sport.

C. She believes that they shouldn't receive special treatment.

D. She accepts that she needs to put on particular events to attract them.

3. What does Emma particularly enjoy about race days?

A. the range of people she meets

B. the enthusiasm of her colleagues

C. the challenge of dealing with the unexpected

D. the pleasure of seeing people enjoying themselves

4. How did Emma feel after cancelling the midwinter race meeting?

A. worried that inadequate precautions had been taken


B. angry that bad weather hadn't been predicted

C. satisfied that she'd made the right decision

D. grateful for the good advice she'd received

5. As a result of cancelling the meeting, Emma has had to ______.

A. put on additional race meetings

B. make changes to her financial planning

C. sell some land belonging to the racecourse


D. postpone making improvements to facilities
Your answer:
1. __________ 2. __________ 3. ___________ 4. ___________ 5. __________

Part 4. Listen to a recording about the origins of the anti-vaccine movement and complete the
summary below using words taken from the recording. Write NO MORE THAN THREE
WORDS in each blank. (20 points)

- The presence of (1)__________ can be traced back to the infancy of vaccines.

- Although he was (2) __________ from his community, Edward Jennerinspired the invention of
(3)__________ with his action of (4)__________ from infected cows into human skin.

- People are still worried about (5) __________ being injected into the human body.

- Doctors had to persuade parents to vaccinate their kids on the promise that they will not be
infected by some(6)__________.

- A discredited studypropagated the false belief that (7)__________ and vaccines are connected.

- The infections among the Amish of Ohio, which contributed to the 2014’s (8)__________ in
measles case, serves as a stark example of how a single (9)__________ traveller from a certain
hotspot can (10)__________.
Your answer:

1. __________ 2. __________ 3. __________ 4. __________ 5. __________

6. __________ 7. __________ 8. ___________ 9. ___________ 10. _________


II: LEXICO - GRAMMAR (40 points)
Part 1: Choose one of the words marked A, B, C, or D which best completes the sentence.
(20 points)

1. Getting my car fixed is going to cost me _________ as the engine has completely blown, but I
have no choice. I need it for work.

A. a drop in the ocean B. an arm and a leg

C. a piece of cake D. a blessing in disguise

2. The policeman chewed out the driver for ________ driving.

A. intense B. reckless C. excessive D. tedious

3. I can’t buy it at that price because I wouldn’t be able to sell it _______ a profit.

A. with B. at C. in D. for

4. She was ________ disappointed when she found out that she and Seb would not be living under
the same roof..

A. bitterly B. deeply C. painfully D. deadly

5. Though you may have training as a radiological technician, you can still ________ to an

executive role.

A. make a career change easy B. make a career change easier

C. make easy a career change D. easily make a career change

6. I have always had the greatest ________ for people who do voluntary work.

A. praise B. respect C. favor D. honor

7. The people of this country fight ________ to protect their common homeland.

A. doom and gloom B. life and death

C. rack and ruin D. tooth and nail

8. She succeeded in making the company a top performer by ______ off unprofitable assets.

A. slicing B. draining C. stripping D. removing

9. Kei Tari was a Japanese comedian and musician ________.

A. who melded seamless jazz and humor


B. who seamlessly melded jazz and humor

C. who was known for his melding of seamless jazz and humor

D. who, with jazz and humor, seamlessly melded the two

10. _______ the pastry thinly and cover the dish with it.

A. Press B. Flatten C. Roll D. Stretch

11. John’s report was thorough and insightful. He deserves a real _______

A. slap on the wrist B. pat on the back

C. pain in the neck D. peck on the cheek

12. _________, the diners settled the bill and left the restaurant.

A. Having hunger satisfied B. Their hunger satisfied

C. Hunger been satisfied D. Satisfying their hunger

13. Apart from a parking fine ten years before, she had an __________ driving record.

A. unsullied B. unalloyed C. unblemished D. untarnished

14. Everyone was ___________ with him after he missed the penalty in the last 2 minutes

of the final match.

A. lamenting B. commiserating C. condoling D. identifying

15. We shall need also to ________ the foundations of mathematics, and even to question the very
nature of physical reality.

A. pertain to B. touch on C. delve into D. slur over

16. When she saw me nearly dropped the precious vase, she _________ in horror.

A. grasped B. grunted C. grumbled D. gasped

17. They have _________ a beautiful film by making these changes.

A. disfigured B. mutilated C. defaced D. amputated


18. You can try reformatting your computer, but once you open that _________, you’ll

probably be working on it for days.

A. apple of discord B. can of worms C. load of cobblers D. spot of bother

19. _________ nocturnal creature(s).

A. Owl is a B. An owl is a C. The owl is a D. The owls are

20. It’s natural for students to worry about whether they will ________ or not at a new school.

A. measure up B. sprout up C. show up D. dig up


Your answer:

1. __________ 2. __________ 3. __________ 4. __________ 5. __________


6. __________ 7. __________ 8. __________ 9. __________ 10. _________
11. _________ 12. _________ 13. _________ 14. _________ 15. _________
16. _________ 17. _________ 18. _________ 19. _________ 20. _________

Part 2: Use the correct form of each of the words given to fill in the blank in each sentence.
(10 points)
1. She’s family suffered from his ………………….. (EXPEND)
2. The Southerners were like the colonists in the Revolutionary War in that they were
_____________ but geographically protected. (MAN)
3. My ankle is really ............................and I can’t walk easily. (SWELL)
4. The government’s............................. approach has brought criticism. (COMPROMISE)
5. Jim is one of the most................................ members of the committee. (SPEAK)
6. You shouldn’t interrupt someone in …………………… (SENTENCE)
7. Low income and little administrative support make teachers …………. with their profession.
(HEART)
8. Don’t you think she is ______? She always looks good in her photographs. (PHOTO)
9.Hardly a day goes by without Tim being __________ of eating sweets. (DESIRE)
10.She resembled an army commander whom nothing could put the __________ on.
(FRIGHTEN)
Your answer:

1. __________ 2. __________ 3. __________ 4. __________ 5. __________


6. __________ 7. __________ 8. __________ 9. __________ 10. _________

III: READING COMPREHENSION (60 points)


Part 1: Read the following passages and decide which answer (A, B, C, or D) best fits each gap.
Write your answers in corresponding numbered boxes. (15 points)
HEARING IN COLOUR
A number of scientists around the world are now investigating a phenomenon called synaesthesia
that may (1) _______as many as one in 2,000 people. The name (2) _______from the Greek
words for together and perception and means that some people’s senses work in combination. For
example, some people (3) _______colour when they hear particular sounds. Similarly, a smell or
taste may be (4) _______as a reaction to information received from the eyes. However, the most
common form of synaesthesia occurs among people who (5) _______certain letters or words with
colours. Scientists at Cambridge University conducted experiments to determine whether this is
actually a product of mental activity or if some individuals are just (6)_______imaginative. They
discovered that synaesthetes, people who experience synaesthesia, (7) _______ associate the same
letters or words with the same colours. Brain scans revealed (8) _______ activity in the brain
when subjects were listening to words, suggesting that it is a physical condition. The most
plausible explanation is that synaesthetes have slightly different connections between the areas of
the brain which control their (9) _______. Synaesthesia is not a medical problem, however, and
synaesthetes often (10) _______from an unusually good memory, probably because they have
extra information to help them recallthings like names and numbers.
1. A. effect B . infect C. suffer
C D . affect
2. A. reminds B . derives C. prescribes D . distracts
3. A. differ B . view C. see D . mind
4. A. retained B . perceived C. thought D . responded
5. A. associate B . elaborate C. conceive D . comply
6. A. deeply B . utterly C. highly D . fully
7. A. perfectly B . earnestly C. practically D .consistently

8. A. unusual B . infallible C. insecure D .incapable


9. A. consciences B . attitudes C. senses D .conditions
10. A. approve B . sting C. cure D .benefit
Your answer:
1. __________ 2. __________ 3. __________ 4. __________ 5. __________
6. __________ 7. __________ 8. __________ 9. __________ 10. _________

Part 2: Read the following passage and choose the best answer to each question. Write your
answers in the corresponding numbered boxes provided below the passage. (15 points)
How can I cope better with stress?
Recent research found that having higher levels of self-esteem, a more positive way of explaining
why things happen, and avoiding perfectionist thinking were strongly 1._________ to bouncing
back when things go wrong.
To begin with, according to Dr Judith Johnson self-esteem was shown to be 2. __________ in two
thirds of the studies in mediating the link between failure and distress. All we need to do is write a
list of our positive qualities and examples of when we have 3. _______them.
Secondly, how we explain things also 4.______. If we can brush off a disappointment by
5.____________ it to external factors, then we have the equivalent of emotional armour.
Finally, if we have perfectionist tendencies, lower them. Perfectionists hold rigid standards that
don’t bend in the stormy 6. _________of life. Johnson says we need to set more realistic 7.
___________.
She also advises self-awareness and noticing when we are stressed. If you feel stuck, do
something that 8._________ you up. You need to plan these things into a schedule – doing them
will chip away at your negative 9. ________, even if you do enjoy them a bit less than usual.
Other research shows that the 10. __________ of friends or family also helps emotional resilience,
as does being physically active.
Your answer:
1. __________ 2. __________ 3. __________ 4. __________ 5. __________
6. __________ 7. __________ 8. __________ 9. __________ 10. _________

Part 3: Read the following passage and choose the best answer to each question. Write your
answers in the corresponding numbered boxes provided below the passage. (15 points)
Archaeological records-paintings, drawings, and carvings of humans engaged in activities
involving the use of hands - indicate that humans have been predominantly right-handed for more
than 5,000 years. In ancient Egyptian artwork, for example, the right hand is depicted as the
dominant one in about 90 percent of the examples. Fracture or wear patterns on tools also indicate
that a majority of ancient people were right-handed.
Cro-Magnon cave paintings some 27,000 years old commonly show outlines of human hands
made by placing one hand against the cave wall and applying paint with the other. Children today
make similar outlines of their hands with crayons on paper. With few exceptions, left hands of
Cro-Magnons are displayed on cave walls, indicating that the paintings were usually done by
right-handers. Anthropological evidence pushes the record of handedness in early human
ancestors back to at least 1.4 million years ago. One important line of evidence comes from
flaking patterns of stone cores used in toolmaking: implements flaked with a clockwise motion
(indicating a right-handed toolmaker) can be distinguished from those flaked with a counter-
clockwise rotation (indicating a left-handed toolmaker).
Even scratches found on fossil human teeth offer clues. Ancient humans are thought to have cut
meat into strips by holding it between their teeth and slicing it with stone knives, as do the
present-day Inuit. Occasionally the knives slip and leave scratches on the users' teeth. Scratches
made with a left-to-right stroke direction (by right-handers) are more common than scratches in
the opposite direction (made by left-handers).
Still other evidence comes from cranial morphology: scientists think that physical differences
between the right and left sides of the interior of the skull indicate subtle physical differences
between the two sides of the brain. The variation between the hemispheres corresponds to which
side of the body is used to perform specific activities. Such studies, as well as studies of tool use,
indicate that right- or left-sided dominance is not exclusive to modern Homo sapiens. Populations
of Neanderthals, such as Homo erectus and Homo habilis, seem to have been predominantly right-
handed, as we are.
1. What is the main idea of the passage?
A. Human ancestors became predominantly right-handed when they began to use tools.
B. It is difficult to interpret the significance of anthropological evidence concerning tool use.
C. Humans and their ancestors have been predominantly right-handed for over a million years.
D. Human ancestors were more skilled at using both hands than modern humans.
2. What does the author say about Cro-Magnon paintings of hands?
A. Some are not very old.
B. It is unusual to see such paintings.
C. Many were made by children.
D. The artists were mostly right-handed.
3. The word "depicted" in the first paragraph refers to ________.
A. written B. portrayed C. referred D. mentioned
4. When compared with implements "flaked with a counter-clockwise rotation”, it can be
inferred that "implements flaked with a clockwise motion" are ______.
A. more common B. larger
C. more sophisticated D. older
5. The word "cranial morphology" in the last paragraph is closest in meaning to ______.
A. the form of crane B. the form of study
C. the study of physical bodies D. the study of skulls
6. The fact that the Inuit cut meat by holding it between their teeth is significant because ______.
A. the relationship between handedness and scratches on fossil human teeth can be verified
B. it emphasizes the differences between contemporary humans and their ancestors
C. the scratch patterns produced by stone knives vary significantly from patterns produced by
modern knives
D. it demonstrates that ancient humans were not skilled at using tools
7. The word "hemispheres" in the last paragraph is closest in meaning to _______.
A. differences B. sides C. activities D. studies
8. Why does the author mention Homo erectus and Habilis in the last paragraph?
A. To contrast them with modern humans
B. To explain when human ancestors began to make tools
C. To show that early humans were also predominantly right-handed
D. To prove that the population of Neanderthals was very large
9. All of the following are mentioned as types of evidence concerning handedness EXCEPT
_______.
A. ancient artwork B. asymmetrical skulls
C. studies of tool use D. fossilized hand bones
10. Which of the following conclusions is suggested by the evidence from cranial morphology?
A. Differences in the hemispheres of the brain probably came about relatively recently
B. There may be a link between handedness and differences in the brain's hemispheres.
C. Left-handedness was somewhat more common among Neanderthals.
D. Variation between the brain's hemispheres was not evident in the skulls of Homo erectus and
Homo habilis.
Your answer:
1. __________ 2. __________ 3. __________ 4. __________ 5. __________
6. __________ 7. __________ 8. __________ 9. __________ 10. _________

Part 4: Read the following passage and do the tasks that follow. (15 points)
Wealth in A Cold Climate
Latitude is crucial to a nation's economic strength.
A Dr William Masters was reading a book about mosquitoes when inspiration struck. “There
was this anecdote about the great yellow-fever epidemic that hit Philadelphia in 1793,"
Masters recalls. “This epidemic decimated the city until the first frost came." The inclement
weather froze out the insects, allowing Philadelphia to recover.
B If weather could be the key to a city's fortunes. Masters thought, then why not to the historical
fortunes of nations? And could frost lie at the heart of one of the most enduring economic
mysteries of all - why are almost all the wealthy, industrialised nations to be found at latitudes
above 40 degrees? After two years of research, he thinks that he has found a piece of the
puzzle. Masters, an agricultural economist from Purdue University in Indiana, and Margaret
McMillan at Tufts University, Boston, show that annual frosts are among the factors that
distinguish rich nations from poor ones. Their study is published this month in the Journal of
Economic Growth. The pair speculate that cold snaps have two main benefits - they freeze
pests that would otherwise destroy crops, and also freeze organisms, such as mosquitoes, that
carry disease. The result is agricultural abundance and a big workforce.
C The academics took two sets of information. The first was average income for countries, the
second climate data from the University of East Anglia. They found a curious tally between
the sets. Countries having five or more frosty days a month are uniformly rich, those with
fewer than five are impoverished. The authors speculate that the five-day figure is important;
it could be the minimum time needed to kill pests in the soil. Masters says: “For example,
Finland is a small country that is growing quickly, but Bolivia is a small country that isn't
growing at all. Perhaps climate has something to do with that." In fact, limited frosts bring
huge benefits to farmers. The chills kill insects or render them inactive; cold weather slows the
break-up of plant and animal material in the soil, allowing it to become richer; and frosts
ensure a build-up of moisture in the ground for spring, reducing dependence on seasonal rains.
There are exceptions to the “cold equals rich" argument. There are well-heeled tropical places
such as Hong Kong and Singapore, a result of their superior trading positions. Likewise, not
all European countries are moneyed - in the former communist colonies, economic potential
was crushed by politics.
D Masters stresses that climate will never be the overriding factor - the wealth of nations is too
complicated to be attributable to just one factor. Climate, he feels, somehow combines with
other factors - such as the presence of institutions, including governments, and access to
trading routes - to determine whether a country will do well. Traditionally, Masters says,
economists thought that institutions had the biggest effect on the economy, because they
brought order to a country in the form of, for example, laws and property rights. With order,
so the thinking went, came affluence. “But there are some problems that even countries with
institutions have not been able to get around,” he says. “My feeling is that, as countries get
richer, they get better institutions. And the accumulation of wealth and improvement in
governing institutions are both helped by a favourable environment, including climate.”
E This does not mean, he insists, that tropical countries are beyond economic help and destined
to remain penniless. Instead, richer countries should change the way in which foreign aid is
given. Instead of aid being geared towards improving governance, it should be spent on
technology to improve agriculture and to combat disease. Masters cites one example: “There
are regions in India that have been provided with irrigation - agricultural productivity has gone
up and there has been an improvement in health.” Supplying vaccines against tropical diseases
and developing crop varieties that can grow in the tropics would break the poverty cycle.
F Other minds have applied themselves to the split between poor and rich nations, citing
anthropological, climatic and zoological reasons for why temperate nations are the most
affluent. In 350BC, Aristotle observed that “those who live in a cold climate ... are full of
spirit”. Jared Diamond, from the University of California at Los Angeles, pointed out in his
book Guns, Germs and Steel that Eurasia is broadly aligned east-west, while Africa and the
Americas are aligned north-south. So, in Europe, crops can spread quickly across latitudes
because climates are similar. One of the first domesticated crops, einkorn wheat, spread
quickly from the Middle East into Europe; it took twice as long for com to spread from
Mexico to what is now the eastern United States. This easy movement along similar latitudes
in Eurasia would also have meant a faster dissemination of other technologies such as the
wheel and writing, Diamond speculates. The region also boasted domesticated livestock,
which could provide meat, wool and motive power in the fields. Blessed with such natural
advantages, Eurasia was bound to take off economically.
G John Gallup and Jeffrey Sachs, two US economists, have also pointed out striking correlations
between the geographical location of countries and their wealth. They note that tropical
countries between 23.45 degrees north and south of the equator are nearly all poor. In an
article for the Harvard International Review, they concluded that “development surely seems
to favour the temperate-zone economies, especially those in the northern hemisphere, and
those that have managed to avoid both socialism and the ravages of war”. But Masters
cautions against geographical determinism, the idea that tropical countries are beyond hope:
“Human health and agriculture can be made better through scientific and technological
research," he says, “so we shouldn’t be writing off these countries. Take Singapore: without
air conditioning, it wouldn’t be rich.”
Questions 1-6
Choose the most suitable heading for paragraphs A-G from the list of headings below. Write
the appropriate number, i-x, in boxes 1-6 on your answer sheet.
Example answer List of Headings
i. The positive correlation between climate and wealth
Paragraph A: iii
ii. Other factors besides climate that influence wealth
iii. Inspiration from reading a book
1. Paragraph B
iv. Other researchers’ results do not rule out exceptional cases
2. Paragraph C v. Different attributes between Eurasia and Africa
vi. Low temperature benefits people and crops
3. Paragraph D
vii. The importance of institution in traditional views
viii. The spread of crops in Europe, Asia and other places
4. Paragraph E
ix. The best way to use aid
x. Confusions and exceptions
5. Paragraph F

6. Paragraph G

Questions 7-10
Complete the summary below. Choose NO MORE THAN TWO WORDS from the passage for
each answer. Write your answers in boxes 7-10 on your answer sheet.
Dr William Masters read a book saying that a(an) 7. _______which struck an American city
hundreds of years ago was terminated by a cold frost. And academics found that there is a
connection between climate and country’s wealth as in the rich but small country of Finland. Yet
besides excellent surroundings and climate, one country still needs to improve their 8. ______to
achieve long prosperity.
Thanks to resembling weather conditions across latitude in the continent of 9. ______, crops such
as einkorn wheat is bound to spread faster than from South America to the North. Other
researchers also noted that even though geographical factors are important, tropical country such
as 10. ______ still became rich due to scientific advancement.

Your answer:
1. __________ 2. __________ 3. __________ 4. __________ 5. __________
6. __________ 7. __________ 8. __________ 9. __________ 10. _________

IV: WRITING (40 points)


Part 1.Rewrite the following sentences using the words given
1. You have to pay taxes or go to prison. PUT
=> Failing___________________________________________________________________
2. Everyone was surprised that the work completely failed. DOWN
=> To ______________________________________________________________________
3. When the goods arrive at the shop, they will be inspected carefully. MICROSCOPE
=> On ____________________________________________________________________
4. I think you should confront your boss right now with how you feel about this. OUT
=> Why ___________________________________________________________________
5. If something’s worrying you, you should tell about it now. OFF
=> If you have______________________________________________________________

Part 2. The diagram shows the number of cars stolen in different countries. Write a 150-word
report to identify the main trends and make comparisons where relevant.(20 points)

Part 3: Some people think the qualities a person needs to become successful in today’s world
cannot be learned at a university or similar academic institution. To what extent do you agree or
disagree ? Write an essay (about 200-250 words) to express your viewpoint. (30 points)

TEST 9

PART 1. LISTENING (50 POINTS)

Section 1. (10 points)

Complete the form. Write NO MORE THAN THREE WORDS for each answer.
PERSONAL DETAILS FORM

Name: Mary (1) ……….

Address: Flat 2 (2) ………. (3) ………. Road, Canterbury

Telephone: (4) ……….

Estimated value of lost items: (5) £ ……….

Section 2. (10 points)

Decide whether the following statements are True (T) or False (F).

6. The swimming pool has eight lanes.

7. Some new fitness equipment has recently been added.

8. It is necessary to give a detailed account of any illnesses you have in the questionnaire.

9. You have to learn the safety rules by yourself.

10. You should spend one month to achieve all the goals planned by the trainer.

Section 3. (10 points)

Choose the answer A, B, C, or D which fits best according to what you hear.

11. Tony says that Time Out was unlike other publications in 1968 because

A. it was written by one person.

B. information was more accurate.

C. it had a comprehensive list of events.

D. it was in the form of a magazine.

12. What experience did Tony have of publishing?

A. He had worked for What’s On.

B. He had written numerous articles.

C. He had transformed an existing magazine.


D. He had started a student magazine.

13. Why did Tony leave university?

A. He wanted to go to France.

B. He didn’t have time to study.

C. He had failed his French examinations.

D. He had found an alternative career.

14. According to Tony, what led to the magazine becoming a weekly?


A. some market research B. the quantity of information
C. technical improvements D. external pressure

15. Tony says the big publishers were not interested in this type of magazine because
A. it was popular with students. B. it was considered too expensive.
C. it came out too frequently. D. it threatened their publications.

Section 4. (20 points)

Fill in each gap with NO MORE THAN TWO WORDS taken from the passage.

STELLA PRIME: MOUNTAINEER

On her first expedition, Stella became aware of feelings of (16) _____ and (17) _____ connected
with mountaineering. Stella’s family (18) _____ by her new interest because she had previously
taken part in several so-called (19) _____. Stella found the (20) _____ for climbing Everest
particularly hard. On her second expedition, Stella was worried about the (21) _____ she would
have to climb through. Stella had regretted taking (22) _____ with her on her first expedition.
She says that she didn’t take a (23) _____ with her beyond a certain altitude. Stella uses the word
(24) _____ to discribe the feelings of her party on reaching the summit. Stella’s book about her
experiences is entitled (25) _____.

PART B. LEXICO - GRAMMAR (40 points)

Part 1. Choose one of the words marked A, B, C, or D which best completes each of the
following sentences. Write your answers in the corresponding numbered boxes. (20 points)

1. Mike __________ the news at school, but you should call and tell him –just in case.
A. should have heard B. could have heard

C. shouldn’t have heard D. musn’t have heard

2. We should participate in the movements _______ the natural environment.

A. organizing to conserve B. organized conserving

C. which organize to conserve D. organized to conserve

3. Plastic pollution is currently one of the biggest problems _________affecting the marine
environment with an estimated 8 million tonnes of plastic entering the ocean annually.

A. affecting B. affected C. having affected D. to have affected

4. Finding the most cost-effective measure to fight against the Covid 19 outbreak is a

______ debated issue.

A. heavily B. profoundly C. hotly D. deeply

5. I don’t suppose there is anyone there, _______?.

A. do I B. isn’t there C. is there D. don’t I

6. He asked me if I had taken ________ the previous semester.

A. house economics B. home economic C. house economic D. home economics

7. ___________, he remained optimistic.

A. Badly wounded as he was B. Badly as he was wounded

C. Though badly wounded he was D. As he was badly wounded

8. The salary of a bus driver is much higher __________.

A. to compare with a teaher B. than that of a teacher

C. than a teacher D. than the teacher is

9. After living one month in New York, I finally _____

A. get into the swing of things B. get in on the ground floor

C. get a grip on myself D. get ahead of myself


10. It is imperative that your facebook password _________confidential

A. need keeping B. need to keep

C. must be kept D. needs keeping

11. Life is so full of both good fortune and misfortune that you have to learn to take the rough
with the _______.

A. smooth B. ready C. calm D. tough

12. The new staff went to great _______ to complete a high quality presentation.

A. difficulty B. efforts C. torment D. pains

13. I’ve been ____________over this decision for ages but I’ve definitely made up my mind
now.

A. rejoicing B. moaning C. agonising D. reckoning

14. ____________ with many personal responsibilities can help students to establish their
independence will make them more flexible in working environment.

A. That living alone B. Living alone

C. What living alone D. To live alone

15. After a great deal of hunting around with flashlights, the plain-clothes policemen were forced
to go away ____________.

A. blank-handed B. clear-handed C. empty-hande D. white-handed

16. Many people are now between a__________and a hard place. They risk being infected with
COVID-19 if they venture outside, but they cannot stay inside all the time.

A. wall B. block C. stone D. rock

17. As a matter of __________ we have six security guards on the premises at all times.

A. prudence B. wisdom C. foresight D. acumen

18. The purpose of the survey was to ________ the inspectors with local conditions.

A. inform B. acquaint C. instruct D. notify

19. They're blowing________over this issue. It's impossible to know what they want!
A. dry and wet B. hot and cold C. warm and cool D. light and heavy

20. I just got a good job for the summer- I don't know if I'll have one this fall, but I’ll
make________while the sun shines and save up for the winter.

A. grass B. weed C. bush D. hay

Your answers:
1. 5. 9. 13. 17.
2. 6. 10. 14. 18.
3. 7. 11. 15. 19.
4. 8. 12. 16. 20.

Part 2. Give the correct form of the words in brackets. Write your answers in the
corresponding numbered boxes. (10 points)

1. With the expansion of agriculture, forests were transformed into ______________and grazing
areas, resulting in significant changes in the landscape. (CULTIVATION)

2. We have long known that hearing loss is America’s number one ________health problem
(FATE)

3. Poor health has __________him from working all his life. (CAPACITY)

4. He's at a very _________ age and just tends to follow all the boys. (IMPRESS)

5. The __________of the process will depend on time available the consumer’s ability to handle
and evaluate he information, and the perceived benefits of doing so. (EXTEND)

6. Some argue ______________ is dissolving the glue that has traditionally held companies
together, and without with they may never flourish. (SIZE)

7. Nothing is more __________than being completely dependent on another person.


(DESTROY)
8. My friends started going out late to nightclubs so I decided to _________myself from the
group. (SOCIAL)

9. One way to make your furniture products __________is by differentiating yourself with high
quality visuals. (STAND)

10. The world is becoming increasingly _____________(DEPEND)

Your answers:
1. 6.
2. 7.
3. 8.
4. 9.
5. 10.

PART C: READING COMPREHENSION (60 points)

Part 1. Read the following passage and decide which answer (A, B, C, or D) best fits each gap.
Write your answers in corresponding numbered boxes. (15 points)

TURN OFF THE HEATING

By 1_________ your hand into a freezing cold bucket of water, you can discover how

quickly your body burns calories. This basic experiment is the starting point for a pioneering

study into the effects of brown- or good - fat in the body.

Everyone is born with brown fat around the shoulder 2________. It is central to keeping a

baby’s body temperature on an even 3________ by using up this 4__________ of fat in

order to keep babies warm. Scientists, though, have long believed that this brown fat

vanishes as babies grow out of 5___________ and it is no longer needed.

However, a few years ago, researchers were carrying out scans 6________ adults during

the winter and realized there were 7__________ of fat that seemed to have been
8________ by the cold weather. This discovery has encouraged scientists to 9_________

further research in the hope it is the 10__________ gun that will help solve weight

problems amongst the obese. They have already determined that the cold, certain foods

and exercise can activate this brown fat to people’s benefit.

1. A. prodding B. thrusting C. propelling D. heaving

2. A. blades B. joints C. bones D. sockets

3. A. path B. track C. course D. keel

4. A. store B. storation C. storage D. storing

5. A. development B. infancy C. adolescent D. childhood

6. A. over B. for C. on D. off

7. A. stretches B. areas C. tracts D. pockets

8. A. developed B. formed C. provoked D. triggered

9. A. make B. conduct C. perform D. absorb

10. A. grease B. zip C. smoking D. light

Your answers
1. 2. 3. 4. 5.
6. 7. 8. 9. 10.

Part 2. Read the text below and think of the word which best fits each space. Use only ONE
word in each space. Write your answers in the corresponding numbered boxes. (15 points)

THE SOLAR SYSTEM

The Solar System may be defined as consisting of all those objects that are governed by the Sun's
gravitational 1_________. Other effects arising 2_________ the proximity of the Sun could
equally 3_________ be used as criteria, such as radiation pressure or interaction with the solar
wind. 4__________ any of these definitions the Solar System extends 5________ to a distance
of about two light-years; the closest star, Proxima Centauri, 6________ lying at a distance of
slightly more than four light-years.

Our knowledge of this region of space certainly does not reach as far as this, 7_________,
because the most distant Solar-System objects that we know about, the comets, seem to originate
at a distance of no more than 50,000 astronomical 8__________, or less than a third of the total
distance. 9_________ for the other Solar-System bodies known to us, they lie at distances of less
than a few hundred AU. Our study is therefore confined 10________ what is primarily the
central region of the Solar System.

Your answers
1. 2. 3. 4. 5.
6. 7. 8. 9. 10.

Part 3. Read the passage and choose the best option A, B, C, or D to answer the questions.
Write your answers in the corresponding numbered boxes. (15 points)
At 7pm on a dark, cold November evening, thousands of people are making their way
across a vast car park. They're not here to see a film, or the ballet, or even the circus. They are all
here for what is, bizarrely, a global phenomenon: they are here to see Holiday on Ice. Given that
most people don't seem to be acquainted with anyone who's ever been, the show's statistics are
extraordinary: nearly 300 million people have seen Holiday on Ice since it began in 1943; it is
the most popular live entertainment in the world.
But what does the production involve? And why are so many people prepared to spend their
lives travelling round Europe in caravans in order to appear in it? It can't be glamorous, and it's
undoubtedly hard work. The backstage atmosphere is an odd mix of gym class and workplace. A
curtained-off section at the back of the arena is laughably referred to as the girls' dressing room,
but is more accurately described as a corridor, with beige, cracked walls and cheap temporary
tables set up along the length of it. Each girl has a small area littered with pots of orange make-
up, tubes of mascara and long false eyelashes.
As a place to work, it must rank pretty low down the scale: the area round the ice-rink is
grey and mucky with rows of dirty blue and brown plastic seating and red carpet tiles. It's an
unimpressive picture, but the show itself is an unquestionably vast, polished global enterprise:
the lights come from a firm in Texas, the people who make the audio system are in California,
but Montreal supplies the smoke effects; former British Olympic skater Robin Cousins is now
creative director for the company and conducts a vast master class to make sure they're ready for
the show's next performance.
The next day, as the music blares out from the sound system, the cast start to go through their
routines under Cousins' direction. Cousins says, The aim is to make sure they're all still getting
to exactly the right place on the ice at the right time - largely because the banks of lights in the
ceiling are set to those places, and if the skaters are all half a metre out they'll be illuminating
empty ice. Our challenge,' he continues, 'is to produce something they can sell in a number of
countries at the same time. My theory is that you take those things that people want to see and
you give it to them, but not in the way they expect to see it. You try to twist it. And you have to
find music that is challenging to the skaters, because they have to do it every night.'
It may be a job which he took to pay the rent, but you can't doubt his enthusiasm. 'The only
place you'll see certain skating moves is an ice show,' he says, 'because you're not allowed to do
them in competition. It's not in the rules. So the ice show world has things to offer which the
competitive world just doesn't.' Cousins knows what he's talking about because he skated for the
show himself when he stopped competing - he was financially unable to retire. He learnt the
hard way that you can't put on an Olympic performance every night. 'I'd be thinking, these
people have paid their money, now do your stuff, and I suddenly thought, "I really can't cope.
I'm not enjoying it".' The solution, he realised, was to give 75 per cent every night, rather than
striving for the sort of twice-a-year excellence which won him medals.
To be honest, for those of us whose only experience of ice-skating is watching top-class
Olympic skaters, some of the movements can look a bit amateurish, but then, who are we to
judge? Equally, it's impossible not to be swept up in the whole thing; well, you'd have to try
pretty hard not to enjoy it.

1. According to paragraph 1 the writer is surprised to see that although Holiday on Ice is popular
______.
A. people often prefer other types of show
B. people prefer to see a film, the ballet, or the circus
C. most people consider it as a holiday
D. few people know someone who has seen it
2. From the phrase "it must rank pretty low down the scale" in paragraph 3 we can infer that
______.
A. Holiday on Ice has rather poor working condition
B. Holiday on Ice has a very dirty place to work
C. skaters do not enjoy working in this place
D. skaters do not earn much money from the job
3. Which of the following adjectives can be used to describe the backstage area?
A. glamorous B. relaxing C. messy D. old
4. It is mentioned in paragraph 3 that ______.
A. many companies are involved in the production
B. it is difficult to find suitable equipment
C. the show needs financial support
D. the show has been staged in many places
5. For Robin Cousins, the aim of the rehearsal is ______.
A. to keep in time with the music
B. to adjust the spotlights
C. to be acquainted with the stage
D. to position the skaters on the ice
6. Cousins' theory on how to produce shows for different audiences is that ______.
A. he adapts movements to suit everyone
B. he selects suitable music
C. he presents performances in an unexpected way
D. he varies the routines every night
7. It is suggested in paragraph 5 that skating in show ______.
A. enables skaters to visit a variety of places
B. is as competitive as other forms of skating
C. can be particularly well paid
D. allows skaters to try out original skating moves
8. The pronoun "them" in paragraph 5 refers to ______.
A. certain skating moves B. some famous skaters
C. some live performances D. certain ice shows
9. The phrase "the hard way" in paragraph 5 most likely means ______.
A. by working very hard
B. by having expectations of others
C. through personal experience
D. through doing things again and again
10. Which of the following is the writer's conclusion of Holiday on Ice?
A. It is more enjoyable than Holiday on Ice.
B. It is hard to know who really enjoys it.
C. It is difficult to dislike it.
D. It requires more skills than Olympic ice-skating.

Your answers
1. 2. 3. 4. 5.
6. . 8. 9. 10.

Part 4. Read the text and do the following tasks. (15 points)

ELECTRIC DREAMS
A. The days of the internal-combustion are numbered,and the fuel cell represents the future
of automotive transport, says PETER BREWER.A. Some of the world's greatest
inventions have been discovery by accident. One such accident led to the discovery of the
fuel cell and another led to its commercialisation. And in around 30 years, when most of
the energy analysts have predicted the oil wells will run dry, motorists will be thankful
for both these strange twists of fate. Why? Simply because without the fuel cell to replace
the combustion engine, private motoring as we all know it would be restricted to only
those who could afford the high price.
B. The exact date of the discovery of the fuel cell is not known, but historians agree it most
likely occurred around 1938 in the laboratories of British physicist Sir William
Grove,who one day disconnectedasimple electrolytic cell (in which hydrogen and oxygen
are produced when water contacts an electric current running throughaplatinum wire) and
reversed the flow of current. As author records in his book Powering the Future, Grove
realized that just as he could use electricity to split water into hydrogen and oxygen it
should be possible to generate electricity by combining these two gases.
C. The principle behind the fuel cell is simple. Hydrogen and oxygen, two of the most
common elements in the world,areavery explosive combination. But separate them
withasophisticated platinum coated barrier and an electro chemical reaction takes place,
where positively charged hydrogen ions react with oxygen and leave the hydrogen
electrons behind. It is this reaction, the excess electrons on one side of the barrier and the
deficit of electrons on the other that creates electrical energy.
D. The early development of the fuel cell was fraught with problems and high cost.But by
1954 US giant General Electric had producedaprototype that proved sufficiently effective
to interest NASA.The Gemini space programme proved the viability of the fuel cell to
provide electrical power. The spacecraft used six stacks of cells with three cells in each
stack. The electrical power output from each stack was quite modest-just one kilowatt
and asabyproduct, produced halfalitre of water for each kilowatt hour of operation. But
the Gemini Cells were very unstable and required constant monitoring.
E. At this time if anyone had suggested to Canadian Scientist Geoffrey Ballard that he
would becomeaworld leader in fuel cell technology, he would have laughed. Ballard's
scientific background was actually geophysics, but during the oil-crisis of 193,the US
government asked the Canadian to explore alternative forms of energy. Ballard threw
himself into the project enthusiastically but soon became disillusioned by the politics of
the programme. Energy systems takealong time to develop, Ballard said. The short-term
vision of politicians, who voted to fund such projects in the desire for quick results to
bolster their re-election chances,were frustrating for the scientists. However, since the US
government lacked the vision for the job, he decided to tackle it himself.
F. The big breakthrough on Ballard's fuel cell came by accident in the search for cheaper
materials. Up until late 1986, Ballard's team had worked with only one type of fuel cell
membrane manufactured by DuPont, but Dow Chemical had also developedasimilar
membrane, which had not been released for sale. Ballard's team tracked down an
experimental sample of the Dow material, put it intoafuel cell and set upastandard
test.Withinafew minutes the fuel cell was generating so much electricity on the test bench
that it had melted through the power-output cable.
G. Ballard immediately knew he hadasaleable product. The problem was: Should he aim his
fuel cell at small markets like military field generators, wheelchairs and golf carts,or try
to sell it asafull blown alternative to the combustion engine? "It was so needed and the
world was ready for it," Ballard said. "Los Angeles is dying; Vancouver is going to be
eaten alive by its own pollution very shortly.It seemed likeatime to go for broke. "Ballard
Power Systems first builtasmall bus to demonstrate the technology,and then an even
bigger bus.
H. Asaresultanumber of multinational motor manufacturers, such as General
Motors,Mitsubishi and Daimler-Benz all tested Ballard's cells. Finally, Daimler formed
an alliance with Ballard that has yielded some impressive prototypes, includingafully
driveable fuel cellpoweredA-class Mercedes-Benz compact car,known as Necar 4.
Daimler Chlysler, as the merged Daimler-Benz and Chlysler Corporation is now known,
says the fuel cell represents the future of automotive transport. "The significance of this
technological advancement (the fuel cell) is comparable to the impact the microchip had
on computer technology when it replaced the transistor, "said Dr Ferdinand Panik, the
head of Daimler Chlysler's fuel cell development team.

Questions 1-7 There are paragraphs numbered A-G in Reading Passage.

From the list below numbered i-x, choose a suitable heading for the paragraphs.
There are more headings than paragraphs, so you will not use all
the headings.
1. Paragraph A
List of headings
2. Paragraph B
i. A conflict of interests
3. Paragraph C
ii. Science is sometimes a question of luck
4. Paragraph D
iii. Using the fuel cell in different ways
5. Paragraph E
iv. How does it work?
6. Paragraph F
v. Deciding how to exploit the new product
7. Paragraph G
vi. Using the fuel cell to be the first in the space race

vii. A key stage in the development of fuel cell

viii. A first step on the road to a new source of energy

ix. Applying the new technology on a global scale

x. The first fuel cell is tested

Your answers:
1. 2. 3. 4. 5. 6.

Questions 8-10

Choose the most appropriate letter A, B, C or D

8. The fuel cell generates electricity because

A. hydrogen and oxygen can be used to create controlled explosions.

B. of the reaction which occurs when hydrogen and oxygen are separated.
C. hydrogen and oxygen are both gases.

D. hydrogen and oxygen both contain electrons.

9 The Gemini space programme demonstrated that

A.The fuel cell was too difficult to use in space programmes.

B. The fuel cell can only work with pure oxygen.

C.Generating a substantial amount of electricity requires many fuel cells.

D. The fuel cell could be used successfully.

10 The US government asked Ballard to carry out fuel cell research because

A. he was an expert in his field.

B. supplies of oil were running out.

C. they wanted to find new sources of energy.

D. te offered to work completely independently.

Your answers:
8. 9. 10.

PART 4. WRITING (50 POINTS)

Task 1 Rewrite the following sentences using the words given

61. Doctors believe that people who smoke regularly are more likely to die prematurely.

LINK

=> Doctors believe that a strong ________________________________________________

62. You are not a bad lawyer Martin, but I don't think it is a very suitable job for you. OUT
=> I just don’t________________________________________________________________

63. I don't have the foggiest idea whether he will be coming or not. WHATSOEVER

=> I have __________________________________________________________________

64. However hard he tries, his shock tactics are often not successful. MARK

=> No matter _______________________________________________________________

65. You can't just suddenly decide to go on a safari. You need to plan things very carefully.

MOMENT

=> Going on safari isn't a_____________________________________________________

Task 2. The charts below show the number of people in Europe who were affected by four
types of noise by day and by night in cities and rural areas in 2007.

Write a report describing the information shown below.

You should write about 150 words.


Task 3. Write an essay on the following topic:

More and more young people from wealthy countries are spending time in communities in
poorer countries doing unpaid work such as teaching or building houses.

Why is this?

Who benefits more from this, the communities or these young people?

You should write about 250 words.

TEST 10
SECTION I. LISTENING (50 points)

Part 1. For questions 1-5, listen to a telephone conversation about voluntary work. Fill the
gap with ONE WORD OR A NUMBER ONLY for each answer (10 points)

Volunteer Applicant details

Name of enquirer: Ben Oppermann

Age: (1) ________

Qualifications: Post graduated

Bachelor of Arts in Social Studies from University of Kent

Postgraduate Certificate in (2) _________ Education

Interested in placement lasting about (3) ________ years

Others skills and interests:

Very experience at (4) __________ work around place of living

Member of a (5) _________ protection group.

Your Answer:
1. 2. 3. 4. 5.

Part 2. For questions 6-10, listen to a talk about the smart grid and decide whether these
statements are True (T) or False (F). Write your answers in the corresponding numbered
boxes provided. (10 points)

6. Eleven deaths and the loss of six billion dollars were the approximate toll of the two-day
period in which people were deprived of power.

7. Households with off-grid power are obliged to electrically assist at times of smart grid’s
failure.

8. The smart grid increases the efficiency of the North American power grid by renovating the
current approach to energy production and dissemination.

9. The percentage of all energy which is depleted through the process of conducting amounts to
nineteen point five billion dollars.
10. Eventually, the smart grid power will prove beneficial to the environment, but it will be
rather prohibitive.

Your answers:
6. 7. 8. 9. 10.

Part 3: You will hear an interview with an archaeologist called Julian Ra questions 11-15,
choose the answer (A, B, C or D) which fit best what you hear. (10 points)

11. Julian attributes his interest in archaeology as a teenager to __________.

A. wish to please his father. B. his natural sense of curiosity.

C. a need to earn some spare cash. D. his dissatisfaction with life on a farm.

12. Julian feels that the public perception of archaeology __________.

A. fails to acknowledge its scientific value.

B. has been negatively influenced by fictional accounts.

C. underestimates the gradual nature of the research process.

D. has tended to concentrate on the physical hardships involved.

13. How does Julian feel about his current research post?

A. He regrets having relatively few opportunities to travel.

B. He wishes his colleagues would take it more seriously.

C. He admits that the problems can get him down.

D. He suggests that it is relatively cost effective.

14. What does Julian hope to show as a result of his current research?

A. population levels in England in different periods

B. the length of time certain villages have existed

C. how wider trends affected local communities

D. the range of ancient agricultural methods


15. Julian's project on humour in archaeology aims to __________.

A. celebrate an otherwise unrecorded aspect of archaeologists' lives.

B. compare archaeological findings with anecdotal evidence.

C. create a database of jokes connected with archaeology.

D. make archaeological reports more widely accessible.

Your answers:
11. 12. 13. 14. 15.

Part 4. For questions 16-25, listen to a piece of news about Brexit and complete the
sentences with NO MORE THAN FOUR WORDS AND/OR A NUMBER taken from the
recording for each answer in the space provided.

● A (16)__________________ was flexibly granted for the UK to leave The European Uninion
on January 31, 2020 at the latest.

● Boris Johnson claimed that he’d rather be (17) _______________ than ask for this delay.

● After the House of Commons passed a bill calling for an election on December 12th, the
country’s now (18) ___________________.

● A number of reasons proposed to explain for this leaving, however, (19) ____________ is the
one that underpins.

● Many Brit become (20) _______________ for having to follow given rules and regulations in
Brussels.

● The EU’s primary emphasis now is placed on (21)_______________ in the negotiations with
the UK.

● While Brexit is beneficial for UK visitors as well as some (22) ______________ businesses, it
causes UK citizens to struggle with the high cost of living.

● Some UK companies importing unprocessed goods are made to prepare themselves with (23)
_____________ during the time of Brexit.
● The (24) ______________ caused by Brexit will seclude the British from allies, alliances ,
and market.

● Despite these difficulties, the UK still has (25) _____________. The future now lies in the
hand of billions of people beyond its borders.

Your answers:
16. 17.
18. 19.
20. 21.
22. 23.
24. 25.

SECTION II. LEXICO AND GRAMMAR (40 POINTS)

Part 1. Choose one of the words marked A, B, C, or D which best completes each of the
following sentences. Write your answers in the corresponding numbered boxes. (20 points)

1. The explosion was of such __________ that it was heard five miles away.

A. intensivemess B. intensity C. intension D. intensification

2. Some mushroom contain a ________ poison and are dangerous to eat.

A. fatal B. mortal C. deadly D.


deathly

3. Some people think that students should be provided with monetary incentive as a
____________ for their extraordinary achievement.

A. quid pro quo B. faux pas C. cul-de-sac D. bona fide

2. That is a real danger which can threaten any country if we are not __________ heedful of its
existence.

A. acutely B. utterly C. blissfully D. reasonably

5. He has turned over a new leaf and sought reconciliation with his ____________ for what he
did.
A. blood and flesh B. rant and rave C. kith and kin D. belt and braces

6. Printed media are, to many people, ____________, superseded by new media which are
entirely digitized.

A. as old as the hills B. tough as old boots

C. as dull as ditchwater D. dead as the dodo

7. Anna is such a(n) ____________ that her idea of perfect activities for a holiday is doing
household chores at home.

A. anorak B. scrounger C. wet blanket D. wallflower

8. Upon finding out that he had been rejected, he cut up ____________.

A. rough B. raw C. loose D. harsh

9. The year 1969 was a ___________ in her life - she changed her career and remarried.

A. beefeater B. touchstone C. watershed D.


loudmouth

10. Dr Madan Kataria says we need more laughter in our lives to _____ stress and loneliness.

A. struggle B. combat C. threaten D. contest

11. The optician says you have to wear glasses, like it or _____ it.

A. jump B. loathe C. dislike D. lump

12. Jack says he appreciates ______ out last weekend.

A. Your help B. helping him C. your helping him D. to be help

13. The profit motives of American companies prevent them from making sure that everybody
has ________ priced internet access.

A. prettily B. nicely C. fairly D. reasonably

14. Her mother, in whom she __________, said she would support her unconditionally.

A. confided B. relied C. trusted D. shared


15. Last week, we were in the National Park on __________ safari so I could watch many wild
animals.

A. a B. an C. the D. x

16. She was sitting on the grass, out of breath. She ________. She couldn’t have run so long.

A. had been running B. was running C. had run D. ran

17. ________________, children can do a lot of marvelous things.

A. Once considerable assisstance and practical suggestion are given.

B. He/she is once given considerable assisstance and practical suggestion.

C. Once given considerable assisstance and practical suggestions.

D. Given considerable assisstance and practical suggestion once.

18. Round and round ____________.

A. the wheels of the engine went B. did the wheels of the engine go

C. went the wheels of the engine D. going the wheels of the engine

19. Many a delegate was in favour of his proposal that a special committee __________ to
investigate the incident.

A. was set up B. was setting up C. set up D. be set up

20. Owning and living in a freestanding house is still a goal of young adults, ____________
earlier generations.

A. as did B. as it was of C. like that of D. so have

Part 2. Give the correct form of the words in brackets. Write your answers in the
corresponding numbered boxes. (10 points)

1. Britain agreed to cut ___________ of nitrogen oxide from power stations. (EMIT)

2. The idea of our group being apportioned blame only is ____________ and outrageous.
(POSTURE)
3. The answer to our request was an _____________ "no". (EQUIVOCATE)

4. The team were basking in the ___________ of winning the cup. (GLOW)

5. He has an extreme fear of height, so it’s of no surprise that he’s suffering from _______.
(VERTICAL)

6. We are _________ although we have to be aware of the political impact our work can have in
some areas. (POLITIC)

7. Scented oils can act as a ____________ or antidepressant if inhaled or massaged. (STRESS)

8. She was a unique and ____________ interviewee who spoke with great confidence.
(VOLUME)

9. Stock markets have crashed in the biggest ____________ this century, with dollars suddenly
fast falling in value. (DIVE)

10. The police said his car had been weaving all over the road, so they pulled him over and gave
him a ____________ test. (SOBER)

SECTION III. READING (60 POINTS)

Part 1. For questions 1-10, read the following passages and decide which answer (A, B, C,
or D) best fits each gap. Write your answer in the numbered boxes. There is an example at
the beginning (0). (15 points)

Rainmaking

The (0) process of making rain is simpler than you might think. As warm, moisture-laden air
rises from the surface of the earth, it cools and some of the air (1) __________ into tiny droplets
that eventually become clouds. These droplets form around the microscopic particles such as
dust and smoke which are floating in the air.

The science of weather modification is now big (2) __________. Using radar and sensitive
equipment that (3) __________ atmospheric changes, weather modifiers fly above or below the
clouds and spray them with billions of minute particles known as seeding agents. These particles
either fall into clouds or are wafted into them from below by warm (4) __________. They then
'attract' tiny water droplets which (5) __________ around each one.

When enough droplets are attached, precipitation - the third and final (6) __________ in the
process which returns water to the earth's surface - occurs, and it rains. It may take as many as a
million droplets to form a single raindrop. If the clouds contain ice crystals, the results are
similar, but now snow will fonn instead of rain.

Current weather manipulation technology only allows scientists to 'encourage' a cloud that is (7)
__________ heavy to produce rain. Some more ambitious scientists (8) __________ a day when
they will be able to (9) __________ rain from blue skies, but this is still in the far (10)
__________ future.

1. A. condenses B. evaporates C. transforms D. gathers

2. A. commerce B. industry C. trade D. business

3. A. takes off B. picks up C. catches on D. puts across

4. A. flows B. draughts C. currents D. tides

5. A. gather B. fasten C. converge D. stick

6. A. division B. stage C. period D. level

7. A. sufficiently B. specifically C. splendidly D. satisfactorily

8. A. forecast B. prophesy C. guess D. foresee

9. A. manufacture B. supply C. conjure D. reveal

10. A. detached B. distant C. isolated D. remote

Your answers
1. 2. 3. 4. 5.
6. 7. 8. 9. 10.

Part 2. For questions 1-10 , read the text below and think of the word which best fits each
space. Use only ONE word in each space. Write your answers in the corresponding
numbered boxes. (15 points)

For centuries, unscrupulous food traders have had it (1) __________.They've passed (2)
__________ dyed tilapia as salmon, python meat as crocodile meat, and imitation crab as the
genuine article to shops and markets around the world. But the trade in fraudulent animal
products may finally have had (3) __________ day. Thanks to a technique known as 'DNA
barcoding', scientists believe they can (4) ___________ an end to food fraud (5) __________ and
for all. The principle behind DNA barcoding is as follows: all species can be identified by their
unique genetic material, Thus, by taking and analysing a sample of DNA from a food product,
scientists can unequivocally identify the species it came from. Countries around the word are
now using DNA barcoding to crack (6) __________ on food fraud. One market where authorities
believe DNA barcoding will prove particularly valuable is the $250-billion-a year global seafood
industry. In the USA (7) __________, as much as 25% of all fish imports are estimated to be
fraudulently labelled. (8)__________certain species, such as red snapper, fraud runs as high as
75%. There is big incentive for fish suppliers to cheat. First, only 2% of fish in the US market is
inspected so the chances are good they will get (9) __________ with the deception. Second,
over-fishing, coupled with a growing consumer demand, has created seafood shortages.
Typically, fraud rests in the area of cheaper seafood being labelled as more expensive species.
Importers will (10) __________ to great lengths to cut and dye their fillets to look like the fish
they're trying to imitate. To combat seafood fraud, the US Food and Drug Administration
recently announced an increase in its use of DNA testing in inspection of seafood manufacturers
and restaurants

Your answers
1. 2. 3. 4. 5.
6. 7. 8. 9. 10.

Part 3. For questions 1-10, read the passage and choose the best option A, B, C, or D to
answer the questions. Write your answers in the corresponding numbered boxes. (15
points)
Cholera, a highly infectious disease, has resulted in millions of deaths time after time
over centuries. It is caused by the bacterium Vibrio cholera, first isolated by Robert Koch in
1883.
The organism enters the body through the digestive tract when contaminated food or
water is ingested. The bacteria multiply in the digestive tract and establish infection. As they die,
they release a potent toxin that leads to severe diarrhea and vomiting. This results in extreme
dehydration, muscle cramps, kidney failure, collapse and sometimes death. If the disease is
treated promptly, death is less likely.
In many countries, a common source of the organism is raw or poorly cooked seafood,
taken from the contaminated waters. The disease is especially prevalent after a natural disaster
or other destruction that results in a lack of fresh water. Sewer systems fail and waste travels into
rivers or streams; piped water is not available so people must take their drinking and cooking
water from rivers or streams. Because people frequently develop communities along waterways,
the disease can be spread easily from one community to the next community down streams,
resulting in serious epidemics.
1. The word infectious in the first sentence is closest in meaning to ______.
A. communicable B. severe C. isolated D. common
2. According to the passage, cholera is caused by ______.
A. a virus B. a bacterium C. kidney failure D. dehydration
3. All of the following are probable causes of infection EXCEPT ______.
A. eating food cooked with contaminated water B. eating undercooked seafood
C. eating overcooked pork D. eating raw oysters
4. According to the passage, what is a symptom of the infection?
A. Release of a toxin by the bacteria B. Regurgitation
C. Overeating D. Epidemics
5. Which of the following would be an appropriate title for this passage?
A. Dysentery and its effects
B. Water Purification Systems and Their Importance
C. Results of Wars and Natural Disasters
D. The Causes and Effects of Cholera
6. The word prevalent in the third paragraph is closest in meaning to ______.
A. dangerous B. commonplace C. unusual D. organized
7. The word lack in the third paragraph is closest in meaning to ______.
A. contamination B. multitude C. shortage D. well
8. According to the passage, cholera ______.
A. is easily passed from one person to another
B. is not a real threat
C. is no more dangerous than the common cold
D. cannot be passed from one to another by casual contact
9. What can you infer from the passage?
A. Careful cooking and hygiene practices can reduce the chance of getting the disease
B. Water mixed with other substances will not pass the disease
C. The respiratory system is the most common area of entrance
D. Kidney disease is the most common cause of the disease
10. The word epidemics at the end of the passage is closest in meaning to ______.
A. studies B. illness C. bacteria D. plagues
Your answers:
1. 2. 3. 4. 5.
6. 7. 8. 9. 10.

Question 4: Read the following passage and do the tasks below


Putting the brakes on climate change: Are hydrogen cars the answer?
A
It is tempting to think that the conservation of coral reefs and rainforests is a separate issue from
traffic and air pollution. But it is not. Scientists are now confident that rapid changes in the
Earth’s climate are already disrupting and altering many wildlife habitats. Pollution from
vehicles is a big part of the problem.
B
The United Nation’s Climate Change Panel has estimated that the global average temperature
rise expected by the year 2100 could be as much as 6°C, causing forest fires and dieback on land
and coral bleaching in the ocean. Few species, if any, will be immune from the changes in
temperature, rainfall and sea levels. The panel believes that if such catastrophic temperature rises
are to be avoided, the quantity of greenhouse gases, especially carbon dioxide, being released
into the atmosphere must be reduced. That will depend on slowing the rate of deforestation and,
more crucially, finding alternatives to coal, oil and gas as our principal energy sources.
C
Technologies do exist to reduce or eliminate carbon dioxide as a waste product of our energy
consumption. Wind power and solar power are both spreading fast, but what are we doing about
traffic? Electric cars are one possible option, but their range and the time it takes to charge their
batteries pose serious limitations. However, the technology that shows the most potential to
make cars climate-friendly is fuel-cell technology. This was actually invented in the late
nineteenth century, but because the world’s motor industry put its effort into developing the
combustion engine, it was never refined for mass production. One of the first prototype fuel-cell-
powered vehicles have been built by the Ford Motor Company. It is like a conventional car, only
with better acceleration and a smoother ride. Ford engineers expect to be able to produce a
virtually silent vehicle in the future.
D
So what’s the process involved – and is there a catch? Hydrogen goes into the fuel tank,
producing electricity. The only emission from the exhaust pipe is water. The fuel-cell is, in some
ways similar to a battery, but unlike a battery, it does not run down. As long as hydrogen and
oxygen are supplied to the cell, it will keep on generating electricity. Some cells work off
methane and a few use liquid fuels such as methanol, but fuel-ceils using hydrogen probably
have the most potential. Furthermore, they need not be limited to transport. Fuel-cells can be
made in a huge range of size, small enough for portable computers or large enough for power
stations. They have no moving parts and therefore need no oil. They just need a supply of
hydrogen. The big question, then, is where to get it from.
E
One source of hydrogen is water. But to exploit the abundant resource, electricity is needed, and
if the electricity is produced by a coal-fired power station or other fossil fuel, then the overall
carbon reduction benefit of the fuel-cell disappears. Renewable sources, such as wind and solar
power, do not produce enough energy for it to be economically viable to use them in the
‘manufacture’ of hydrogen as a transport fuel. Another source of hydrogen is, however, available
and could provide a supply pending the development of more efficient and cheaper renewable
energy technologies. By splitting natural gas (methane) into its constituent parts, hydrogen and
carbon dioxide are produced. One way round the problem of what to do with the carbon dioxide
could be to store it back below ground – so-called geological sequestration. Oil companies, such
as Norway’s Statoil, are experimenting with storing carbon dioxide below ground in oil and gas
wells.
F
With freak weather conditions, arguably caused by global warming, frequently in the headlines,
the urgent need to get fuel-cell vehicles will be available in most showrooms. Even now, fuel-
cell buses are operating in the US, while in Germany a courier company is planning to take
delivery of fuel-cell-powered vans in the near future. The fact that centrally-run fleets of buses
and vans are the first fuel-cell vehicles identifies another challenge – fuel distribution. The
refueling facilities necessary to top up hydrogen-powered vehicles are available only in a very
few places at present. Public transport and delivery firms are logical places to start since their
vehicles are operated from central depots.
G
Fuel-cell technology is being developed right across the automotive industry. This technology
could have a major impact in slowing down climate change, but further investment is needed if
the industry – and the world’s wildlife – is to have a long-term future.
Reading Passage has seven paragraphs, A-G.
Choose the correct heading for paragraphs A-F from the list of headings below.
Write the correct number, i-ix, in boxes 1-6 on your answer sheet.
List of Headings
i Action already taken by the United Nations
ii Marketing the hydrogen car
iii Making the new technology available worldwide
iv Some negative predictions from one group of experts
v How the new vehicle technology works
vi The history of fuel-cell technology
vii A holistic view of climatic change
viii Locating the essential ingredient
ix Sustaining car manufacture
1. Paragraph A - _____________
2. Paragraph B - _____________
3. Paragraph C - _____________
4. Paragraph D - _____________
5. Paragraph E - _____________

6. Paragraph F-______________
For statements 7-10 on your answer sheet, write
TRUE if the statement agrees with the information
FALSE if the statement contradicts the information
NOT GIVEN if there is no information on this
7. Using electricity produced by burning fossil fuels to access sources of hydrogen may increase
the positive effect of the fuel-cell.
8. The oil company Statoil in Norway owns gas wells in other parts of the world.
9. Public transport is leading the way in the application of fuel-cell technology.
10. More funding is necessary to ensure the success of the fuel-cell vehicle industry.
Your answers:
1. 2. 3. 4. 5.
6. 7. 8. 9. 10.

SECTION IV: WRITING (50 POINTS)

Part 1: Rewrite the following sentences using the words given

1. When it comes to political view, his comments are sometimes inappropriate. PALE

=> As ______________________________________________________________________

2. Looking back, we could have managed things in a more organized way. SHIP

=> With____________________________________________________________________

3. I’ve become extremely good at driving cars over the last few weeks. FINE

=> I’ve got __________________________________________________________________

4. What you was saying has nothing in common with the facts. DOES

=> What____________________________________________________________________

5. I really think you should be more assertive about your rights. UP

=> It’s about ____________________________________________________________more.

Part 2 .Graph writing (20 points)

The tables presents the distribution of world population in 1950, 2000 and its estimated
figure in 2050

Summaries the information by selecting and reporting the main features, and make
comparisons where relevant.

Write at least 150 words.


Part 3: Essay writing (30 points)

Write an essay of 250 words about the following topic.

Some people think young people are not suitable for important positions in the government,
while other people think it is a good idea for young people to take on these positions.

Discuss both views and give your own opinion.

TEST 11
A. LISTENING (50 points)
Part 1. Complete the notes below. For questions 1-5, write THREE WORD AND/OR A
NUMBER for each answer. (10 points)
Notes – Clark’s Bicycle Hire
Example:
Type: …….. touring …. bike
Rental: £50 a week, or 1 £ ………………………… a day
Late return fee: £ 1.25 per extra hour
Deposite: 2 £ ………………………… returnable
Accessories: £5 for 3 …………………………: pannier or handlebar type
Free: pump, 4 …………………………, strong lock
Insurance: included, but must pay first 5 £ ………………………… of claim
Part 2. You will hear Tristram Stuart talking about Food waste. For questions 1-5, decide
whether the statement is TRUE (T) or FALSE (F). (10 points)
1. He realised that bins full of food were being sent to the supermarkets. True False
2. Farmers in Korea waste thousands of tonnes of good food every year. True False
3. According to Tristram Stuart, society needs to change. It needs to believe True False
that food is far too valuable to waste.
4. Tristram Stuart also says the best way to spread the global food waste True False
revolution is to have a massive food festival.
5. Tristram Stuart has been a campaigner of food waste for twenty-two years. True False
Your answers:
1. 2. 3. 4. 5.
Part 3. You will hear an interview in which two journalists called Jenny Langdon and Peter
Sharples are talking about their work. For questions 6-10, choose the answer (A, B, C or D)
which fits best according to what you hear. (10 points)
1. What does Jenny say about the story which made her name?
A. She’d been on the lookout for just such a lucky break.
B. She resented colleagues trying to take the credit for it.
C. She wasn’t actually responsible for the finished article.
D. She asked for a more prestigious job on the strength of it.
2. What does Jenny suggest about the editor she worked for on her first national daily
newspaper?
A. He respected her for standing up to him.
B. He tended to blame her for things unfairly.
C. He wasn’t as unreasonable as everyone says.
D. He taught her the value of constructive criticism.
3. Peter thinks he got a job on Carp Magazine thanks to _____
A. his academic achievements at college.
B. his practical knowledge of everyday journalism.
C. his familiarity with the interests of its main target audience.
D. his understanding of how best to present himself at interview.
4. Peter and Jenny agree that courses in journalism _____
A. need to be supplemented by first-hand experience.
B. are attractive because they lead to paid employment.
C. are of little value compared to working on a student newspaper.
D. provide an opportunity for writers to address contentious issues.
5. When asked about their novels, Peter and Jenny reveal _____
A. an ambition to gain recognition for their craft.
B. a desire to develop careers outside journalism.
C. a need to prove how versatile they are as writers.
D. a wish to keep their journalism fresh and appealing.
Your answers:
1. 2. 3. 4. 5.
Part 4. Listen to a recording about an invention and complete each sentence with NO MORE
THAN THREE WORDS. (20 points)
1. The tube looks like a _______________.
2. The device’s _______________ is to catch the ocean’s rubbish.
3. _______________ make lots of plastic rubbish drift together.
4. For a school project, Boyan designed a system of _______________.
5. Plastic takes a very long time to _______________.
6. The plastic in our oceans is killing our _______________.
7. Boyan hopes to _______________ the problem of plastic pollution.
8. The aim of the first unit is to trap some of _______________ pieces of plastic swirling around
the Garbage Patch.
9. Boats will _______________ the collected plastic every few months.
10. Boyan says the system will get some more ________________ in the following months.
Your answers:
1. 6.
2. 7.
3. 8.
4. 9.
5. 10.

SECTION B. LEXICO- GRAMMAR (40 points)


Part 1. Choose the best option A, B, C, or D to complete the following sentences and write
your answers in the corresponding numbered boxes. (20 points)
1. "Who is that strange woman ___________ you from across the room? She looks quite
aggressive."
A. gazing B. staring C. eyeing D. peering
2. The company's financial ____________ was so serious that investors were warned that they
could lose all their money.
A. plight B. dead-end C. impasse D. breakdown
3. If the manager says anything about the missed deadline, just ___________ your tongue, say
nothing and we can discuss it later.
A. eat B. swallow C. bite D. suck
4. Despite the scandal, the leader emerged with his reputation ___________.
A. untarnished B. unpolluted C. unimpaired D. unfettered
5. The extensive winds that have been ___________ Florida will move away as the weekend
draws to a close.
A. battering B. punching C. cracking D. thumping
6. Young children are ___________ often willing to behave stupidly to impress their friends.
A. all so B. all very C. all too D. all but
7. Apparently, the sailor spent three days on the beach before anyone found him and sent for
help. It doesn't surprise me: that is a very deserted ___________ of coastline.
A. zone B. stretch C. corridor D. swathe
8. I retired three years ago and didn't know what to do with myself. Getting this dog has given
me a new ___________ of life.
A. burst B. loan C. lease D. extension
9. The winning team were roundly criticised by the local media for the way in which they had
___________ over the losing team. It was considered very unsporting.
A. gloated B. relished C. showed up D. dominated
10. Many celebrities complain about the media attention but I think many of them thrive on
being in the ___________.
A. torch-light B. headlight C. floodlight D. limelight
11. If you are looking for Sheila, she is over in the corner of the library with her head
___________ in her books.
A. covered B. entombed C. buried D. drowned
12. Hubert ___________ remembered locking the door and couldn't understand how it was now
standing wide open.
A. distinctly B. sharply C. totally D. utterly
13. And also among our guests tonight, we are fortunate enough to have the ___________
environmentalist Kathy Wong.
A. notorious B. eminent C. prestigious D. monumental
14. Here's something that will give you ___________ for thought. Every year, there are 90
million more people on the Earth than the year before!
A. stop B. break C. reason D. pause
15. Out upon the cloud covered hills, we only caught the occasional glimpse of the town lights
___________ far below in the valley.
A. sparkling B. glimmering C. flashing D. beaming
16. I felt let down by a very unsupportive boardroom, ___________ by the very chairman
himself, Wilson Gray. I had no choice but to resign.
A. at most B. not least C. notwithstanding D. henceforth
17. The police were called out in the early hours of the morning when there was a ___________
involving a few teenagers.
A. disturbance B. riot C. outcry D. campaign
18. The spacecraft ________ into space and then spent three days getting to the Moon.
A. hurled B. threw C. blasted D. orbited
19. When I say I want you to be here at six o'clock, I mean six o'clock ____________. You
cannot be late under any circumstances!
A. on the edge B. at the moment C. on the dot D. in detail
20. I will not be made the ____________ for this disaster. I acted on the advice of people above
me in this company and if I go, I am taking them with me!
A. loser B. sacrifice C. scapegoat D. target
Your answers:
1. 2. 3. 4. 5.
6. 7. 8. 9. 10.
11. 12. 13. 14. 15.
16. 17. 18. 19. 20.
Part 2. Write the correct form of the words given in the brackets. Write your answers in the
spaces provided below. (10 points)
1. His witty retorts ___________ the otherwise dull atmosphere. LIVE
2. We agreed that the problem was rather ___________ . TICKLE
3. The government promised to ___________ the public transport next year. SUBSIDY
4. She is not exactly ___________, but she is not very good at counting. NUMERATE
5. At last our plans came to ___________ after months of hard work. FRUIT
6. I guess the annual ___________ of your car is high if you travel so much. MILE
7. His ___________ concern for the poor is disgusting – he’s just a hypocrite. OSTENTATION
8. That country has an ___________ climate which allows for lush vegetation. EQUATOR
9. Nobody can be convicted of a crime on ___________ evidence alone. CIRCUMSTANCE
10. Our children are at a ___________ age we have to cope with, REBEL
Your answers:
1. 6.
2. 7.
3. 8.
4. 9.
5. 10.
C. READING (50 points)
Part 1. Read the following passage and decide which answer (A, B, C, or D) best fits each gap.
Write your answers in corresponding numbered boxes. (15 points)
Peoples' personalities vary considerably from one another, as there are no two alike. Our
ingrained characteristics which (1) ________ the patterns of our behaviour, our reactions and
temperaments are unparalleled on (2) ________ of the diversified processes that (3) ________
our personality in the earliest stages of human development.
Some (4) ________ of character may to some extent be hereditary simulating the attributes that
(5) ________ our parents. Others may (6) ________ from the conditions experienced during
pregnancy and infancy in this way reflecting the parents' approach towards (7) ________ their
offspring.
Consequently, the environmental factor plays a crucial role in strengthening or eliminating
certain behavioural systems making an individual more prone to
(8) ________ to the patterns that deserve a prize.
Undoubtedly, human personality (9) ________ the most profound and irreversible formation
during the first period of its development, yet, certain characteristics may still be (10) ______ to
considerable changes conditioned by different circumstances and situations.
1. A. denote B. resolve C. inflict D. determine
2. A. account B. means C. token D. event
3. A. mould B. design C. conceive D. fabricate
4. A. factors B. traits C. items D. breeds
5. A. pertain B. recognize C. associate D. identify
6. A. stem B. relate C. rise D. formulate
7. A. breeding B. rearing C. growing D. yielding
8. A. comfort B. pledge C. acquiesce D. obey
9. A. underacts B. undertakes C. undergoes D. underlies
10. practicable B. feasible C. subject D. potential
Your answers:
1. 2. 3. 4. 5.
6. 7. 8. 9. 10.
Part 2. Read the following text and fill in the blank with ONE suitable word. Write your
answers in corresponding numbered boxes. (15 points)
Football in the United Kingdom
Both the varieties of football that originally hailed (1) ___________ Britain, the Rugby and
Association games, have probably (2) ___________ been as popular as they are now. The
association game, or soccer (3) ___________ it has become generally known, is now almost
globally popular and judging by the last few World Cups, (4) ___________ long a team from
Asia, Africa or North America will go on to win the trophy. Rugby’s progress has been (5)
___________ dramatic but the game is becoming increasingly international and is surely
destined to continue to expand.
Of course, periodically it is (6) ___________ that both varieties of football will be afflicted
(7)___________ problems stemming from a downturn in the world economy and difficulties
generated by financial mismanagement. Collectively, (8)___________, their future appears
bright. Peculiarly enough, (9) __________ this very little is known about the origins of modern
football and many questions relating to (10) ___________ growth and development remain
unanswered. Football is a very old game, especially in Britain.
Your answers:
1. 2. 3. 4. 5.
6. 7. 8. 9. 10.
Part 3. Read the following passage and circle the best answer to each of the following
questions. Write your answers in corresponding numbered boxes. (15 points)
Tattoos in Tribal and Ancient Cultures
In today’s culture, tattoos are very popular as a form of body decoration. As seen in their
prevalence among rebellious teenagers, social outsiders, dangerous outlaws, and many other
people, tattoos are viewed as luxurious fashion statements that possess a forbidden attraction
– mainly as long as they remain unseen. However, some people get tattoos in order to identify
themselves with certain groups, such as military units, fraternities, or even criminal
organizations, like the Japanese yakuza. Tattoos have been used in this manner throughout
history by many different cultures in the world. In fact, they have always been an important
functional and symbolic feature in most tribal and ancient cultures, whether they were used for
identification, worship, therapy, decoration, or in other rituals.
The earliest written records describing tattoo usage date back to Ancient Egypt, from
where the practice spread to Crete, Greece, Persia, and Arabia. However, tribal cultures
throughout northern Europe, the western hemisphere, and east Asia had been applying tattoos
for several thousands of years before this, with people in Japan reportedly creating tattoos ten
thousand years ago. One of the best pieces of evidence of this usage came from the excavation
of Otzi the Ice Man, the oldest naturally preserved body discovered in Europe. Dating back to
3300 BC, this mummy had fifty- seven tattoos on its body, mostly located on the inside of the
left knee, ankles, and legs. In China and Russia, naturally mummified bodies that are just as old
have been found to bear tattoos in similar spots. Based on the location of these tattoos, some
researchers speculate that they were part of a therapeutic treatment for bones.
Ancient and tribal cultures used tattoos primarily for identification, though. A person’s
tattoo would indicate their tribe, family, or social status. For instance, the Norse, the Danes, the
Saxons, and other Germanic peoples used tattoos to identify their particular clans. The Maori in
New Zealand used tattoos to indicate their social ranking. In many southeast Asian tribes, as
well as American tribes like the Inuit, women used tattoos to indicate that they were eligible for
marriage and already married. Tattoos were also very common among warriors who displayed
bravery and ferocity in combat. Chickasaw warriors distinguished themselves with tattoos, and
the Dayak tribes used them to boast of the people they killed in battle. In later military usage,
ancient Greek spies had tattoos that related their military ranks to each other. Originally,
Roman soldiers didn’t have tattoos. However, after witnessing the fierceness of British tribes
they fought, Roman soldiers began to get tattoos in order to emulate their enemy’s ferocity.
Tribes also used tattoos for ritualistic purposes. South American tribes like the Mayans,
Incas, and Aztecs regularly employed tattoos as part of their religious ceremonies. Tattoos were
also used in many burial and mourning ceremonies. Hawaiians would tattoo their tongues three
times to mourn the deaths of loved ones, and Mohave Indians would tattoo faces of the dead
upon burial. Many tribes applied tattoos to young men and women as a fundamental rite of
passage that would inaugurate their adulthood. In Tahiti, a girl’s posterior would be tattooed
completely black to show that she had reached puberty. In Samoa, an adolescent male was
expected to endure the long, agonizing process of a full body tattoo in order to be accepted as a
man. Many tribes also used tattoos as magical symbols and charms. In Borneo, an eye tattooed
on the hand represented a spiritual guide. Burmese tribes used tattoos as protection against
danger and pain; Egyptian priestesses may have used such markings as mystical therapies
against pain during pregnancy. For these groups of people, ritualistic tattoos expressed a link to
the spiritual world that was the foundation of most of these cultures.
In more modern cultures, where mysticism has been mostly rejected, tattoos have found
somewhat different uses. Officially, many societies have applied tattoos to those who were
considered outcast; convicts in medieval Japan and prisoners in Nazi concentration camps are
two notable examples of this. Through such uses, tattoos began to acquire negative
associations. Many modern societies harbored religious objections and ethnic prejudices that
led them to interdict the art of tattoos in colonies where it had existed for thousands of years.
Even though tattoos eventually became legal and permissible in many modern societies, they
still retain a somewhat dubious distinction, and even today they are generally associated with
outcasts, criminals, and impropriety.
(TOEFL iBT Insider Reading)

1. In paragraph 1, the author mentions the modern use of tattoos as a form of identification in
order to
A. introduce their origins in older cultures
B. explain their associations with criminals
C. provide technical details on tattoos
D. condemn them as fashion statements
2. According to paragraph 2, some researchers believe that the tattoos found on mummies were
used to
A. mark them as slaves B. treat bone ailments
C. begin their adulthood D. assist them in worship
3. Based on the information in paragraph 2, what can be inferred about origins of tattoos?
A. All ancient cultures copied the practice of Egypt.
B. Otzi the Ice Man is the first to have ancient tattoos.
C. It took thousands of years for tattoos to be used globally.
D. Tattoos developed independently in different cultures.
4. The word them in the passage refers to
A. warriors B. themselves C. tattoos D. tribes
5. According to paragraph 3, why did the Roman soldiers begin to get tattoos?
A. They were inspired by ferocity of their tattooed enemies.
B. They wanted to boost of enemies they killed.
C. They needed to identify their military ranks.
D. They used tattoos to treat their battle wounds.
6. In paragraph 3, all of the following are listed as things identifies by tattoos in tribal cultures
EXCEPT
A. marital status B. tribal affiliation
C. social rank D. criminal status
7. According to paragraph 4, when did a Samoan male get a full body tattoo?
A. When he committed a crime B. When he needed to heal pain
C. When he reached manhood D. When he was in mourning
8. Based on the information in paragraph 4, what can be identifies about tattoos in ancient and
tribal rituals?
A. They were a luxury only a few priests had.
B. They were an essential part pf most rituals.
C. They were common decorations, not serious.
D. They were only used in the rarest ceremonies.
9. The word interdict in the passage is closest in meaning to
A. legalize B. study C. prohibit D. encourage
10. Based on the information in the passage, what can be inferred about the use of tattoos in
cultures?
A. Tattoos have been critical in identifying criminals.
B. Tattoos have been useful in stopping outlaw gangs.
C. Tattoos represent a sophisticated but rare art form.
D. Tattoos have lost their ritualistic significance.

Your answers:
1. 2. 3. 4. 5.
6. 7. 8. 9. 10.
Part 4: Read the passage answer the questions. (15 points)
Inside the mind of the consumer
A Marketing people are no longer prepared to take your word for it that you favour one
product over another. They want to scan your brain to see which one you really prefer. Using the
tools of neuroscientists, such as electroencephalogram (EEG) mapping and functional magnetic-
resonance imaging (FMRI), they are trying to learn more about the mental processes behind
purchasing decisions. The resulting fusion of neuroscience and marketing is inevitably, being
called ‘neuromarketing’.
B The first person to apply brain-imaging technology in this way was Gerry Zaltman of
Harvard University, in the late 1990s. The idea remained in obscurity until 2001, when
BrightHouse, a marketing consultancy based in Atlanta, Georgia, set up a dedicated
neuromarketing arm, BrightHouse Neurostrategies Group. (Bright House lists Coca-Cola, Delta
Airlines and Home Depot among its clients.) But the company’s name may itself simply be an
example of clever marketing. BrightHouse does not scan people while showing them specific
products or campaign ideas, but bases its work on the results of more general FMRI-based
research into consumer preferences and decision-making carried out at Emory University in
Atlanta.
C Can brain scanning really be applied to marketing? The basic principle is not that
different from focus groups and other traditional forms of market research. A volunteer lies in an
FMRI machine and is shown images or video clips. In place of an interview or questionnaire, the
subject’s response is evaluated by monitoring brain activity. FMRI provides real-time images of
brain activity, in which different areas “light up” depending on the level of blood flow. This
provides clues to the subject’s subconscious thought patterns. Neuroscientists know, for
example, that the sense of self is associated with an area of the brain known as the medial
prefrontal cortex. A flow of blood to that area while the subject is looking at a particular logo
suggests that he or she identifies with that brand.
D At first, it seemed that only companies in Europe were prepared to admit that they used
neuromarketing. Two carmakers, DaimlerChrysler in Germany and Ford’s European arm, ran
pilot studies in 2003. But more recently, American companies have become more open about
their use of neuromarketing. Lieberman Research Worldwide, a marketing firm based in Los
Angeles, is collaborating with the California Institute of
Technology (Caltech) to enable movie studios to market-test film trailers. More controversially,
the New York Times recently reported that a political consultancy, FKF Research, has been
studying the effectiveness of campaign commercials using neuromarketing techniques.
E Whether all this is any more than a modern-day version of phrenology, the Victorian
obsession with linking lumps and bumps in the skull to personality traits, is unclear. There have
been no large-scale studies, so scans of a handful of subjects may not be a reliable guide to
consumer behaviour in general. Of course, focus groups and surveys are flawed too: strong
personalities can steer the outcomes of focus groups, and people do not always tell opinion
pollsters the truth. And even honest people cannot always explain their preferences.
F That is perhaps where neuromarketing has the most potential. When asked about cola
drinks, most people claim to have a favourite brand, but cannot say why they prefer that brand’s
taste. An unpublished study of attitudes towards two well-known cola drinks. Brand A and Brand
B carried out last year in a college of medicine in the US found that most subjects preferred
Brand B in a blind tasting FMRI scanning showed that
drinking Brand B lit up a region called the ventral putamen, which is one of the brains ‘reward
centres,’ far more brightly than Brand A. But when told which drink was which, most subjects
said they preferred Brand A, which suggests that its stronger brand outweighs the more pleasant
taste of the other drink.
G “People form many unconscious attitudes that are obviously beyond traditional methods
that utilise introspection,” says Steven Quartz, a neuroscientist at Caltech who is collaborating
with Lieberman Research. With over $100 billion spent each year on marketing in America
alone, any firm that can more accurately analyse how customers respond to products, brands and
advertising could make a fortune.
H Consumer advocates are wary. Gary Ruskin of Commercial Alert, a lobby group, thinks
existing marketing techniques are powerful enough. “Already, marketing is deeply implicated in
many serious pathologies,” he says. “That is especially true of children, who are suffering from
an epidemic of marketing- related diseases, including obesity and type-2 diabetes.
Neuromarketing is a tool to amplify these trends.”
I Dr Quartz counters that neuromarketing techniques could equally be used for benign
purposes. “There are ways to utilise these technologies to create more responsible advertising,”
he says. Brain-scanning could, for example, be used to determine when people are capable of
making free choices, to ensure that advertising falls within those bounds.
J Another worry is that brain-scanning is an invasion of privacy and that information on the
preferences of specific individuals will be misused. But neuromarketing studies rely on small
numbers of volunteer subjects, so that seems implausible. Critics also object to the use of
medical equipment for frivolous rather than medical purposes. But as Tim Ambler, a
neuromarketing researcher at the London Business School, says: ‘A tool is a
tool, and if the owner of the tool gets a decent rent for hiring it out, then that subsidises the cost
of the equipment, and everybody wins.’ Perhaps more brain-scanning will some day explain why
some people like the idea of neuromarketing, but others do not.
Questions 1-6
The reading passage has ten paragraphs A-J. Choose the correct heading for Paragraphs B-G
from the list of headings below.
1. Paragraph B
2. Paragraph C
3. Paragraph D
4. Paragraph E
5. Paragraph F
6. Paragraph G
List of Headings
i. A description of the procedure
ii. An international research project
iii. An experiment to investigate consumer responses
iv. Marketing an alternative name
v. A misleading name
vi. A potentially profitable line of research
vii. Medical dangers of the technique
viii. Drawbacks to marketing tools
ix. Broadening applications
x. What is neuromarketing?
Questions 7-10
Complete the summary below using words from the passage. Choose ONE WORD ONLY from
the passage for each answer.
Neuromarketing can provide valuable information on attitudes to particular (7)…………………
It may be more reliable than surveys, where people can be (8)……………………..or focus
groups, where they may be influenced by others. It also allows researchers to identify the
subject’s (9)…………………thought patterns. However, some people are concerned that it could
lead to problems such as an increase in disease among (10)………………..
Your answers:
1. 2. 3. 4. 5.
6. 7. 8. 9. 10.

D. WRITING (50 points)


Part 1. Rewrite the following sentences using the words given
1. Please do those photocopies whenever you have the time. HAPPEN
=> Please do _______________________________________________________________
2. He didn’t want to get into a position where he might lose all his money. EXPOSE
=> He __________________________________________________________all his money.
3. Sadists seem content to let people die in pain rather than make any effort to help them.
LIFT
=> Sadists give the impression that _____________________________________________
4. Had it not been for John, she couldn’t have got the job. AGENCY
=> Only ____________________________________________________________________
5. The way he analyzed the situation was far too complex for me to grasp. HEAD
=> His analysis _______________________________________________________________
Part 2. Describe the chart provided. (20 points)
The table below shows the average band scores for students from different language groups
taking the IELTS General Test in 2010.
Summarise the information by selecting and reporting the main features, and make comparisons
where relevant.
Write at least 150 words.
Listening Reading Writing Speaking Overall

German 6.8 6.3 6.6 6.9 6.7

French 6.3 6.1 6.5 6.6 6.5

Indonesian 6.3 6.1 6.1 6.7 6.3

Malay 6.2 6.4 6.0 6.6 6.4

Part 3. Essay writing. (30 points)


Some people believe that allowing children to make their own choices on everyday matters (such
as food, clothes and entertainment) is likely to result in a society of individuals who only think
about their own wishes. Other people believe that it is important for children to make decisions
about matters that affect them.
Discuss both these views and give your own opinion.

TEST 12
A. LISTENING (50 points)
Part 1. Listen and fill in each blank with NO MORE THAN THREE WORDS OR/AND A
NUMBER. (Section 1 IELTS) (10 points)
Example
Type of event: Dragon Boat Race
Day and date: 1 _________
Place: Brighton 2 _________
Registration time: 3 _________
Sponsorship
• Aim to raise over 4 _________ pounds as a team and get a free t-shirt
• Free prize draw for trip to 5 _________

Your answers:
1. 2. 3. 4. 5.

Part 2. (CAE Listening practice Tests - Test 1 – Part 3)(10 points)


You will hear part of an interview with the astronaut Charles Duke, who is talking
about his trip to the moon. Choose the answer (А, В or C) which fits best according to what
you hear.
1. How did Charles feel about space travel as a boy?
A. He thought it was unlikely to happen
B. He regarded it as more than science fiction
C. He showed no particular interest in it
2. What did Charles consider to be the hardest part of the training?
A. feeling trapped in the heavy spacesuit
B. endlessly practising the lunar surface landing
C. constantly being afraid of making a mistake
3. How did the crew feel when they had landed on the moon?
A. They felt as if they were coming home
B. They realised they had achieved something special
C. They were afraid of what they might find on the surface
4. What feature of the moon made the greatest impact on Charles?
A. the brightness of the
B. the vastness of the sky
C. the loneliness of the place
5. What does Charles feel was the most memorable part of his mission?
A. holding a piece of the moon
B. walking on the moon’s surface
C. seeing things never seen before

Your answers:
1. 2. 3. 4. 5.

Part 3. Listen and decide which of the following statements are TRUE (T) or FALSE (F)
(CAE) (10 points)
1. Maggie has met the teacher once before.
2. The teacher believes there hasn't really been an improvement since last years.
3. Billy's mother didn't know that he had a story published by the school.
4. Billy's mother was always aware of Billy's spelling problems.
5. The teacher had dyslexia himself when he was much younger.

Your answers:
1. 2. 3. 4. 5.
Part 4. Listen and fill in the blank with NO MORE THAN THREE WORDS (FCE) (20
points)
You will hear part of a radio talk for young people about animals communicating with
each other. Complete the notes below which summarize what the speaker says. Write NO
MORE THAN THREE WORDS for each numbered space.
Bees do a (1)_________________ to communicate where to find food.
Although parrots seem to speak, they are only (2) ________________ the human sounds.
Primates can communicate a few (3) ________________ using simple sounds.
Monkeys have not been observed to use any kind of (4) _________________.
Although dolphins can make vowel sounds, they cannot accurately imitate our (5)
_______________.
Amazingly, dolphins demonstrate (6) ________________ of when phrases should be used.
The sounds made by whales contain (7) _______________ than human speech.
The songs of the bottle-nosed whale have many of the (8) _______________ of human speech.
The unique grammatical nature of human language arose due to life in (9) ______________.
Indeed, a young child needs enough (10) ________________with other people to develop
speech.
Your answers:
1. 6.
2. 7.
3. 8.
4. 9.
5. 10.

B. LEXICO-GRAMMAR
Part 1. Choose the best answer to fill in each blank. (20 points)
1. The thick fog _______ out any possibility of our plane taking off before morning.
A. ruled B. struck C. stamped D. crossed
2. Ellaine decided that the election to the local council would provide a ________ to a career in
national politics.
A. springboard B. turning point C. milestone D. highway
3. We do not know what our guests will be wanting to do this weekend. We will have to ____
A. play it by ear B. bend our ears about it
C. be on our ear D. turn a deaf ear to it.
4. Although she had been told quite ________ to pull herself together, she simply couldn’t stop
crying.
A. rigidly B. unsympathetically C. unrelentingly D. sternly
5. Now I associate public transport with one of the worst experiences of my life and the
_________ of it is that I will never catch a bus again.
A. thick and thin B. hard and fast
C. long and short D. ups and downs
6. As they travelled across the _________ landscape, each one of them wondered how it was
possible to grow anything there.
A. barren B. bustling C. grimy D. mundane
7. The rain ________ down slowly under his coat collar, making him feel thoroughly damp and
miserable.
A. crept B. waded C. trickled D. teemed
8. It was only when we heard a stifled cough that we realised someone was _____ in the
underground.
A. concealing B. ambushing C. lurking D. sneaking
9. Aunt Doris has always been a bit of a hoarder but when we realized just how much ________
she had accumulated of late, even we were shocked.
A. chaos B. confusion C. muddle D. clutter
10. The hotel’s description in the brochure was ________ in the extreme and we were left utterly
disappointed on arrival.
A. fallacious B. pretentious C. perplexing D. erratic
11. The search ________ had no luck in finding the missing child.
A. crew B. group C. staff D. party
12. A strange rattling noise forced the driver to _______ onto the hard shoulder.
A. stop over B. pull over C. get away D. throw away
13. Peter: “ You don’t need a raincoat. The weather’s fine.”
Mary: “ ________, I think I’ll take one; you never know.”
A. Same again B. One and the same C. All the same D. Same old, same old
14. I don’t understand why people can’t just ________ instead of trying to bend everyone to their
will.
A. live and let live B. live and breathe C. live rough D. live in clover
15. If we _______ over the details, we’ll never finish filming this episode by today.
A. conserve B. niggle C. huddle D. mob
16. The CEO refused _________ to discuss any of the staff’s demaands for more pay.
A. down and about B. outright C. outlook D. downright
17. “Please don’t _________ the pots and pans or you’ll wake the baby up.”
A. whoosh B. squeak C. clatter D. crush
18. We knew the concert was sold out, but we still went to the stadium _______ the off-chance
that someone might want to sell us their tickets.
A. with B. by C. on D. in
19. They were the best economic analysists in the United States- a team hand- ______by the
President himself.
A. selected B. picked C. named D. settled
20. After travelling all day, he was completely _________.
A. done in B. done up C. broken down D. used up

Your answers:
1. 5. 9. 13. 17.
2. 6. 10. 14. 18.
3. 7. 11. 15. 19.
4. 8. 12. 16. 20.
Part 4. Give the correct form of each word in the brackets (10 points)
1. The mystery of Loch Ness Monster still remains ________. (SOLVE)
2. In the 19th century, in America, many black children were sold into ________. (SLAVE)
3. The potential benefits of COVID vaccination __________ the risks. (WEIGHT)
4. Doctors and nurses sometimes complain of being ___________ and overworked. (PAY)
5. The ___________ system of the USA bases on gold. (MONEY)
6. The ___________ in agriculture is essential if we want to produce more food. (MACHINE)
7. Quang Hai is an _______ football player in Vietnam. (EXCEPT)
8. This remote village is __________ during the flood. It is completely isolated by rivers.
(ACCESS)
9. If you don't tease the dog, it is ________ to bite you. (LIKE)
10. It's difficult to teach people what they are ________ to learn. (WILL)

Your answers:
1. 6.
2. 7.
3. 8.
4. 9.
5. 10.

C. READING (50 points)


Part 1. Read the following passage and decide which answer (A, B, C, or D) best fits each gap.
Write your answers in corresponding numbered boxes. (10 points)
OXFORD
Oxford is a city with such a mind-bowling reputation that many who come here find
themselves intimidated by the place and can’t wait to leave, while others, taking to it like a
________(1) to water, find themselves returning again and again. The college lawns provide a
gorgeous ________(2) to serious study, and in the right light, on a sunny winter’s morning say,
one feels as if one is ________(3) on air, such is the sense of unreality. Oxford may like to
________(4) that it is at the intellectual ________(5) of things, but in many ways it is no more
than a sleepy ________(6) where, to mix metaphors, transitory students, the ________(7) of their
generation, wait in the ________(8), allowing their talents to ________(9) before moving off
into the industrial or political ________(10).

1. A. fish B. boat C. duck D. swimmer


2. A. curtain B. backdrop C. scene D. screen
3. A. flying B. floating C. swimming D. gliding
4. A. act B. pretend C. dissemble D. produce
5. A. wheel B. engine C. spoke D. hub
6. A. stream B. tributary C. backwater D. watershed
7. A. froth B. cream C. fat D. caviar
8. A. pavilion B. dressing room C. wings D. foyer
9. A. flourish B. open C. spread D. float
10. A. peak B. abattoir C. dead-end D. fast-lane
Your answers
1. 2. 3. 4. 5.
6. 7. 8. 9. 10.

Part 2. Read the text below and think of the word which best fits each space. Use only ONE
word in each space. Write your answers in the corresponding numbered boxes. (15 points)

NOISE POLLUTION
Most of us are very used to the sounds we hear in (1) __________ life. Loud music, the
television, people talking (2) __________ their phone, the traffic and even pets barking in the (3)
__________ of the night. All of these have become a part of the urban culture and (4)
__________ disturb us. However, when the sound of the television keeps you (5) __________
sleeping all night or the traffic starts to give you a headache, it stops becoming just noise and
starts turning (6) __________ noise pollution. For many of us, the concept of pollution is limited
to nature and resources. (7) __________, noise that tends to disrupt the natural rhythm of life
makes for one solid pollutant.
By definition, noise pollution takes place when there is (8) __________ excessive
amount of noise or an unpleasant sound that causes temporary disruption in the natural balance.
This definition is usually applicable to sounds or noises that are unnatural in their volume or their
production. Our environment is (9) __________ that it has become difficult to escape noise.
Even electrical appliances at home have a constant hum or beeping sound. By and large, lack of
urban planning increases the (10) __________ to unwanted sounds. This is why understanding
noise pollution is necessary to control it in time.

Your answers
1. 2. 3. 4. 5.
6. 7. 8. 9. 10.

Part 3. Read the passage and choose the best option A, B, C, or D to answer the questions.
Write your answers in the corresponding numbered boxes. (10 points)
Before the 1500’s, the western plains of North America were dominated by farmers. One
group, the Mandans, lived in the upper Missouri River country, primarily in present – day North
Dakota. They had large villages of houses built close together. The tight arrangement enabled
the Mandans to protect themselves more easily from the attacks of others who might seek to
obtain some of the food these highly capable farmers stored from one year to the next.
The women had primary responsibility for the fields. They had to exercise considerable
skill to produce the desired results, for their northern location meant fleeting growing seasons.
Winter often lingered; autumn could be ushered in by severe frost. For good measure, during the
spring and summer, drought, heat, hail, grasshoppers, and other frustrations might await the wary
grower.
Under such conditions, Mandan women had to grow maize capable of weathering
adversity. They began as early as it appeared feasible to do so in the spring, clearing the land,
using fire to clear stubble from the fields and then planting. From this point until the first green
corn could be harvested, the crop required labor and vigilance.
Harvesting proceeded in two stages. In August the Mandans picked a smaller amount of
the crop before it had matured fully. This green corn was boiled, dried and shelled, with some of
the maize slated for immediate consumption and the rest stored in animal – skin bags. Later in
the fall, the people picked the rest of the corn. They saved the best of the harvest for seeds or for
trade, with the remainder eaten right away or stored for alter use in underground reserves. With
appropriate banking of the extra food, the Mandans protected themselves against the disaster of
crop failure and accompany hunger.
The woman planted another staple, squash, about the first of June, and harvested it near
the time of the green corn harvest. After they picked it, they sliced it, dried it, and strung the
slices before they stored them. Once again, they saved the seeds from the best of the year’s crop.
The Mandans also grew sunflowers and tobacco; the latter was the particular task of the older
men.
1. What is the main topic of the passage?
A. The problems encountered by farmers specializing in growing once crop
B. The agricultural activities of a North American Society
C. Various ways corn can be used.
D. Weather conditions on the western plains.
2. The Mandans built their houses close together in order to _____.
A. guard their supplies of food B. protect themselves against the weather
C. share farming implements D. allow more room for growing corn
3. The word “enabled” in the first paragraph is closest in meaning to _____.
A. covered B. helped C. reminded D. isolated
4. Why does the author believe that the Mandans were skilled farmers?
A. They developed effective fertilizers.
B. They could grow crops in most types of soil.
C. They could grow crops despite adverse weather.
D. They developed new varieties of corn.
5. The word “consumption” in paragraph 4 is closest in meaning to _____.
A. decay B. conversation C. eating D. planting
6. Which of the following processes does the author imply was done by both men and women?
A. clearing fields B. Harvesting squash
C. Harvesting corn D. Planting corn
7. The word “disaster” in paragraph 4 is closest in meaning to _____.
A. control B. catastrophe C. history D. avoidance
8. The word “them” in the last paragraph refers to _____.
A. women B. seeds C. slices D. the Mandans
9. Which of the following crops was cultivated primarily by men?
A. Sunflower B. Corn C. Squash D. Tobacco
10. Throughout the passage, the author implies that the Mandans _____.
A. valued individuality B. were very adventurous
C. were open to strangers D. planned for the future

Your answers
1. 2. 3. 4. 5.
6. 7. 8. 9. 10.

Part 4. Read the text and do the following tasks. (15 points)
Questions 1-5
Choose the correct heading for paragraphs B-G from the list of headings below..
Write the correct number i-ix, in boxes 27-32 on your answer sheet.

List of Headings

i. The reaction of the Inuit community to climate change


ii. The benefits of an easier existence
iii. Alternative sources of essential supplies
iv. Respect for Inuit opinion grows
v. A healthier choice of food
vi. A difficult landscape
vii. Negative effects on well-being
viii. Alarm caused by unprecedented events in the Arctic

Example Answer
Paragraph A viii
Your answers:
1. Paragraph B ………….
2. Paragraph C .………….
3. Paragraph D …………..
4. Paragraph E …………..
5. Paragraph F ………….
Climate change and the Canada's Inuit people
A. Unusual incidents are being reported across the Arctic. Inuit families going off on
snowmobiles to prepare their summer hunting camps have found themselves cut off from home
by a sea of mud, following early thaws. There are reports of igloos losing their insulating
properties as the snow drips and refreezes, of lakes draining into the sea as permafrost melts, and
sea ice breaking up earlier than usual, carrying seals beyond the reach of hunters. Climate change
may still be a rather abstract idea to most of us, but in the Arctic it is already having dramatic
effects - if summertime ice continues to shrink at its present rate, the Arctic Ocean could soon
become virtually ice-free in summer. The knock-on effects are likely to include more warming,
cloudier skies, increased precipitation and higher sea levels. Scientists are increasingly keen to
find out what's going on because they consider the Arctic the 'canary in the mine' for global
warming - a warning of what's in store for the rest of the world.
B. For the Inuit the problem is urgent. They live in precarious balance with one of the toughest
environments on earth. Climate change, whatever its causes, is a direct threat to their way of life.
Nobody knows the Arctic as well as the locals, which is why they are not content simply to stand
back and let outside experts tell them what's happening. In Canada, where the Inuit people are
jealously guarding their hard-won autonomy in the country's newest territory, Nunavut, they
believe their best hope of survival in this changing environment lies in combining their ancestral
knowledge with the best of modern science. This is a challenge in itself.
C. The Canadian Arctic is a vast, treeless polar desert that's covered with snow for most of the
year. Venture into this terrain and you get some idea of the hardships facing anyone who calls
this home. Farming is out of the question and nature offers meagre pickings. Humans first settled
in the Arctic a mere 4,500 years ago, surviving by exploiting sea mammals and fish. The
environment tested them to the limits: sometimes the colonists were successful, sometimes they
failed and vanished. But around a thousand years ago, one group emerged that was uniquely well
adapted to cope with the Arctic environment. These Thule people moved in from Alaska,
bringing kayaks, sleds, dogs, pottery and iron tools. They are the ancestors of today's Inuit
people.
D. Life for the descendants of the Thule people is still harsh. Nunavut is 1.9 million square
kilometres of rock and ice, and a handful of islands around the North Pole. It's currently home to
2,500 people, all but a handful of them indigenous Inuit. Over the past 40 years, most have
abandoned their nomadic ways and settled in the territory's 28 isolated communities, but they
still rely heavily on nature to provide food and clothing. Provisions available in local shops have
to be flown into Nunavut on one of the most costly air networks in the world, or brought by
supply ship during the few ice-free weeks of summer. It would cost a family around £7,000 a
year to replace meat they obtained themselves through hunting with imported meat. Economic
opportunities are scarce, and for many people state benefits are their only income.
E. While the Inuit may not actually starve if hunting and trapping are curtailed by climate
change, there has certainly been an impact on people's health. Obesity, heart disease and diabetes
are beginning to appear in a people for whom these have never before been problems. There has
been a crisis of identity as the traditional skills of hunting, trapping and preparing skins have
begun to disappear. In Nunavut's 'igloo and email' society, where adults who were born in igloos
have children who may never have been out on the land, there's a high incidence of depression.
F. With so much at stake, the Inuit are determined to play a key role in teasing out the
mysteries of climate change in the Arctic. Having survived there for centuries, they believe their
wealth of traditional knowledge is vital to the task. And Western scientists are starting to draw
on this wisdom, increasingly referred to as 'Inuit Qaujimajatugangit', or IQ. 'In the early days
scientists ignored us when they came up here to study anything. They just figured these people
don't know very much so we won't ask them,' says John Amagoalik, an Inuit leader and
politician. 'But in recent years IQ has had much more credibility and weight.' In fact it is now a
requirement for anyone hoping to get permission to do research that they consult the
communities, who are helping to set the research agenda to reflect their most important concerns.
They can turn down applications from scientists they believe will work against their interests, or
research projects that will impinge too much on their daily lives and traditional activities.
Questions 6-10: Complete the summary of paragraphs C and D.
Choose ONE word from paragraphs C and D for each numbered blank. Write the
answers in boxes.
If you visit the Canadian Arctic, you immediately appreciate the problems faced by
people for whom this is home. It would clearly be impossible for the people to engage in
(6)________ as a means of supporting themselves. For thousands of years they have had to rely
on catching sea (7)________ and fish as a means of sustenance. The harsh surroundings saw
many who tried to settle there pushed to their limits, although some were successful. The
(8)__________ people were an example of the latter and for them the environment did not prove
unmanageable. For the present inhabitants, life continues to be a struggle. The territory of
Nunavut consists of little more than ice, rock and a few (9) __________ . In recent years, many
of them have been obliged to give up their nomadic lifestyle, but they continue to depend mainly
on (10) __________ for their food and clothes.
Your answers:
6. 7. 8. 9. 10.
D. WRITING (50 points)
Part 1. Rewrite the following sentences using the words given
1. After years of separation from my girlfriend, I cheered up when I saw her. SIGHT
=> After years of separation from my girlfriend, my spirits ___________________________
of her.
2. He loses his temper at all things, even the slightest one. OFF
=> He ______________________________________________________________________
3. My parents are furious with me for getting a tattoo and so have grounded me. ARMS
=> My parents, who __________________________________________________________
4. I resent the way that she clearly feels herself to be superior to me. NOSE
=> I am ____________________________________________________________________
5. Although Jack never means to hurt other people, Jennie always complains about his
manner. FAULT
=> Despite the _____________________________________________________________
Part 2: Graph Description (20 points)
The chart below provides information about a pharmacy's sales in 2021. Summarize
the information by selecting and reporting key features, and compare if relevant.
Part 3. Essay writing. (30 points)
Write an essay of about 250 words to express your opinion on the following topic:
The Law on Road Traffic stipulates that only students aged 16 and over can use
electric bicycles. Many people agree that this is necessary, while others argue that it will create
difficulties for both students and parents.
Discuss both sides of the issue and give your own opinions.
Use your own knowledge and experience to support your arguments with examples and relevant
evidence.

TEST 13

LISTENING : 50 points

Part 1. You will hear a phone call between a receptionist and a customer

Questions 1-7

Complete the notes below.

Write ONE WORD AND/OR A NUMBER for each answer.

Enquiry about booking hotel room for event


Rooms

Adelphi Room

number of people who can sit down to eat: 1………………….

has a gallery suitable for musicians

can go out and see the 2………………… in pots on the terrace

terrace has a view of a group of 3……………………

Carlton Room

number of people who can sit down to eat: 110

has a 4……………………

view of the lake

Options

Master of Ceremonies:

can give a 5………………… while people are eating

will provide 6………………… if there are any problems

Accommodation:

in the hotel rooms or 7…………………

Your answer
1 2 3 4 5 6 7

Part2

You will hear a photographer called lan Gerrard talking about his career. For questions 8-15,

complete the sentences with a word or short phrase.

lan Gerrard - Photographer

The subject that lan studied at university was 8_________________________________


lan did a presentation on 9_____________________ as part of his final year.

lan worked for a 10_______________________ in the USA for a year after leaving university.

When he travelled around the USA, lan chose 11______________________ as the theme for his
photographs.

lan says that winter is the season when he takes the best photographs.

When lan came back to Britain, he travelled around by 12__________________________ taking


photographs.

lan says he was surprised by how few photographers specialise in shots of 13 ______________
communities.

lan's book will be available in bookshops in March next year.

The title of lan's book is 14_____________________

lan has chosen 15 __________________ as the theme for his next tour.

Your answer
8 9 10 11 12 13 14 15

Part 3

You will hear an explorer called Richard Livingstone talking about a trip he made in the
rainforest of South America. Listen and indicate true (T) or false (F) statements. (10 points)

Statements T F

16. They went all the way by boat.

17. Richard say that during the walk, they were always both cold and wet.

18. In a deserted camp, they found some soup made from unusual meat and vegetables.

19. After the meal, they began to feel worried about what they have done.

20. Before leaving the camp, they left the sum of 50 dollars to thank the host.

Your answer
16 17 18 19 20
Part 4

You will hear an interview with a student athlete called Chelsea Matthews, who plays soccer
for her college. For questions 21-25 choose the best answer (A, B, C or D).

21 What impact does playing soccer have on Chelsea's life?

A She needs private tuition from her teachers.

B She doesn't take part in some other student activities.

C She never gets to travel to other countries.

D She misses out vacation abroad

22 Chelsea had to start planning to be a student athlete at 16 because

A there was much paperwork she had to finish

B there were many requirements that had to be met.

C there were few colleges that offered the course she wanted.

D there was a lot of competition for places in good colleges.

23 When Chelsea and her team-mates finish training, they

A can take a break by going to the movies.

B relax with other sports teams.

C are too tired to do very much except sleep.

D spend time participating parties

24 Chelsea says if she and her team-mates miss too many classes

A they may get poor grades and have to leave the team.

B they remain have high grades although other students are jeolous

C their professors will complain to the head of faculty.

D the other students are understanding about the reason for their absence.
25 What problem did Chelsea herself have in keeping up with her studies?

A She was away sick for some of her classes.

B She was expected to commit herself to extra training for away games.

C She could be absent from class if she was sick

D She had to study one subject under difficult conditions.

Your answer
21 22 23 24 25

LEXICO- GRAMMAR

Part 1: Choose the best option A, B, C, or D to complete the following sentences and write
your answers in the corresponding numbered boxes. (20 points)
1. We must be careful not to put the cart before the ________.
A. bull B. horse C. buffalo D. cow
2. The article is about how children use these representations to _______ language learning.
A. facilitate B. communicate C. complicate D. fascinate
3. Could you possibly _______ in for me at the next committee meeting? I’ll be busy then.
A. be B. go C. put D. sit
4. I don’t believe there’s a _________ of evidence that could be held against him.
A. shred B. grain C. drop D. strain
5. ________ boy talking to my mom is ________ famous musician. He can play _______
guitar very well.
A. The/a/the B. A/ϴ/ϴ C. The/ϴ/the D. A/a/the
6. The computers needed a strong, clear ________ light to track, as a kind of reference
point .
A. leading B. guiding C. flashing D. heading
7. I used to kick up a _______ about eating pho with spring onion, but I’ve got used to it.
A. mess B. fuss C. trouble D. riot
8. I can’t _______ where the thieves have gone. It’s so foggy out here.
A. see through B. look into C. make out D. take in
9. Either John or Elsa _________ computer now.
A. is using B. are using C. uses D. use
10. Far more should be done to ______ the sufferings of unwanted domestic pets.
A. ease B. improve C. remove D. remedy
11. Johnny has got his degree with flying ______.
A. colours B. flags C. balloons D. hands
12. I don’t like ______ by a cop car ______ I am driving alone because it makes me nervous.
A. being followed/while B. being followed/where
C. to follow/while D. to be followed/that
13. Don’t look so worried! You should take the boss’s remarks with a ______ of salt.
A. teaspoon B. dose C. pinch D. grain
14. After I finished playing, she told me that I hadn’t ______ the instrument correctly.
A. had B. carried C. held D. kept
15. Nick threw ______ his chances of getting admitted into Stanford by spending too much
time wondering about his abilities.
A. off B. on C. in D. away
16. Tony was in a ______ of absolute shock.
A. mode B. state C. time D. mess
17. The earthquake _______ 6.5 on the Richter scale.
A. weighed B. measured C. achieved D. counted
18. He keeps trying to get it published but I think he's flogging a dead _________.
A. duck B. dog C. horse D. mouse
19. People ________ the Earth was flat until Aristotle _________ that it was not.
A. believed/was proving B. had believed/proved
C. believed/had proved D. had believed/ would prove
20. The latest novel by Grant is hilarious. It had me ______ stitches.
A. in B. at C. to D. with

Your answer

1 2 3 4 5 6 7 8 9 10

11 12 13 14 15 16 17 18 19 20

Part 2. Put the words in the brackets in the correct forms ( 10 points)

1 Human faces are all built pretty much the same: two eyes above a nose that’s above a mouth,
the features ______ (VARIETY) from person to person generally by mere millimeters.

2. The abolitionist movement began in the 1600s when the Quakers in Pennsylvania objected to
__________________ (SLAVE) on moral grounds.

3. This part of the law is only ___________________ (APPLY) to companies employing more
than five people.

4 The architects have made________________ (IMAGINE) use of glass and transparent plastic.

5. Improved safety measures in cars can be ____________________ ( PRODUCE) as they


encourage people to drive faster.
6. The car broke down, which was an______________________ (PROMISE) start to the
holiday.

7. The people of this country will continue in their struggle for ______________________
(DEPEND).

8. Taking the wrong antibiotic, or taking it ____________________ (REGULARITY) could


compromise your immune system.

9. The sad condition of the horses suggested a long period of ______________________


(TREAT) and neglect.

10 I know he's_____________________ (MEAN), but I wish he'd leave us alone

Your answer
1 2 3 4 5
6 7 8 9 10

READING: 60 points

Part 1: Read the following passage and decide which answer (A, B, C, or D) best fits each
gap. Write your answers in corresponding numbered boxes. (10 points)

THE STICKING PLASTER

Nowadays, one of the most common items found in the home is the sticking plaster.

Protecting a cut by covering the affected area with a piece of material that sticks to the skin

may seem a rather obvious idea, so it is perhaps surprising to learn that the plaster was not

1_______ until about ninety years ago.

The person who thought 2________ the idea was Earle Dickson, an employee of the

Johnson and Johnson company. Concerned that his wife Josephine sometimes had

accidents while cooking and doing 3________ jobs, he used pieces of cotton materials
placed inside strips of sticky tape to cover her injuries. This prevented dirt from getting into

the 4_______ and protected it from further harm as she did the housework.

Dickson’s boss was impressed, so in 1921, Johnson and Johnson put the new sticking plaster

into 5________ under the brand name Band-Aid. Sales at first were slow, but somebody at

the company came up with the 6_______ idea of giving free plasters to the Boy Scouts.

This created publicity and from then on it became a 7________ success. Dickson was

8_______ within the company, eventually becoming a senior executive.

Although the basic design of the sticking plaster has remained similar to the 9_______,

there have been many developments in the materials used and it is now 10_______ in a

variety of shapes, sizes and colors. Total worldwide sales are believed to have exceeded 10

billion.

1. A. realised B. imagined C. dreamt D. invented

2. A. forward B. in C. over D. up

3. A. others B. other C. another D. the others

4. A. wound B. breakage C. damage D. tear

5. A. construction B. creation C. production D. formation

6. A. shining B. bright C. eager D. keen

7. A. monetary B. economical C. commercial D. financial

8. A. promoted B. raised C. lifted D. advanced

9. A. original B. model C. sample D. standard

10. A. accessible B. available C. attainable D. achievable

Your answer:
1. 2. 3. 4. 5.
6. 7. 8. 9. 10.

Part 2. Read the text below and think of the word which best fits each space. Use only ONE
word in each space. (10 points)

SEASON AND BABY SIZE

People born in certain months of the year tend to be taller than those born in others. This
discovery was (1) __________ in a recent study carried out at the Danish Epidemiology Science
Centre in Copenhagen.

The researchers looked at the height and weight details (2) __________birth of over a
million Danes born in the 1970s, 1980s and 1990s, and found that a child born in April is, on
average, nearly two centimeters taller than one born in December. As (3) __________ as being
taller, the April baby is also likely to be heavier, and the odds (4) __________ that he or she will
remain bigger throughout life.

The study also shows that (5) __________ children born in December tend to be the
smallest, the size of those born in June and July is also well below (6) __________ of the April
babies. In these two midsummer months, however, the difference in height and weight is only
half as much as in midwinter.

Similar results (7) __________ these emerged from a separate study in Australia. There
they found that by the time children reached 18, the gap between the April and the December
ones had increased to about five centimeters.

Another seasonal variation that (8) __________ to light concerned actual birth dates.
Winter babies, it was discovered, were born an average of one day earlier than spring, summer or
autumn babies.

In (9) __________ of these studies were scientists able to explain these differences. In
both cases they called (10) __________ further research into the subject.

Your answers:

1. 6.
2. 7.
3. 8.
4. 9.
5. 10.

Part 3. For questions 1-10, read an extract from an article on a model and choose the answer
A, B, C or D that fits best according to the text. Write your answers in the corresponding
numbered boxes provided. (10 pts)

1 My lawyer, Mr. Turner, is the only man I know who has seen a ghost. He is a
quiet even-tempered man whose life is spent in dealing with facts. He is the last
person in the world to give way to fantasy. He has a wife and two children of whom
he is proud, takes a modest holiday abroad every year and spends his Sundays
5 gardening. He is knowledgeable about art and architecture, though he doesn't
pretend to be an expert by any means. It is, therefore, all the more surprising that he
should so insistent about the ghost. It happened, so he says, like this:
He was travelling from London to the North of England by train. It was a
misty November evening and the train was half empty. In fact for the first part of
10 the journey Mr Turner had the carriage to himself and sat dozing over a newspaper.
However at the first stop a passenger jumped in, slamming the door behind him. He
seemed out of breath as if he had been running. He was a striking looking young
man with dark, bushy hair and bright intelligent eyes. He was dressed rather oddly
in a long waisted coat with silver buttons, tight trousers and an embroidered
15 waistcoat. Mr Turner did not pay much attention to this because people wear all
sorts of extravagant clothes these days and he had long grown accustomed to them.
Presently, the two men got into conversation, as people do on long journeys.
Mr Turner was interested to cover that the young man was very knowledgeable
about Art - in particular portraits. His name, he said, was Joseph Hart, and he was
20 on his way to visit an exhibition. It seemed that he worked in a famous London Art
Gallery - a picture restorer, perhaps, thought Mr Turner, for he seemed to know a
great deal about varnishes and paints, and even more about the subjects of certain
portraits. When Mr Turner asked his opinion of the portrait of a famous judge by an
artist he admired, his companion laughed and said, “He’s only a reproduction - a
25 good one I agree but you can't talk to a reproduction.” He spoke as though the
person in the portrait were still living.
After a while the carriage got hot and steamy and Mr Turner dropped off. He
woke up just as the train was drawing up at a junction with a grinding of brakes. His
companion had disappeared.
30 A few days later, having returned to London, Mr Turner found himself near the
Art Gallery. Moved by some impulse, he went in and enquired for Joseph Hart. The
attendant directed him to a room devoted to early nineteenth century portraits of
well-known men. There was no-one in the room and Mr Turner looked about him.
Without knowing quite how he had got there, he found himself standing in front of
35 a full-length portrait of a dark young man in tight trousers and an embroidered
waistcoat. The eyes smiled at him with a hint of amusement. The name-plate at the
foot of the picture read: Joseph Hart, Gentleman, 1800-1835.

1. What kind of person was Mr Turner?


A. imaginative B. fantastic C. sensible D. insensitive
2. Although he was a lawyer, Mr Turner
A. pretended to know a lot about Art B. knew something about Art.
C. pretended to take an interest in Art. D. intended to learn more about Art.
3. When the passenger entered Mr Turner’s compartment.
A. he was painting. B. he was running.
C. the train was just leaving D. the carriage was half-empty.
4. The passenger’s clothes didn’t seem strange to Mr Turner because
A. he was used to wearing strange clothes. B. he liked people who wore strange clothes.
C. everyone he knew wore strange clothes. D. he had seen a lot of people in strange
clothes.
5. Mr Turner thought the young man might
A. be an art dealer. B. be an Art Expert.
C. renew old pictures. D. paint reproductions of old pictures.
6. Why wouldn’t the passenger give an opinion on the portrait of the judge?
A. the judge wasn’t alive. B. the judge was still alive.
C. the picture was a copy. D. he hadn’t seen it.
7. When did Mr Turner first realize that the passenger had gone?
A. When the train started. B. After the train had stopped.
C. Just before the train stopped. D. When the train was leaving the station.
8. Why did Mr Turner go into the Art Gallery?
A. He was walking past there. B. He had never been there before.
C. He had planned to do so. D. He suddenly decided to.
9. In the past of the gallery that Mr Turner was directed to
A. there were a lot of pictures by unknown B. there were a lot of nineteenth century
people. people.
C. no-one else was looking at the pictures. D. he only saw one portrait.

10. When Mr Turner looked at the portrait of Joseph Hart


A. he smiled at it. B. he thought it smiled at him.
C. he didn’t recognize it. D. he was amused.

Your answers:

1 2. 3. 4. 5. 6. 7. 8. 9. 10.

Part 4. Read the following passage and do the following tasks. (20 points)

IV. For questions 1–10, read the following passage.


List of Headings
i. A business-model approach to education

ii. The reforms that improved education in Finland

iii. Educational challenges of the future

iv. Ways in which equality is maintained in the Finnish education system

v. The benefits of the introduction of testing

vi. An approach that helped a young learner

vii. Statistical proof of education success

viii. Support for families working and living in Finland

ix. The impact of the education system on Finland's economy

Why Are Finland's Schools Successful?


The country's achievements in education have other nations doing their homework
A. At Kirkkojarvi Comprehensive School in Espoo, a suburb west of Helsinki, Kari
Louhivuori, the school's principal, decided to try something extreme by Finnish standards.
One of his sixth-grade students, a recent immigrant, was falling behind, resisting his
teacher's best efforts. So he decided to hold the boy back a year. Standards in the country
have vastly improved in reading, math and science literacy over the past decade, in large
part because its teachers are trusted to do whatever it takes to turn young lives around. 'I
took Besart on that year as my private student,' explains Louhivuori. When he was not
studying science, geography and math, Besart was seated next to Louhivuori's desk, taking
books from a tall stack, slowly reading one, then another, then devouring them by the
dozens. By the end of the year, he had conquered his adopted country's vowel-rich language
and arrived at the realization that he could, in fact, learn.

B. This tale of a single rescued child hints at some of the reasons for Finland's amazing record
of education success. The transformation of its education system began some 40 years ago
but teachers had little idea it had been so successful until 2000. In this year, the first results
from the Programme for International Student Assessment (PISA), a standardized test given
to 15-year-olds in more than 40 global venues, revealed Finnish youth to be the best at
reading in the world. Three years later, they led in math. By 2006, Finland was first out of
the 57 nations that participate in science. In the latest PISA scores, the nation came second
in science, third in reading and sixth in math among nearly half a million students
worldwide.

C. In the United States, government officials have attempted to improve standards by


introducing marketplace competition into public schools. In recent years, a group of Wall
Street financiers and philanthropists such as Bill Gates have put money behind private-sector
ideas, such as charter schools, which have doubled in number in the past decade. President
Obama, too, apparently thought competition was the answer. One policy invited states to
compete for federal dollars using tests and other methods to measure teachers, a philosophy
that would not be welcome in Finland. 'I think, in fact, teachers would tear off their shirts,'
said Timo Heikkinen, a Helsinki principal with 24 years of teaching experience. 'If you only
measure the statistics, you miss the human aspect.'

D. There are no compulsory standardized tests in Finland, apart from one exam at the end of
students' senior year in high school. There is no competition between students, schools or
regions. Finland's schools are publicly funded. The people in the government agencies
running them, from national officials to local authorities, are educators rather than business
people or politicians. Every school has the same national goals and draws from the same
pool of university-trained educators. The result is that a Finnish child has a good chance of
getting the same quality education no matter whether he or she lives in a rural village or a
university town.

E. It's almost unheard of for a child to show up hungry to school. Finland provides three years
of maternity leave and subsidized day care to parents, and preschool for all five-year-olds,
where the emphasis is on socializing. In addition, the state subsidizes parents, paying them
around 150 euros per month for every child until he or she turns 17. Schools provide food,
counseling and taxi service if needed. Health care is even free for students taking degree
courses.
F. Finland's schools were not always a wonder. For the first half of the twentieth century, only
the privilege a got a quality education. But In 1963, the Finnish Parliament made the bold
decision to choose public education as the best means of driving the economy forward and
out of recession. Public schools were organized into one system of comprehensive schools
for ages 7 through 16. Teachers from all over the nation contributed to a national curriculum
that provided guidelines, not prescriptions, for them to refer to. Besides Finnish and Swedish
(the country's second official language), children started learning a third language (English
is a favorite) usually beginning at age nine. The equal distribution of equipment was next,
meaning that all teachers had their fair share of teaching resources to aid learning. As the
comprehensive schools improved, so did the upper secondary schools (grades 10 through
12). The second critical decision came in 1979, when it was required that every teacher gain
a fifth-year Master's degree in theory and practice, paid for by the state. From then on,
teachers were effectively granted equal status with doctors and lawyers. Applicants began
flooding teaching programs, not because the salaries were so high but because autonomous
decision-making and respect made the job desirable. And as Louhivuori explains, 'We have
our own motivation to succeed because we love the work.'

Your answers:
0. Paragraph A: vi. (example) 1. Paragraph B: 2. Paragraph C:
3. Paragraph D: 4. Paragraph E: 5. Paragraph F:

Complete the notes below. Choose NO MORE THAN TWO WORDS AND/OR A
NUMBER from the passage for each answer.

The school system in Finland


PISA tests

● In the most recent tests, Finland's top subject was (6) …………………………………

History
1963:

● A new school system was needed to improve Finland's (7) ……………………………


● Schools followed (8) …………………………… that were created partly by teachers.

● All teachers were given the same (9) ………………………………… to use.

1979:

● Applicants were attracted to the (10) …………………………… that teaching received.

Your answers:

6. 7. 8. 9. 10.

WRITING: 50 points
Part 1. Finish each of the following sentences in such a way that it means the same as the one
printed before it. Write your answers in the space provided. (10 points)

1. The only thing that makes this job worthwhile is the experience.

Were it _________________________________________________________________

2. He forgot about the appointment with the dentist until he came home

Not until ________________________________________________________________

3. If the weather is fine, we may go camping at the weekend

Weather _____________________________________________________________.

4. Government guidelines really do emphasize the importance of starting education early.

A lot of emphasis _____________________________________________________.

5 He worked very hard, but he was unable to earn enough for his living.
Hard-working _____________________________________
Part 2. Use the word given in brackets and make any necessary additions to write a new
sentence in such a way that it is as similar as possible in meaning to the original sentence. Do
NOT change the form of the given word. You must use between THREE and EIGHT words,
including the word given. ( 10 points)

1. The news of the merger came as a complete surprise to the workers. ABACK
The workers ____________________________________ the news of the merger

2. The excuse for the declaration of the war is the defence of their territorial rights. PRETEXT

They declared war ___________________________________ their territorial rights.

3. I would give up my job in no time if I could find a better one. HAT

I would give up my job_______________________________________________

4. The letter arrived completely unexpectedly this morning. BLUE

The letter arrived ____________________________________________________

5. Jack has such a vivid imagination; it is possible that he invented the whole story. MADE

Jack has such a vivid imagination ________________________________________

Part3. Write a paragraph about 150-180 words ( 30 points)

What are the effects of tourism on the communities ?

TEST 14

SECTION A: LISTENING (50 points)

● There is a piece of music at the beginning and at the end of the listening part

● There are three parts, each will be played twice


● Before each part, students have 30 seconds to look at the questions
PART I. Complete the form below (14pts)

Write NO MORE THAN THREE WORDS AND OR A NUMBER for each answer

PACK HAM’S SHIPPING AGENCY – customer quotation form


Example

Kenya
Country of destination: ..........…..

Name:Jacob 1 …………

Address to be collected from: 2 ………… College, Downlands Rd

Town: Bristol

Postcode: 3 ………

Size of container:
Length: 1.5

Width: 4 ………… Height: 5 …………

Contents: clothes

books

6 …………

Total estimated value: 7 £…………

PART II. You will hear a radio programme about the history of roller skating and complete
the sentences. Write NO MORE THAN THREE WORDS AND OR A NUMBER for each
answer. (16pts)

HISTORY OF ROLLER SKATING

The country where the first roller skates were probably made was Holland. In 1760, John
Merlin went to a ball in London playing a violin whilst on roller skates. Unfortunately, John
Merlin injured himself when he broke a (1) __________ at the ball. In Germany, roller skating
was used in a ballet called (2) ________ . Plimpton's invention helped roller skaters to control
the (3) _______ of their skates. The first team sport to be played on roller skates was (4)
__________ . In Detroit in 1937, the first (5) ________ in the sport took place. The use of
plastics meant that both the (6) ________ and ________ of roller skates improved. The musical
Starlight Express was seen by as many as (7) _______ in London. The speaker says that modern
roller skates are now (8) ________ than ever before.

Your answers
1. 2. 3. 4.

5. 6. 7. 8.

PART III. You will hear an introduction to a radio phone-in programme about modern
lifestyles. Listen and indicate true (T) or false (F) statements. (10pts)

1. Ron is a record-breaking athlete.

2. Ron thinks an accountant can lead a health and fulfilled life.


3. “Total Living” is believed to be good for athletes.

4. “Total Living” means that we should develop one aspect of our life to the full.

5. According to Ron Clarke, some current health trends are harming us.

Your answers
1. 2. 3. 4. 5.

PART IV. Listen to the recording. (10pts)


You are going to hear two women talking about a holiday in France. Read the sentences, and
choose the best option: A, B, or C, to complete the statements about the recording.
1. Paula's friend says that
A. she has been ill.
B. Paula doesn't look very well.
C. she's pleased to see Paula.
2. Before the trip, Paula
A. was enthusiastic about it.
B. wanted to go to the Lake District.
C. didn't tell anybody she was going.
3. Before Mark and Paula went to Paris,
A. Mark's boss didn't want him to go.
B. Paula arranged for somebody to look after the hamster.
C. Paula's sister promised to look after the children.
4. The journey across the Channel
A. was very smooth.
B. was unpleasant for Paula.
C. lasted eight hours.
5. The return trip from Paris was
A. disturbed by a flood.
B. an enjoyable experience.
C. earlier than planned.
Your answers
1. 2. 3. 4. 5.

SECTION B: LEXICO-GRAMMAR (40 points)

Part I. Circle the correct answer A, B, C or D to each of the following questions. (20 points)
1. I’m afraid we haven’t got a spare bed. Can you ____ with a mattress on the floor?
A. make do B. make by C. make over D. make up

2. What ______ the smoke and the noise, the party made me feel quite ill.
A. if B. with C. through D. of

3. The _____ in our building often falls asleep at the front desk.
A. caretaker B. stockbroker C. undertaker D. bookmaker

4. Tom handed in the test and awaited the results _____.


A. in the same breath C. with bated breath
B. out of breath D. under her breath

5. Many people are- feeling the _____ now that there is an economic recession.
A. strain B. pain C. prod D. pinch

6. The various locales where he painted, many _____, continued to be a source of inspiration
throughout his long career.
A. off the beaten path B. away from it C. for a wander D. lazy round

7. Tax ______ deprives the state of several million pounds a year.


A. retention B. desertion C. escapism D. evasion

8. What are you going to do when all you money ________.


A. runs down B. runs dry C. runs deep D. runs short

9. He is scheduled to be the next chairman of the council and, although he does not want to
______ before they are hatched, is rationalizing his commitments.
A. have his cake and eat it C. count his chickens
B. make a meal of D. eat your word

10. The environmental benefits of telecommuting are pretty apparent, but business leaders no
doubt want to be sure that ____ employees _____ to telecommute yields bottom-line benefits.
A. putting / on the map C. catching / on the hop
B. giving / the green light D. winning / back

11. Stephen really lost his ______ when his dental appointment was cancelled again.
A. head B. voice C. calm D. rag

12. We were working overtime to cope with a sudden ______ in demand.


A. boost B. impetus C. surge D. thrust

13. It was decided that the cost of the project would be ______ so it was abandoned.
A. repressive B. prohibitive C. restrictive D. exclusive

14. She was determined to become wealthy and to that ______ she started her own company.
A. view B. aim C. end D. object

15. He made a number of ______ remarks about my cooking, which upset us.
A. slashing B. stabbing C. chopping D. cutting

16. She is afraid she is rather ______ about the existence of the ghost.
A. skeptical B. partial C. adaptable D. incapable

17. I am sorry to have bothered you, I was under the ______ that you wanted me to call you.
A. mistake B. miscalculation C. misconception D. misapprehension

18. Many children who get into trouble in their early teens go on to become ______ offenders.
A. persistent B. insistent C. inverted D. innate

19. ______, Americans eat a light breakfast. They usually don’t eat a lot of food in the morning.
A. By and large B. Fair and square C. Ins and outs D. Odds and ends

20. If that boy doesn’t stop stealing, he will ______ in jail.


A. end up B. bring about C. get round D. go by

Your answers

1. 2. 3. 4. 5. 6. 7. 8. 9. 10.

11. 12. 13. 14. 15. 16. 17. 18. 19. 20.

PART II. WORD FORM (10points)

Complete the following passage with the correct forms of the bold words given in the bracket.

1. When we arrived at the hotel, we were amazed at the _____ hospitality of the staff.
(COMPARE)

2. He is the bad manager in the factory and everyone is in attempt to _____ him. (FAME)

3. The _____ of an epidemic will be unavoidable unless measures are taken to prevent the rural
population from drinking the contaminated water. (BREAK)

4. Please keep the email short. _____ makes everyone’s lives easier. (BRIEF)
5. With the help of the computer, checking information has become less _____. (LABOR)

6. You mustn’t leave your luggage ______ for even a moment on the train. (ATTEND)

7. She looked in on the baby _____ to check that it was all right. (PERIOD)

8. They were totally _____ by the girl’s disappearance. (MYSTERY)

9. The talks were totally ____. We didn’t reach agreement on anything at all. (PRODUCT)

10. The boy was very violent and his parents found him _____. (MANAGE)

Your answers
1. 2. 3. 4. 5.
6. 7. 8. 9. 10.

SECTION C: READING COMPREHENSION (60 points)


Part I: Read the text below and decide which answer A, B, C or D best fits each gap. Write
your answer in the corresponding numbered boxes provided. (10 points)

THE RAVEN

A very large fierce black bird, the raven, has always been (1) _______ with evil women.
But the myths and stories that surround ravens also take account of their unusual intelligence,
their ability to (2) _______ sounds and voices and the way they seem to (3) _______ up a
situation. The fact is, people have never known quite how to (4) _______ the raven. In many
northern myths he was creator of the world, bringer of daylight, but also an aggressive trickster.
Many traditional stories turn on the unpleasant ways in which Raven gets the (5) _______ of a
human adversary.

Legend (6) _______ it that when there are no more ravens in the Tower of London, the
monarchy will fall. In the seventeenth century King Charles II (7) _______ that at least six
ravens should always be kept in the Tower. Today there are seven; six to preserve the monarchy,
and a seventh in (8) _______. To the amusement of tourists, the ravens are officially enlisted as
defenders of the kingdom, and, as is the (9) _______ with soldiers, can be dismissed for
unsatisfactory (10) _______.

1. A. related B. coupled C. associated D. accompanied

2. A. fake B. mimic C. mirror D. simulate


3. A. size B. match C. eye D. catch

4. A. put B. work C. pick D. take

5. A. best B. most C. better D. good

6. A. holds B. states C. has D. keeps

7. A. decreed B. compelled C. required D. enacted

8. A. substitution B. reserve C. continuity D. standby

9. A. truth B. issue C. circumstance D. case


10. A. conduct B. mistake C. action D. carriage
Your answers:
1. _____ 2. _______ 3. __________ 4. ___________ 5. ___________
6. _____ 7. _______ 8. __________ 9. ___________ 10. ___________

Part II. Read the passage below and fill each of the following numbered spaces with ONE
suitable word. Write your answers in the corresponding boxes provided. (10points)

(0) has been done as an example.


SIGNS OF THE TIMES

We are familiar (0) with the saying “a picture paints a thousand words” and in the global
village the world has become, information in pictorial form is (1) _____we turn. Much
communication takes place through symbols rather than words, a case in point (2) ______
airports, where you can see the majority of the thirty-four symbols devised (3) ______ the
American Institute of Graphic Arts in the 1970s. Such signs as a knife and fork for a restaurant
or a telephone for a phone booth are a boon for (4) _______ a traveller who does not speak
English or use the Latin alphabet. (5) _______ worldwide "languages" of this kind are musical
and mathematical notation, circuit diagrams, road signs and computer icons, (6)______, again,
bypass the need for words. Even a label on a garment will carry, in symbols, washing and ironing
instructions. All these (7) ______ to be sufficient to their , (8)______ restricted worlds but would
it really be possible to devise a universal symbolic system of communication independent of any
spoken language, culture-free and value-free, as dreamt of by the seventeenth-century
philosopher Leibniz? It would seem (9) ______. Chinese and Japanese pictograms and ancient
Egyptian hieroglyphics are sometimes cited as examples of such a system, yet both Japanese
script and Egyptian hieroglyphics include sound-base elements and Chinese is often
transliterated (10)______ romanised sound based "pin yin" script. In a word, words are
inescapable.
Your answers:

1. _______ 2. _______ 3. _______ 4. _______ 5. _______


6. _______ 7. _______ 8. _______ 9. _______ 10. _______

Part III. Read the following passage and choose the best answer to each question. (10points)
The first scientific attempt at coaxing moisture from a cloud was in 1946, when scientist
Vincent Schaefer dropped 3 pounds of dry ice from an airplane into a cloud and, to his delight,
produced snow. The success of the experiment was modest, but it spawned optimism among
farmers and ranchers around the country. It seemed to them that science had finally triumphed
over weather.
Unfortunately, it didn’t work out that way. Although there were many cloud-seeding
operations, during the late 1940s and the 1950s, no one could say whether they had any effect on
precipitation. Cloud seeding, or weather modification as it came to be called, was dearly more
complicated than had been thought. It was not until the early 1970s that enough experiments had
been done to understand the processes involved. What these studies indicated was that only
certain types of clouds are amenable to seeding. One of the most responsive is the winter
orographic cloud, formed when air currents encounter a mountain slope and rise. If the
temperature in such a cloud is right, seeding can increase snow yield by 10 to 20 percent.
There are two major methods of weather modification. In one method, silver iodide is
burned in propane-fired ground generators. The smoke rises into the clouds where the tiny silver-
iodide particles act as nuclei for the formation of ice crystals. The alternate system uses airplanes
to deliver dry-ice pellets. Dry ice does not provide ice-forming nuclei. Instead, it lowers the
temperature near the water droplets in the clouds so that they freeze instantly—a process called
spontaneous nucleation. Seeding from aircraft is more efficient but also more expensive.
About 75 percent of all weather modification in the United States takes place in the
Western states. With the population of the West growing rapidly, few regions of the world
require more water. About 85 percent of the waters in the rivers of the West comes from melted
snow. As one expert put it, the water problems of the future may make the energy problems of
the 70s seem like child’s play to solve. That’s why the U.S. Bureau of Reclamation, along with
state governments, municipal water districts, and private interests such as ski areas and
agricultural cooperatives, is putting increased effort into cloud-seeding efforts. Without
consistent and heavy snowfalls in the Rockies and Sierras, the West would literally dry up. The
most intensive efforts to produce precipitation was during the West’s disastrous snow drought of
1976-77. It is impossible to judge the efficiency of weather modification based on one crash
program, but most experts think that such hurry-up programs are not very effective.
1. What is the main subject of the passage?
A. The scientific contributions of Vincent Schaefer
B. Developments in methods of increasing precipitation
C. The process by which snow crystals form
D. The effects of cloud seeding
2. The word spawned in paragraph 1 is closest in meaning to _____.
A. intensified B. reduced C. preceded D. created
3. Which of the following can be inferred from the passage about the term weather
modification?
A. It is not as old as the term cloud seeding. B. It has been in use since at least 1946.
C. It refers to only one type of cloud seeding. D. It was first used by Vincent Schaefer.
4. According to the passage, winter orographic clouds are formed _____.
A. on relatively warm winter days B. over large bodies of water
C. during intense snow storms D. when air currents rise over
mountains
5. To which of the following does the word they in paragraph 3 refer?
A. Water droplets B. Clouds C. Ice-forming nuclei D. Airplanes
6. When clouds are seeded from the ground, what actually causes ice crystals to form?
A. Propane B. Silver-iodide smoke

C. Dry-ice pellets D. Nuclear radiation


7. Clouds would most likely be seeded from airplanes when _____.
A. it is important to save money
B. the process of spontaneous nucleation cannot be employed
C. the production of precipitation must be efficient
D. temperatures are lower than usual
8. What does the author imply about the energy problems of the 1970s?
A. They were caused by a lack of water.
B. They took attention away from water problems.
C. They may not be as critical as water problems will be in the future.
D. They were thought to be minor at the time but turned out to be serious.
9. The author mentions agricultural cooperatives (paragraph 4) as an example of _____.
A. state government agencies
B. private interests
C. organizations that compete with ski areas for water
D. municipal water districts
10. It can be inferred from, the passage that the weather-modification project of 1976-77 was
_____.
A. put together quickly B. a complete failure
C. not necessary D. easy to evaluate
Your answers
1. 2. 3. 4. 5.
6. 7. 8. 9. 10.
Your answers :
1. _______ 2. _______ 3. _______ 4. _______ 5. _______
6. _______ 7. _______ 8. _______ 9. _______ 10. _______
IV. Read the passage and do the tasks. (20 points)
The Search for Extra-terrestrial Intelligence
The question of whether we are alone in the Universe has haunted humanity for centuries, but we
may now stand poised on the brink of the answer to that question, as we search for radio signals
from other intelligent civilisations. This search, often known by the acronym SETI (search for
extra-terrestrial intelligence], is a difficult one. Although groups around the world have been
searching intermittently for three decades, it is only now that we have reached the level of
technology where we can make a determined attempt to search all nearby stars for any sign of
life.
A.
The primary reason for the search is basic curiosity - the same curiosity about the natural world
that drives all pure science. We want to know whether we are alone in the Universe. We want to
know whether life evolves naturally if given the right conditions, or whether there is something
very special about the Earth to have fostered the variety of life forms that we see around us on
the planet. The simple detection of a radio signal will be sufficient to answer this most basic of
all questions. In this sense, SETI is another cog in the machinery of pure science which is
continually pushing out the horizon of our knowledge. However, there are other reasons for
being interested in whether life exists elsewhere. For example, we have had civilization on Earth
for perhaps only a few thousand years, and the threats of nuclear war and pollution over the last
few decades have told us that our survival may be tenuous. Will we last another two thousand
years or will we wipe ourselves out? Since the lifetime of a planet like ours is several billion
years, we can expect that, if other civilizations do survive in our galaxy, their ages will range
from zero to several billion years. Thus any other civilization that we hear from is likely to be far
older, on average, than ourselves. The mere existence of such a civilization will tell us that long-
term survival is possible, and gives us some cause for optimism. It is even possible that the older
civilization may pass on the benefits of their experience in dealing with threats to survival such
as nuclear war and global pollution, and other threats that we haven’t yet discovered.
B.
In discussing whether we are alone, most SETI scientists adopt two ground rules. First, UFQs
(Unidentified Flying Objects) are generally ignored since most scientists don’t consider the
evidence for them to be strong enough to bear serious consideration (although it is also important
to keep an open mind in case any really convincing evidence emerges in the future). Second, we
make a very conservative assumption that we are looking for a life form that is pretty well like
us, since if it differs radically from us we may well not recognize it as a life form, quite apart
from whether we are able to communicate with it. In other words, the life form we are looking
for may well have two green heads and seven fingers, but it will nevertheless resemble us in that
it should communicate with its fellows, be interested in the Universe, live on a planet orbiting a
star like our Sun, and perhaps most restrictively, have a chemistry, like us, based on carbon and
water.
C.
Even when we make these assumptions, our understanding of other life forms is still severely
limited. We do not even know, for example, how many stars have planets, and we certainly do
not know how likely it is that life will arise naturally, given the right conditions. However, when
we look at the 100 billion stars in our galaxy (the Milky Way), and 100 billion galaxies in the
observable Universe, it seems inconceivable that at least one of these planets does not have a life
form on it; in fact, the best educated guess we can make, using the little that we do know about
the conditions for carbon-based life, leads us to estimate that perhaps one in 100,000 stars might
have a life-bearing planet orbiting it. That means that our nearest neighbors are perhaps 100 light
years away, which is almost next door in astronomical terms.
D.
An alien civilization could choose many different ways of sending information across the galaxy,
but many of these either require too much energy, or else are severely attenuated while traversing
the vast distances across the galaxy. It turns out that, for a given amount of transmitted power,
radio waves in the frequency range 1000 to 3000 MHz travel the greatest distance, and so all
searches to date have concentrated on looking for radio waves in this frequency range. So far
there have been a number of searches by various groups around the world, including Australian
searches using the radio telescope at Parkes, New South Wales. Until now there have not been
any detections from the few hundred stars which have been searched. The scale of the searches
has been increased dramatically since 1992, when the US Congress voted NASA $10 million per
year for ten years to conduct a thorough search for extra-terrestrial life. Much of the money in
this project is being spent on developing the special hardware needed to search many frequencies
at once. The project has two parts. One part is a targeted search using the world’s largest radio
telescopes, the American-operated telescope in Arecibo, Puerto Rico and the French telescope in
Nancy in France. This part of the project is searching the nearest 1000 likely stars with high
sensitivity for signals in the frequency range 1000 to 3000 MHz. The other part of the project is
an undirected search which is monitoring all of space with a lower sensitivity, using the smaller
antennas of NASA’s Deep Space Network.

E.
There is considerable debate over how we should react if we detect a signal from an alien
civilization. Everybody agrees that we should not reply immediately. Quite apart from the
impracticality of sending a reply over such large distances at short notice, it raises a host of
ethical questions that would have to be addressed by the global community before any reply
could be sent. Would the human race face the culture shock if faced with a superior and much
older civilization? Luckily, there is no urgency about this. The stars being searched are hundreds
of light years away, so it takes hundreds of years for their signal to reach us, and a further few
hundred years for our reply to reach them. It’s not important, then, if there’s a delay of a few
years, or decades, while the human race debates the question of whether to reply, and perhaps
carefully drafts a reply.
Reading Passage has five paragraphs, A-E.
Choose the correct heading for paragraphs A-E from the list of headings below.
Write the correct number, i-vii, in boxes 1-5.
List of Headings
i. Seeking the transmission of radio signals from planets
ii. Appropriate responses to signals from other civilizations
iii. Vast distances to Earth’s closest neighbors
iv. Assumptions underlying the search for extra-terrestrial intelligence
v. Reasons for the search for extra-terrestrial intelligence
vi. Knowledge of extra-terrestrial life forms
vii. Likelihood of life on other planets
Your answers: 1. Paragraph A ______
2. Paragraph B ______
3. Paragraph C ______
4. Paragraph D ______
5. Paragraph E ______
Questions 6-10
In boxes 6-10, write
YES if the statement agrees with the views of the writer
NO if the statement contradicts the views of the writer
NOT GIVEN if it is impossible to say what the writer thinks about this
6. Alien civilizations may be able to help the human race to overcome serious problems.
7. SETI scientists are trying to find a life form that resembles humans in many ways.
8. The Americans and Australians have co-operated on joint research projects.
9. So far SETI scientists have picked up radio signals from several stars.
10. The NASA project attracted criticism from some members of Congress.
Your answers:6. ______ 7. ______ 8. ______ 9. ______ 10. ______

PART IV. (20points)

Read the following passage. Choose the correct heading for paragraphs A-G from the list of
headings below. Write the correct number, i- ix, in blanks.

SECTION D. WRITING (50 points)

PART I. SENTENCE TRANSFORMATION


1. Finish each of the following sentences in such a way that it is as similar as possible in
meaning to the sentence printed before it. (10points)

1. I won’t go all that way to visit him again on any account!


On no account ___________________________________________________

2. I wasn’t surprised when they refused to pay me.

As I ___________________________________________________________

3. We have credited the money to your current account at this bank.

We have placed __________________________________________________

4. Your silly questions distracted me.

You drove ______________________________________________________

5. Edward eventually organised himself and started work.

Edward eventually got his __________________________________________


2. Rewrite the sentences below using the words in brackets without changing their original
form. (10points)

1. I don’t think this record will ever be popular. (CATCH)

.............................................................................................

2. Mike is never reluctant to make tough decisions as a manager. (SHRINKS)

...........................................................................................

3. You can’t possibly expect me to have supper ready by eight o’clock. (QUESTION)

.............................................................................................

4. It is my opinion that there is no advantage in further discussion. (SEE)

.............................................................................................

5. Please excuse Jane’s poor typing: she’s only been learning for a month. (ALLOWANCES)

.............................................................................................

PART II. PASSAGE WRITING (30points)


Creativity is considered an essential factor to spur development in all aspects of life,
especially in 4.0 Era. In about 250 words, write a passage to suggest ways to promote
creativity at school.

TEST 15
I. LISTENING (50 points)
Section 1. You will hear a woman talking to a man about joining a drama club. Complete the
note below with NO MORE THAN TWO WORDS AND/OR A NUMBER for each answer.
MIDBURY DRAMA CLUB
Background
Example: Answer

⮚ Club started in …1957…

⮚ prize recently won by youth section

⮚ usually performs (1)_______________________ plays

Meetings

⮚ next auditions will be on Tuesday, (2)_______________________

⮚ help is needed with (3)________________ and _______________

⮚ rehearsals take place in the (4)_______________________ hall

⮚ nearest carpark for rehearsals in Ashburton Road opposite the sports center

Costs

⮚ annual membership fee is £ (5)_______________________

⮚ extra payment for (6)_______________________

Contact

⮚ secretary’s name is Sarah (7)_______________________


⮚ secretary’s phone number is (8)_______________________

Section 2: You will hear an interview with Monica Darcey, who has written a bestselling book
about gardening. For questions 1-, choose the best answer (A, B or C).
1 Monica says that most people who buy her book
A have made mistakes in gardening.
B are knowledgeable about gardening
C do not trust professional gardeners
2 How did Monica's parents feel about her early interest in gardening?
A They were concerned about the effects on her health.
B They were worried that she lacked other interests.
C They feared her enthusiasm would affect her studies.
3 Monica applied to work as a gardening journalist because
A it would give her an extra source of income.
B she'd found the experience of writing rewarding.
C there might be opportunities to do some research.
4 Why did Monica give up her job on a magazine?
A She got an offer of work somewhere else.
B She didn't get on with other members of staff.
C She was not interested in the type of work she was doing.
5 According to Monica, what makes her gardening books special?
A They are written in an entertaining style.
B They are aimed at amateur enthusiasts.
C They are the result of detailed research.
6 What does Monica dislike about the photographs in many gardening books?
A They reduce the importance of the writer.
B They help to sell poor quality writing.
C They show an unrealistic view of their subject.
7 What makes Monica unsure whether to accept a job on television?
A Her publisher may disapprove of it.
B It may make her suddenly famous.
C She would have less time for writing.
Your answer
1 2. 3. 4. 5. 6 7
Section 3. Are the sentences true or false?
1. Vegans eat eggs and drink milk.
2. Some people choose to be vegan as a way to be healthier.
3. The study says it’s important that we eat more meat to help the planet.
4. A lot of the earth’s water is used for producing meat.
5. Cows produce six per cent of the world’s carbon emissions.
Section 4
You’ll hear a talk about a model maker. For questions 1-5 complete the sentences with a word or
short phrase.
The Model Maker
Before becoming a model maker, Peter did a course in 1 ……………………………… at a
college.
Peter compares his job to the type of work done by a 2 ………………………………
Most of Peter’s work is exported to and 3 …………….... and ………………….
Peter says his models look best when they have 4 …………………………. directed onto them.
Peter uses watercolour paint to reproduce the effects of the weather and 5
……………………………

II. GRAMMAR AND VOCABULARY (45 points).


1. Choose the best answers to completes each sentence.
1. There is a large effort ________ to rebuild arts education in the New York city public schools.
A. under way B. a long way C. out of the way D. in the way
2. It included 105 job losses and a _____________ in shift work, with some of the large staff
numbers recruited in the past year expected to be let go first.
A. breakdown B. layout C. cutback D. out-take
3. Our teacher tends to ______ certain subjects which she finds difficult to talk about.
A. boil down B. string along C. skate over D. track down
4. I couldn’t remember where I had left my car, when it suddenly ______ me that I didn’t have a
car any longer!
A. dawned on B. ran into C. went through D. tumbled to
5. The manager hesitated to assign the job to the newcomer as he was__________
A. wet behind the ears B. feeling your ears burning
C. ringing in your ears D. keeping your ears open
6. My cousin was nervous about being interviewed on television, but she rose to the _____
wonderfully.
A. event B. performance C. incident D. occasion
7. We need a more _______ assessment of the chances of ending this war.
A. moral B. sober C. plain D. vivid
8. As their bookshop wasn’t doing well, they decided to branch ______and sell compact discs
and cassettes as well.
A. out B. over C. down on D. out for
9. Lisa attempted to ______ herself with her new boss by volunteering to take on extra work.
A. ingratiate B. please C. gratify D. command
10. They continued fighting despite all the ______they met with.
A. adversities B. amenities C. properties D. liabilities
11. I have been back to the doctor three times and he still hasn't ______ the reason for all the
pain I have been suffering from recently.
A. indicated B. highlighted C. pinpointed D. looked
12. I realized ______ that he was a thief.
A. sooner of later B. all along C. at the beginning D. eventually
13. Find someone who will let you talk things through, or _____ that, write down your thoughts.
A. except B. failing C. for all of D. given
14. Karen was terribly nervous before the interview but she managed to pull herself ________
and act confidently.
A. through B. over C. together D. off
15. His flat looks so ______ that it is difficult to believe he just had a party last night.
A. spick and span B. by and large C. safe and sound D. sick and tired
16. Brain cancer requires ______ treatment such as surgery.
A. aggressive B. confrontational C. malignant D. rigorous
17. Harry was offered a scholarship to study in Japan and he _____ the opportunity with both
hands.
A. robbed B. grabbed C. held D. passed grab
18. Until your finances are in the _______, it is not a good idea to take out a loan.
A. credit B. funds C. profit D. black
19. Our firm is so successful because it is at the cutting ______ of computer technology.
A. limit B. fringe C. verge D. edge
20. The most important parts of your job may seem difficult now but they will become second
______ to you within a couple of weeks.
A. instinct B. thought C. nature D. mind

Your answers
1. 2. 3. 4. 5.
6. 7. 8. 9. 10.
11. 12. 13. 14. 15.

2. Use the most suitable form of the words in brackets.


1. The prison service has the twin goals of punishment and ________ (HABIT)
2. They were now faced with seemingly________(MOUNT) technical problems.
3. The teacher said that he found it difficult to cope with a class of ________ (AFFECT)
teenagers.
4. He is ________ (FAIL) polite and tries desperately to understand other people's views.
5. Their refusal to ________ (TAIL) spending plans and to increase the burden on poll tax
payers is expected.
6. ________ (SIMILAR) is consequently difficult, particularly as the minority groups
experience considerable hostility.
7. The new policy only serves to ________ (ACCENT) the inadequacy of provision for the
homeless.
8. And then, to the audience's ________ (MYSTIC), the band suddenly stopped playing.
9. The system of counties was essential to Frankish government, and a count could
wield considerable power, particularly in ________ (FAR) regions.
10. If evolution has________ (WIRE) into us a belief that there are objective moral
obligations, then we will believe that there are.
Your answers
1. 2.
3. 4.
5. 6.
7. 8.
9. 10.
PART IV. READING (45 points).
1. Fill in each blank with one suitable word to complete this passage.
In a village on the east coast of Scotland, people were waiting anxiously for news. Two of
their fishing-boats been caught in the storm (1) ________ had blown up during the night. In the
cottages round the harbour people stood (2) ________ their door, (3) ________ worried to talk.
The rest of the fishing fleet had (4) ________the harbour before dark, and the men from
these ships waited and watched with the wives and families of the (5) ________ men. Some had
brought thick (6) ________ and some hot drinks, knowing that the men (7) ________ be cold
and tired. As dawn began to (8) ________ over in the east, a small point of light was (9)
________ in the darkness of the water and a few minutes later, there was a shout. (10) ________
long, the two boats were turning in, past the lighthouse, to the inside of the harbour. The men
were cold and tired but they were all safe.
Your answers
1. 2. 3. 4. 5.
6. 7. 8. 9. 10.

2. Read the text and decide which answer (A, B, C or D) best fits each space.
Reports that the government is about to (1)_________ the go-ahead to plans for the
building of a new runway at London's Healthrow airport have angered local (2)_________ and
raised fears of increased noise and exhaust pollution. The (3)_________ plans also include a new
sixth terminal building, and (4)________ the disappearance of a whole village, (5)_________ the
demolition up to 700 other homes. According to sources close to the Ministry of Transport, the
government is known to be concerned by the increasing (6)________ of traffic at London
Heathrow. At Gatwick, London's second airport, there are no plans for further runways in the
foreseeable (7)________ and Heathrow is widely (8)________ as a better (9)________ for
expansion. (10)_______ the planned building work would not take place until 2020, local people
have already raised strong objections.
1. A. sign B. make C. give D. approve
2. A. inhabitants B. dwellers C. occupants D. residents
3. A. controversial B. agreeable C. notorious D. doubtful
4. A. involve B. concern C. contain D. need
5. A. further to B. as well as C. moreover D. what's
more
6. A. sum B. size C. volume D. length
7. A. years B. period C. time D. future
8. A. regarded B. believed C. felt D. held
9. A. potential B. outlook C. prospect D. likelihood
10. A. When B. Despite C. Because D. Although
Your answer
1. 2. 3. 4. 5.
6. 7. 8. 9. 10.
3. Read the passage carefully and choose the best answer to the questions
THE PANAMA CANAL
While the Panama Canal continues to stand as one of the greatest human achievements in
history, its locks and canals were not built overnight. Its ultimate completion in 1914 was the
result of decades of planning, preparation, and construction, not to mention loss of human life.
Two countries, France and the United States, were the main players in its construction, and both
faced numerous hardships during the project, so many hardships that France, the instigator of the
project, eventually had to sell out to the US., the country that finally completed the canal. Still,
the United States faced a trio of major hurdles that threatened its completion. These obstacles
were political, environmental, and geographical. Yet, through perseverance and will, the United
States was ultimately able to create the canal, a vital link between the Pacific and Atlantic
Oceans.
One of the major reasons France had to abandon the Panama Canal project was that it
underestimated the environs of the local area. The region of Panama within which the French
worked was a dense, tropical jungle. Intense heat and humidity did not help their situation either.
Before long, many workers began to succumb to diseases like yellow fever and malaria. Proper
measures were not taken to reduce their exposure and vulnerability, and many died as the
workforce and the project as a whole suffered greatly. However, once the Americans took over
the canal project, they immediately implemented better living conditions and infrastructure for
the workforce, including better healthcare facilities. With a stronger workforce and more
extensive healthcare system in place, the Americans stood a better chance of completing the
project than the French ever did.
But, before The United States could continue with the canal project the French had
begun, it had to receive permission from Colombia. At the time, Panama was within the borders
of the country of Colombia. President Theodore Roosevelt offered the Colombian government
ten million dollars, which it immediately rejected. Ever patient, Roosevelt did not press the issue,
and before long, the Panamanians revolted against Colombia for independence. This gave
Roosevelt the opportunity he had been waiting for. He immediately sent in a substantial military
presence to the area to guarantee Panama's independence and to ensure the future construction of
the Panama Canal. With Panama free, the door was open for the Americans to continue building
a canal, which would save 18,000 miles on a trip from San Francisco to New York and open
trade in the Pacific realm.
Once the U.S. was able to get its hands on the area, the next immediate obstacle became a
geological one. While the verdant hills of Panama looked benign enough, the diversity and
makeup of the underlying sediment made it an engineering nightmare. Initially, landslides
regularly destroyed weeks or even months of digging and construction as they did to the French.
Yet, in a stroke or two of engineering brilliance, through the implementation of a system of
dams, this issue was reduced and all but alleviated. Also, as the tidal levels of the Pacific and
Atlantic were vastly different, a new canal system, unlike the sea-level canal attempted by the
French, had to be erected. The American engineers decided to install a system of locks to raise
and lower ships to the designated sea level. This way in which they were able to manipulate
water helped the Americans overcome the tough geological conditions which had thwarted the
French.
Once completed, the Panama Canal stretched for fifty-one miles across Central America,
connecting the Pacific and Atlantic Oceans by sheer human ingenuity and patience. The canal
opened endless new possibilities for trade and commerce between Asia and the Americans,
which still exist today. But the canal did not come about without severe difficulties and tragedy.
It took two countries two separate attempts and over twenty years of backbreaking labor to
achieve. One of these countries, France, had to pack up and go home in failure. The other, the
United States, could relish the milestone it had achieved. Still, in the end, over thirty thousand
men lost their lives directly or indirectly in the building of the Panama Canal, which proves once
and for all what a monumental task it truly was, especially for the age in which it was attempted.

1. The word perseverance in the passage is closest in meaning to _______


A. determination B. procrastination C. cooperation D.
precision
2. According to paragraph 1, which of the following is true of the Panama Canal?
A. It was finally completed in the first decade of the twentieth century.
B. The United States and France worked in unison on its construction.
C. The original construction of the canal faced few difficult issues.
D. The United States eventually purchased the project from France.
3. Which of the following can be inferred from paragraph 2 about the Americans?
A. They tried to prevent the laborers from deserting the construction sites.
B. They destroyed the dense jungle first to eliminate any form of disease.
C. They were able to learn from the shortcomings that had affected France.
D. They were not prepared for the punishing climate in Central America.
4. According to paragraph 3, politics became a problem because _____
A. Roosevelt was not aggressive enough when dealing with the canal
B. the presence of the U.S. military frightened many of the workers
C. Panama wished to remain a territory of the country of Colombia
D. Colombia did not wish to give up the right to the land for the canal
5. In stating that Roosevelt did not press the issue, the author means that Roosevelt did not
A. care about the canal
B. want to force the project
C. resort to the media
D. wish to abandon his goal
6. The author discusses the geological obstacle in paragraph 4 in order to _____
A. note the natural beauty of Panama, which was destroyed by the canal's construction
B. suggest that the workers had to spend a lot of time and effort on reconstruction
C. contrast the geological issues with the oceanic ones the engineers of the project faced
D. show how landslides were more of a problem for the French than they were for the
Americans.
7. The word thwarted in the passage is closest in meaning to _____
A. defeated B. protected C. frustrated D.
destroyed
8. According to paragraph 5, the completion of the Panama Canal
A. created a forty-one-mile link between the Atlantic and Pacific Oceans
B. helped allow for greater economic benefit between the East and West
C. took much longer than original planned by the first French engineers
D. resulted from the United States having better funding than the French
9. Which of the sentences below best expresses the essential information in the highlighted
sentence in the passage?
A. Tens of thousands of men died during the construction of the Panama Canal.
B. Though the project caused many fatalities, it stands as a lofty achievement.
C. The Panama Canal could have been completed later with less loss of life.
D. Men were killed on the job and by residual effects such as disease and injury
10. According to the passage, which of the following is NOT true of the construction of the
Panama Canal?
A. It helped shave thousands and thousands of miles from trips between the eastern and
western United States.
B. it incorporated dams to accelerate the construction process, which helped to make it
successful.
C. It was able to proceed when Panama decided to fight for freedom against Colombia.
D. It was easier to achieve because of the similar tidal levels of the Pacific and Atlantic
Oceans.
4. Read the text carefully and do the task that follow
Television and Sport
when the medium becomes the stadium
A
The relationship between television and sports is not widely thought of as problematic. For many
people, television is a simple medium through which sports can be played, replayed, slowed
down, and of course conveniently transmitted live to homes across the planet. What is often
overlooked, however, is how television networks have reshaped the very foundations of an
industry that they claim only to document. Major television stations immediately seized the
revenue-generating prospects of televising sports and this has changed everything, from how
they are played to who has a chance to watch them.
B
Before television, for example, live matches could only be viewed in person. For the majority of
fans, who were unable to afford tickets to the top-flight matches, or to travel the long distances
required to see them, the only option was to attend a local game instead, where the stakes were
much lower. As a result, thriving social networks and sporting communities formed around the
efforts of teams in the third and fourth divisions and below. With the advent of live TV,
however, premier matches suddenly became affordable and accessible to hundreds of millions of
new viewers. This shift in viewing patterns vacuumed out the support base of local clubs, many
of which ultimately folded.
C
For those on the more prosperous side of this shift in viewing behaviour, however, the financial
rewards are substantial. Television assisted in derailing long-held concerns in many sports about
whether athletes should remain amateurs or ‘go pro’, and replaced this system with a new
paradigm where nearly all athletes are free to pursue stardom and to make money from their
sporting prowess. For the last few decades, top-level sports men and women have signed
lucrative endorsement deals and sponsorship contracts, turning many into multi-millionaires and
also allowing them to focus full-time on what really drives them. That they can do all this
without harming their prospects at the Olympic Games and other major competitions is a
significant benefit for these athletes.
D
The effects of television extend further, however, and in many instances have led to changes in
sporting codes themselves. Prior to televised coverage of the Winter Olympics, for example,
figure skating involved a component in which skaters drew ‘figures’ in the ice, which were later
evaluated for the precision of their shapes. This component translated poorly to the small screen,
as viewers found the whole procedure, including the judging of minute scratches on ice, to be
monotonous and dull. Ultimately, figures were scrapped in favour of a short programme
featuring more telegenic twists and jumps. Other sports are awash with similar regulatory shifts –
passing the ball back to the goalkeeper was banned in football after gameplay at the 1990 World
Cup was deemed overly defensive by television viewers.
E
In addition to insinuating changes into sporting regulation, television also tends to favour some
individual sports over others. Some events, such as the Tour de France, appear to benefit: on
television it can be viewed in its entirety, whereas on-site enthusiasts will only witness a tiny part
of the spectacle. Wrestling, perhaps due to an image problem that repelled younger (and highly
prized) television viewers, was scheduled for removal from the 2020 Olympic Games despite
being a founding sport and a fixture of the Olympics since 708 BC. Only after a fervent outcry
from supporters was that decision overturned.

F
Another change in the sporting landscape that television has triggered is the framing of sports not
merely in terms of the level of skill and athleticism involved, but as personal narratives of
triumph, shame and redemption on the part of individual competitors. This is made easier and
more convincing through the power of close-up camera shots, profiles and commentary shown
during extended build-ups to live events. It also attracts television audiences – particularly
women – who may be less interested in the intricacies of the sport than they are in broader
‘human interest’ stories. As a result, many viewers are now more familiar with the private
agonies of famous athletes than with their record scores or matchday tactics.
G
And what about the effects of male television viewership? Certainly, men have always been
willing to watch male athletes at the top of their game, but female athletes participating in the
same sports have typically attracted far less interest and, as a result, have suffered greatly
reduced exposure on television. Those sports where women can draw the crowds – beach
volleyball, for example – are often those where female participants are encouraged to dress and
behave in ways oriented specifically toward a male demographic.
H
Does all this suggest the influence of television on sports has been overwhelmingly negative?
The answer will almost certainly depend on who among the various stakeholders is asked. For all
those who have lost out – lower-league teams, athletes whose sports lack a certain visual appeal
–there are numerous others who have benefitted enormously from the partnership between
television and sports, and whose livelihoods now depend on it.

Questions 1-7
Reading Passage 3 has eight paragraphs, A–H.
Choose the correct heading for paragraphs A–H from the list of headings below.
Write the correct number, i–xi, in boxes 1-7 on your answer sheet.
List of Headings
i Gender bias in televised sport
ii More money-making opportunities
iii Mixed views on TV’s role in sports
iv Tickets to top matches too expensive
v A common misperception
vi Personal stories become the focus
vii Sports people become stars
viii Rules changed to please viewers
ix Lower-level teams lose out
x Skill levels improve
xi TV appeal influences sports’ success
1 Paragraph B
2 Paragraph C
3 Paragraph D
4 Paragraph E
5 Paragraph F
6 Paragraph G
7 Paragraph H
Questions 8-10
Complete the notes below.
Choose NO MORE THAN TWO WORDS from the passage for each answer.
Write your answers in boxes 8-10 on your answer sheet.
Effect of television on individual sports
Ice skating – viewers find ‘figures’ boring so they are replaced with a 8………………..
Back-passing banned in football.
Tour de France great for TV, but wrestling initially dropped from Olympic Games due to
9………………..
Beach volleyball aimed at 10………………..
V. WRITING ( 50 points)
1. Complete the following sentences. The second sentence must be as close as possible in
meaning to the first. (10 points)
1. I am sure it wasn’t Mrs. Brown you saw yesterday because she had gone abroad.
It can’t _______________________________________________________________
2. Some scientists report that dolphins have a brain capacity larger than human beings’
Dolphins are __________________________________________________________
3. Sue is too slow to understand what you might say.
So slow _____________________________________________________________
4. Although it was expected that he would stand for election, he didn’t.
Contrary to __________________________________________________________
5. The news of the merger came as a complete surprise to the workers.
The workers were taken _______________________________________________
2. Write a new sentence similar in meaning to the given one, using the word given in the
brackets. Do not alter the word in any way. (10 points)
1. His arrival was completely unexpected to us. (took)
His arrival _______________________________________________ surprise.
2. If I help you now, don’t assume I’ll help you next time.(count)
If I help you now, ________________________________________ next time.
3. He owes his life to that surgeon. (indebted)
He is ____________________________________________ for saving his life.
4. Don’t pay any attention when she complains. (notice)
Don’t __________________________________________ when she complains.
5. A rejection of their offer would have been unwise. (accepted)
Not to _____________________________________ would have been unwise.

3. Some people may think that being gifted students is such hard work, even a burden
while others consider it as a challenge, a privilege for one student. What is your opinion?
Support your opinion by using specific reasons & details. Write an essay in about 250
words. (30 points)

You might also like